Concussions in American Football

Article Type
Changed
Thu, 09/19/2019 - 13:25
Display Headline
Concussions in American Football

Football is an important component of American culture, with approximately 3 million youth athletes, 1.1 million high school athletes, and 100,000 college athletes participating each year.1 Participation in football provides athletes with physical, social, psychological, and academic benefits. Despite these benefits, widespread focus has been placed on the safety of football due to the risk for sport-related concussion (SRC) and potentially long-term effects; however, little recognition has been given to the advancements in concussion management across time and occurrence of concussions during most life activities. Although it is reasonable for concerns to be presented, it is important to better understand SRC and the current factors leading to prolonged recoveries, increased risk for injury, and potentially long-term effects.

What Is a Concussion?

Concussions occur after sustaining direct or indirect injury to the head or other parts of the body, as long as the injury force is transmitted to the head. Athletes often experience physical, cognitive, emotional, and sleep-related symptoms post-concussion secondary to an “energy crisis” within the brain.2 The energy crisis occurs as the result of transient neurological dysfunction triggered by changes in the brain (eg, release of neurotransmitters, impaired axonal function).2,3 Concussion is undetectable with traditional imaging; however, advanced imaging techniques (eg, diffuse tensor imaging) have shown progress in assessing axonal injury.3 Symptom duration post-concussion is highly variable due to individual differences; a recent study showed recovery took 3 to 4 weeks for memory and symptoms.4,5

Previous Concussion Management

Identification techniques and return-to-play guidelines for concussion have significantly changed across time. In the past, concussion grading scales were utilized for diagnosis and return to play was possible within the same contest.6,7 It has since been recognized that initial concussion severity makes it difficult to predict recovery.3 For example, research revealed memory decline and increased symptoms 36 hours post-injury for athletes with a grade 1 concussion (ie, transient confusion, no loss of consciousness, concussion symptoms or mental status changes that resolve within 15 minutes of injury) compared to baseline.7 Another study found duration of mental status changes to be related to slower symptom resolution and memory impairment 36 hours to 7 days post-injury.6 Consequently, return to play within the same contest was likely too liberal. Guidelines today recommend immediate removal from play with suspected SRC. Nevertheless, the “play through pain” culture has led athletes to continue playing after SRC, contributing to prolonged recoveries and potentially long-term effects.

Current Concussion Management: Continued Concerns and Areas of Improvement

Despite increased awareness of concussions, recent estimates revealed high rates (ie, 27:1 ratio for general players) of underreporting in college football, particularly amongst offensive linemen.8 Researchers have studied recovery implications for remaining in play, with one study revealing a 2.2 times greater risk for prolonged recovery in college athletes with delayed vs immediate removal.9 Another similar study discovered an 8.8 times greater risk for prolonged recovery in adolescent and young adult athletes not removed vs removed from play.10 Further analysis found remaining in play to be the greatest risk factor for prolonged recovery compared to other previously studied risk factors (eg, age, sex, posttraumatic migraine).10 Additionally, significant differences in neurocognitive data were seen between the “removed” and “not removed” groups for verbal memory, visual memory, processing speed, and reaction time at 1 to 7 days and 8 to 30 days.10 The recovery implications of remaining in play and the additional risk for second impact syndrome (SIS), or repeat concussion when recovering from another injury, emphasizes the need for further education efforts amongst athletes to encourage immediate reporting of injury.11

Sideline Assessment

Sideline assessment has become a vital component of concussion management to rule out concussion and/or significant injury other than concussion. Assessment should include observation, cognitive/balance testing, neurologic examination, and possible exertion testing to ensure a comprehensive evaluation of all areas of potential dysfunction.12 Indications for emergency department evaluation include suspicion for cervical spine injury, intracranial hemorrhage, or skull fracture as well as prolonged loss of consciousness, high-risk mechanisms, posttraumatic seizure(s), and/or significant worsening of symptoms.12

Observation

On the sideline, it is important to identify any immediate signs of injury (ie, loss of consciousness, anterograde/retrograde amnesia, and disorientation/confusion). Since immediate signs are not always present, it is important to be aware of the most commonly reported symptoms, including headache, difficulty concentrating, fatigue, drowsiness, and dizziness.13 If symptoms are not reported by the athlete, balance problems, lack of coordination, increased emotionality, and difficulty following instructions may be observed during play.12

On-Field Assessment

Cognitive and balance testing are essential in determining if an athlete has sustained a concussion. Immediate declines in memory, concentration abilities, and balance abilities are common. Given limitations in administering long testing batteries on the sideline, brief standardized tests such as the Standardized Assessment of Concussion (SAC), Balance Error Scoring System (BESS), and Sport Concussion Assessment Tool (SCAT) are commonly utilized. Identification of cognitive and/or balance abnormalities can help the athlete recognize deficits following injury.12 Balance problems are experienced due to abnormalities in sensory organization and generally resolve during the acute recovery period.14,15 Cognitive difficulties typically persist longer than balance problems, though duration varies widely.

 

 

Neurologic Evaluation

A neurologic evaluation including cranial nerve testing and evaluation of motor-sensory function (ie, assessment for the strength and sensation of upper and lower extremities) is important to identify focal deficits (ie, sensation changes, loss of fine motor control) indicative of serious intracranial pathology.12 Additionally, clinicians have suggested inclusion of vestibular and oculomotor assessments due to frequent dysfunction post-concussion.12,15,16 Examination of the vestibular/oculomotor systems through tools such as the Vestibular/Ocular Motor Screening (VOMS) assessment (assesses both the vestibular and oculomotor systems) and King-Devick Test (primarily assesses saccadic eye movements) can elicit symptoms that may not present immediately. If assessment appears normal, exertion testing can be utilized to determine if symptoms are provoked through physical exercise that should include cardio, dynamic, and sport-specific activities to stress the vestibular system.12

Risk Factors for Injury and Prolonged Recovery

Medical professionals must consider the presence of risk factors when managing concussion in order to make appropriate treatment recommendations and return-to-play decisions. Research has demonstrated the role of female gender, learning disability, attention-deficit/hyperactivity disorder, psychiatric history, young age, motion sickness, sleep problems, somatization, concussion history, on-field dizziness, posttraumatic migraine, and fogginess in increased risk for injury and/or prolonged recovery.17-25 Additionally, athletes with ongoing symptoms from a previous injury are at risk for sustaining another injury.

Acute Home Concussion Management

Although strict rest has been recommended post-concussion, recent research evaluating strict rest vs usual care for adolescents revealed greater symptom reports and longer recovery periods for the strict rest group.26 Based on these findings and emphasis for regulation within the migraine literature (due to the common pathophysiology between migraine and concussion27), we recommend that athletes follow a regulated daily schedule post-concussion including: 1) regular sleep-wake schedule with avoidance of naps, 2) regular meals, 3) adequate fluid hydration, 4) light noncontact physical activity (ie, walking, with progressions recommended by a physician), and 5) stress management techniques. Use of these strategies immediately can help in preventing against increased symptoms and stress, and decreases the need for medication in select cases. Additionally, over-the-counter medications should be limited to 2 to 3 doses per week to avoid rebound headaches.28

In-Office Concussion Management

Athletes diagnosed with SRC will experience different symptoms based on the injury mechanism, risk factors, and management approach. Comprehensive evaluation should include assessment of risk factors, injury details, symptoms, neurocognitive functioning, vestibular/oculomotor dysfunction, tolerance of physical exertion, balance functioning, and cervical spine integrity (if necessary).29,30 Due to individual differences and the heterogeneous symptom profiles, concussion management must move beyond a “one size fits all” approach to avoid nonspecific treatment strategies and consequently prolonged recoveries.29 Clinicians and researchers at University of Pittsburgh Medical Center have identified 6 concussion clinical profiles (ie, vestibular, ocular, posttraumatic migraine, cervical, anxiety/mood, and cognitive/fatigue) that are generally identifiable 48 hours after injury.29,30 Identification of the clinical profile(s) through a comprehensive evaluation guides the development of individualized treatment plans and targeted rehabilitation strategies.29,30

Vestibular. The vestibular system is responsible for stabilizing vision while the head moves and balance control.15 Athletes can experience central and/or peripheral vestibular dysfunction to include benign paroxysmal positional vertigo (BPPV), visual motion sensitivity, vestibular ocular reflex impairment, and balance impairment.30,31 Symptoms typically include dizziness, impaired balance, blurry vision, difficulty focusing, and environmental sensitivity.15,29,30 Potential treatment options include vestibular rehabilitation, exertion therapy, and school/work accommodations.

Ocular. The oculomotor system is responsible for control of eye movements. Athletes can experience many different posttraumatic vision changes, including convergence problems, eye-tracking difficulties, refractive error, difficulty with pursuits/saccades, and accommodation insufficiency. Symptoms typically include light sensitivity, blurred vision, double vision, headaches, fatigue, and memory difficulties.15,29,30 Potential treatment options include vision therapy, vestibular rehabilitation, and school/work accommodations.32

Posttraumatic Migraine. Headache, the most common post-concussion symptom, can persist and meet criteria for posttraumatic migraine (ie, unilateral headache with accompanying nausea and/or photophobia and phonophobia).29,30,33 Implementation of a routine schedule, daily physical activity, exertion therapy, pharmacologic intervention, and school/work accommodations are potential treatment options.

Cervical. The cervical spine can be injured during whiplash-type injuries. Therefore, determining the location, onset, and typical exacerbations of pain can be helpful in identifying cervical involvement.29,30 Symptoms typically include headaches, neck pain, numbness, and tingling. Evaluation and therapy by a certified physical therapist and pharmacologic intervention (eg, muscle relaxants) are potential treatment options. 29,30

Anxiety/Mood. Anxiety, or worry and fear about everyday situations, is common post-concussion and can sometimes be related to ongoing vestibular impairment. Symptoms typically include ruminative thoughts, avoidance of specific situations, hypervigilance, feelings of being overwhelmed, and difficulty falling asleep.29,30 Potential treatment options include implementation of a routine schedule, exposure to provocative situations, psychotherapy, pharmacologic intervention, and school/work accommodations.34

 

 


Cognitive/Fatigue. A global concussion factor (including cognitive, fatigue, and migraine symptoms) has been identified within 1 to 7 days of injury. Although this factor of symptoms generally resolves during the acute recovery period, it persists in select cases.13 Symptoms typically include fatigue, decreased energy levels, nonspecific headaches, potential sleep disruption, increased symptoms towards the end of the day, difficulty concentrating, and increased headache with cognitive activities.29,30,35 Routine schedule, daily physical activity, exertion therapy, pharmacologic intervention (eg, amantadine), and school/work accommodations are potential treatment options.30

Conclusion

Advancements in SRC management warrant change in the conversations regarding concussion in football. Specifically, conversations should address the current understanding of concussion and improvements in the safety of football through stricter concussion guidelines, detailed sideline evaluations, recognition of risk factors, improved acute management, and identification of concussion profiles that help to direct individualized treatment plans and targeted rehabilitation strategies. The biggest concerns related to concussions in football include underreporting of injury, premature return to play, and receiving routine rather than individualized treatment. Therefore, to further improve the safety of football and management of concussion it is essential that future efforts focus on the following 6 areas:

Education: Improved understanding of concussion is imperative to reducing poor outcomes and widespread concerns.

Immediate reporting: Reporting of concussion must be expected and encouraged through consistent responses by coaches to reduce underreporting and fear of reporting in athletes.

Prevention techniques: Athletes must be taught proper form and playing techniques to reduce the risk for concussion. Proper form and technique should be incentivized.

Targeted treatment: Individualized treatment plans and targeted rehabilitation strategies must be developed based on the identified clinical profile(s) to avoid nonspecific treatment recommendations.

Multidisciplinary treatment teams: Given the heterogeneous symptoms profiles and need for care provided by different medical specialties, multidisciplinary teams are essential.

Remain current: With the progress in understanding concussion, providers must remain vigilant of future advances in concussion management to further improve the safety of football.


Am J Orthop. 2016;45(6):352-356. Copyright Frontline Medical Communications Inc. 2016. All rights reserved.

References

1. Dompier TP, Kerr ZY, Marshall SW, et al. Incidence of concussion during practice and games in youth, high school, and collegiate American football players. JAMA Pediatrics. 2015;169(7):659-665.

2. Giza C, Hovda D. The new neurometabolic cascade of concussion. Neurosurgery. 2014;75 Suppl 4:S24-S33.

3. Barkhoudarian G, Hovda DA, Giza CC. The molecular pathophysiology of concussive brain injury - an update. Phys Med Rehabil Clin N Am. 2016;27:373-393.

4. Henry L, Elbin R, Collins M, Marchetti G, Kontos A. Examining recovery trajectories after sport-related concussion with a multimodal clinical assessment approach. Neurosurgery. 2016;78(2):232-241.

5. McCrory P, Meeuwisse WH, Aubry M, et al. Consensus statement on concussion in sport: the 4th International Conference on Concussion in Sport held in Zurich, November 2012. Brit J Sports Med. 2013;47(5):250-258.

6. Lovell MR, Collins MW, Iverson GL, et al. Recovery from mild concussion in high school athletes. J Neurosurg. 2003;98(2):296-301.

7. Lovell MR, Collins MW, Iverson GL, Johnston KM, Bradley JP. Grade 1 or “ding” concussions in high school athletes. Am J Sports Med. 2004;32(1):47-54.

8. Baugh CM, Kroshus E, Daneshvar DH, Filali NA, Hiscox MJ, Glantz LH. Concussion management in United States college sports: compliance with National Collegiate Athletic Association concussion policy and areas for improvement. Am J Sports Med. 2015;43(1):47-56.

9. Asken BM, McCrea MA, Clugston JR, Snyder AR, Houck ZM, Bauer RM. “Playing through it”: Delayed reporting and removal from athletic activity after concussion predicts prolonged recovery. J Athl Train. 2016;51(4):329-335.

10. Elbin RJ, Sufrinko A, Schatz P, et al. Athletes that continue to play with concussion demonstrate worse recovery outcomes than athletes immediately removed from play. J Pediatr. In press.

11. Signoretti S, Lazzarino G, Tavazzi B, Vagnozzi R. The pathophysiology of concussion. PM R. 2011;3(10 Suppl 2):S359-S368.

12. Bloom J, Blount JG. Sideline evaluation of concussion. UpToDate. 2016. http://www.uptodate.com/contents/sideline-evaluation-of-concussion. Accessed July 13, 2016.

13. Kontos AP, Elbin RJ, Schatz P, et al. A revised factor structure for the post-concussion symptom scale: baseline and postconcussion factors. Am J Sports Med. 2012;40(10):2375-2384.

14. Guskiewicz KM, Ross SE, Marshall SW. Postural stability and neuropsychological deficits after concussion in collegiate athletes. J Athl Train. 2001;36(3):263.

15. Mucha A, Collins MW, Elbin R, et al. A brief Vestibular/Ocular Motor Screening (VOMS) assessment to evaluate concussions preliminary findings. Am J Sports Med. 2014;42(10):2479-2486.

16. Bloom J. Vestibular and ocular motor assessments: Important pieces to the concussion puzzle. Athletic Training and Sports Health Care. 2013;5(6):246-248.

17. Covassin T, Elbin R, Harris W, Parker T, Kontos A. The role of age and sex in symptoms, neurocognitive performance, and postural stability in athletes after concussion. Am J Sports Med. 2012;40(6):1303-1312.

18. Kontos A, Sufrinko A, Elbin R, Puskar A, Collins M. Reliability and associated risk factors for performance on the Vestibular/Ocular Motor Screening (VOMS) tool in healthy collegiate athletes. Am J Sports Med. 2016;44(6):1400-1406.

19. Guskiewicz KM, McCrea M, Marshall SW, et al. Cumulative effects associated with recurrent concussion in collegiate football players: the NCAA Concussion Study. JAMA. 2003;290(19):2549-2555.

20. Lau B, Lovell MR, Collins MW, Pardini J. Neurocognitive and symptom predictors of recovery in high school athletes. Clin J Sport Med. 2009;19(3):216-221.

21. Lau BC, Kontos AP, Collins MW, Mucha A, Lovell MR. Which on-field signs/symptoms predict protracted recovery from sport-related concussion among high school football players? Am J Sports Med. 2011;39(11):2311-2318.

22. Mihalik JP, Register-Mihalik J, Kerr ZY, Marshall SW, McCrea MC, Guskiewicz KM. Recovery of posttraumatic migraine characteristics in patients after mild traumatic brain injury. Am J Sports Med. 2013;41(7):1490-1496.

23. Covassin T, Moran R, Elbin RJ. Sex differences in reported concussion injury rates and time loss from participation: An update of the National Collegiate Athletic Association injury surveillance program from 2004-2005 through 2008-2009. J Athl Train. 2016;51(3):189-194.

24. Root JM, Zuckerbraun NS, Wang L, et al. History of somatization is associated with prolonged recovery from concussion. J Pediatr. 2016;174:39-44.

25. Sufrinko A, Pearce K, Elbin RJ, et al. The effect of preinjury sleep difficulties on neurocognitive impairment and symptoms after sport-related concussion. Am J Sports Med. 2015;43(4):830-838.

26. Thomas DG, Apps JN, Hoffmann RG, McCrea M, Hammeke T. Benefits of strict rest after acute concussion: a randomized controlled trial. Pediatrics. 2015;135(2):213-223.

27. Choe M, Blume H. Pediatric posttraumatic Headache: a review. J Child Neurol. 2016;31(1):76-85.

28. Tepper SJ, Tepper DE. Breaking the cycle of medication overuse. Cleve Clin J Med. 2010;77(4):236-242.

29. Collins M, Kontos A, Reynolds E, Murawski C, Fu F. A comprehensive, targeted approach to the clinical care of athletes following sport-related concussion. Knee Surg Sports Traumatol Arthrosc. 2014;22(2):235-246.

30. Reynolds E, Collins MW, Mucha A, Troutman-Ensecki C. Establishing a clinical service for the management of sports-related concussions. Neurosurgery. 2014;75 Suppl 4:S71-S81.

31. Broglio SP, Collins MW, Williams RM, Mucha A, Kontos AP. Current and emerging rehabilitation for concussion: a review of the evidence. Clin Sports Med. 2015;34(2):213-231.

32. Master C, Scheiman M, Gallaway M, et al. Vision diagnoses are common after concussion in adolescents. Clin Pediatr (Phila). 2016;55(3):260-267.

33. Headache Classification Committee of the International Headache Society (IHS). The international classification of headache disorders, 3rd edition (beta version). Cephalalgia. 2013;33(9):629-808.

34. Kontos A, Deitrick JM, Reynolds E. Mental health implication and consequences following sport-related concussion. Brit J Sports Med. 2016;50(3):139-140.

35. Kontos AP, Covassin T, Elbin R, Parker T. Depression and neurocognitive performance after concussion among male and female high school and collegiate athletes. Arch Phys Med Rehabil. 2012;93(10):1751-1756.

Article PDF
Author and Disclosure Information

Authors’ Disclosure Statement: Dr. Collins is a cofounder of ImPACT Applications, Inc. However, the ImPACT test is not the focus of this article. The authors report no actual or potential conflict of interest in relation to this article.

Issue
The American Journal of Orthopedics - 45(6)
Publications
Topics
Page Number
352-356
Sections
Author and Disclosure Information

Authors’ Disclosure Statement: Dr. Collins is a cofounder of ImPACT Applications, Inc. However, the ImPACT test is not the focus of this article. The authors report no actual or potential conflict of interest in relation to this article.

Author and Disclosure Information

Authors’ Disclosure Statement: Dr. Collins is a cofounder of ImPACT Applications, Inc. However, the ImPACT test is not the focus of this article. The authors report no actual or potential conflict of interest in relation to this article.

Article PDF
Article PDF

Football is an important component of American culture, with approximately 3 million youth athletes, 1.1 million high school athletes, and 100,000 college athletes participating each year.1 Participation in football provides athletes with physical, social, psychological, and academic benefits. Despite these benefits, widespread focus has been placed on the safety of football due to the risk for sport-related concussion (SRC) and potentially long-term effects; however, little recognition has been given to the advancements in concussion management across time and occurrence of concussions during most life activities. Although it is reasonable for concerns to be presented, it is important to better understand SRC and the current factors leading to prolonged recoveries, increased risk for injury, and potentially long-term effects.

What Is a Concussion?

Concussions occur after sustaining direct or indirect injury to the head or other parts of the body, as long as the injury force is transmitted to the head. Athletes often experience physical, cognitive, emotional, and sleep-related symptoms post-concussion secondary to an “energy crisis” within the brain.2 The energy crisis occurs as the result of transient neurological dysfunction triggered by changes in the brain (eg, release of neurotransmitters, impaired axonal function).2,3 Concussion is undetectable with traditional imaging; however, advanced imaging techniques (eg, diffuse tensor imaging) have shown progress in assessing axonal injury.3 Symptom duration post-concussion is highly variable due to individual differences; a recent study showed recovery took 3 to 4 weeks for memory and symptoms.4,5

Previous Concussion Management

Identification techniques and return-to-play guidelines for concussion have significantly changed across time. In the past, concussion grading scales were utilized for diagnosis and return to play was possible within the same contest.6,7 It has since been recognized that initial concussion severity makes it difficult to predict recovery.3 For example, research revealed memory decline and increased symptoms 36 hours post-injury for athletes with a grade 1 concussion (ie, transient confusion, no loss of consciousness, concussion symptoms or mental status changes that resolve within 15 minutes of injury) compared to baseline.7 Another study found duration of mental status changes to be related to slower symptom resolution and memory impairment 36 hours to 7 days post-injury.6 Consequently, return to play within the same contest was likely too liberal. Guidelines today recommend immediate removal from play with suspected SRC. Nevertheless, the “play through pain” culture has led athletes to continue playing after SRC, contributing to prolonged recoveries and potentially long-term effects.

Current Concussion Management: Continued Concerns and Areas of Improvement

Despite increased awareness of concussions, recent estimates revealed high rates (ie, 27:1 ratio for general players) of underreporting in college football, particularly amongst offensive linemen.8 Researchers have studied recovery implications for remaining in play, with one study revealing a 2.2 times greater risk for prolonged recovery in college athletes with delayed vs immediate removal.9 Another similar study discovered an 8.8 times greater risk for prolonged recovery in adolescent and young adult athletes not removed vs removed from play.10 Further analysis found remaining in play to be the greatest risk factor for prolonged recovery compared to other previously studied risk factors (eg, age, sex, posttraumatic migraine).10 Additionally, significant differences in neurocognitive data were seen between the “removed” and “not removed” groups for verbal memory, visual memory, processing speed, and reaction time at 1 to 7 days and 8 to 30 days.10 The recovery implications of remaining in play and the additional risk for second impact syndrome (SIS), or repeat concussion when recovering from another injury, emphasizes the need for further education efforts amongst athletes to encourage immediate reporting of injury.11

Sideline Assessment

Sideline assessment has become a vital component of concussion management to rule out concussion and/or significant injury other than concussion. Assessment should include observation, cognitive/balance testing, neurologic examination, and possible exertion testing to ensure a comprehensive evaluation of all areas of potential dysfunction.12 Indications for emergency department evaluation include suspicion for cervical spine injury, intracranial hemorrhage, or skull fracture as well as prolonged loss of consciousness, high-risk mechanisms, posttraumatic seizure(s), and/or significant worsening of symptoms.12

Observation

On the sideline, it is important to identify any immediate signs of injury (ie, loss of consciousness, anterograde/retrograde amnesia, and disorientation/confusion). Since immediate signs are not always present, it is important to be aware of the most commonly reported symptoms, including headache, difficulty concentrating, fatigue, drowsiness, and dizziness.13 If symptoms are not reported by the athlete, balance problems, lack of coordination, increased emotionality, and difficulty following instructions may be observed during play.12

On-Field Assessment

Cognitive and balance testing are essential in determining if an athlete has sustained a concussion. Immediate declines in memory, concentration abilities, and balance abilities are common. Given limitations in administering long testing batteries on the sideline, brief standardized tests such as the Standardized Assessment of Concussion (SAC), Balance Error Scoring System (BESS), and Sport Concussion Assessment Tool (SCAT) are commonly utilized. Identification of cognitive and/or balance abnormalities can help the athlete recognize deficits following injury.12 Balance problems are experienced due to abnormalities in sensory organization and generally resolve during the acute recovery period.14,15 Cognitive difficulties typically persist longer than balance problems, though duration varies widely.

 

 

Neurologic Evaluation

A neurologic evaluation including cranial nerve testing and evaluation of motor-sensory function (ie, assessment for the strength and sensation of upper and lower extremities) is important to identify focal deficits (ie, sensation changes, loss of fine motor control) indicative of serious intracranial pathology.12 Additionally, clinicians have suggested inclusion of vestibular and oculomotor assessments due to frequent dysfunction post-concussion.12,15,16 Examination of the vestibular/oculomotor systems through tools such as the Vestibular/Ocular Motor Screening (VOMS) assessment (assesses both the vestibular and oculomotor systems) and King-Devick Test (primarily assesses saccadic eye movements) can elicit symptoms that may not present immediately. If assessment appears normal, exertion testing can be utilized to determine if symptoms are provoked through physical exercise that should include cardio, dynamic, and sport-specific activities to stress the vestibular system.12

Risk Factors for Injury and Prolonged Recovery

Medical professionals must consider the presence of risk factors when managing concussion in order to make appropriate treatment recommendations and return-to-play decisions. Research has demonstrated the role of female gender, learning disability, attention-deficit/hyperactivity disorder, psychiatric history, young age, motion sickness, sleep problems, somatization, concussion history, on-field dizziness, posttraumatic migraine, and fogginess in increased risk for injury and/or prolonged recovery.17-25 Additionally, athletes with ongoing symptoms from a previous injury are at risk for sustaining another injury.

Acute Home Concussion Management

Although strict rest has been recommended post-concussion, recent research evaluating strict rest vs usual care for adolescents revealed greater symptom reports and longer recovery periods for the strict rest group.26 Based on these findings and emphasis for regulation within the migraine literature (due to the common pathophysiology between migraine and concussion27), we recommend that athletes follow a regulated daily schedule post-concussion including: 1) regular sleep-wake schedule with avoidance of naps, 2) regular meals, 3) adequate fluid hydration, 4) light noncontact physical activity (ie, walking, with progressions recommended by a physician), and 5) stress management techniques. Use of these strategies immediately can help in preventing against increased symptoms and stress, and decreases the need for medication in select cases. Additionally, over-the-counter medications should be limited to 2 to 3 doses per week to avoid rebound headaches.28

In-Office Concussion Management

Athletes diagnosed with SRC will experience different symptoms based on the injury mechanism, risk factors, and management approach. Comprehensive evaluation should include assessment of risk factors, injury details, symptoms, neurocognitive functioning, vestibular/oculomotor dysfunction, tolerance of physical exertion, balance functioning, and cervical spine integrity (if necessary).29,30 Due to individual differences and the heterogeneous symptom profiles, concussion management must move beyond a “one size fits all” approach to avoid nonspecific treatment strategies and consequently prolonged recoveries.29 Clinicians and researchers at University of Pittsburgh Medical Center have identified 6 concussion clinical profiles (ie, vestibular, ocular, posttraumatic migraine, cervical, anxiety/mood, and cognitive/fatigue) that are generally identifiable 48 hours after injury.29,30 Identification of the clinical profile(s) through a comprehensive evaluation guides the development of individualized treatment plans and targeted rehabilitation strategies.29,30

Vestibular. The vestibular system is responsible for stabilizing vision while the head moves and balance control.15 Athletes can experience central and/or peripheral vestibular dysfunction to include benign paroxysmal positional vertigo (BPPV), visual motion sensitivity, vestibular ocular reflex impairment, and balance impairment.30,31 Symptoms typically include dizziness, impaired balance, blurry vision, difficulty focusing, and environmental sensitivity.15,29,30 Potential treatment options include vestibular rehabilitation, exertion therapy, and school/work accommodations.

Ocular. The oculomotor system is responsible for control of eye movements. Athletes can experience many different posttraumatic vision changes, including convergence problems, eye-tracking difficulties, refractive error, difficulty with pursuits/saccades, and accommodation insufficiency. Symptoms typically include light sensitivity, blurred vision, double vision, headaches, fatigue, and memory difficulties.15,29,30 Potential treatment options include vision therapy, vestibular rehabilitation, and school/work accommodations.32

Posttraumatic Migraine. Headache, the most common post-concussion symptom, can persist and meet criteria for posttraumatic migraine (ie, unilateral headache with accompanying nausea and/or photophobia and phonophobia).29,30,33 Implementation of a routine schedule, daily physical activity, exertion therapy, pharmacologic intervention, and school/work accommodations are potential treatment options.

Cervical. The cervical spine can be injured during whiplash-type injuries. Therefore, determining the location, onset, and typical exacerbations of pain can be helpful in identifying cervical involvement.29,30 Symptoms typically include headaches, neck pain, numbness, and tingling. Evaluation and therapy by a certified physical therapist and pharmacologic intervention (eg, muscle relaxants) are potential treatment options. 29,30

Anxiety/Mood. Anxiety, or worry and fear about everyday situations, is common post-concussion and can sometimes be related to ongoing vestibular impairment. Symptoms typically include ruminative thoughts, avoidance of specific situations, hypervigilance, feelings of being overwhelmed, and difficulty falling asleep.29,30 Potential treatment options include implementation of a routine schedule, exposure to provocative situations, psychotherapy, pharmacologic intervention, and school/work accommodations.34

 

 


Cognitive/Fatigue. A global concussion factor (including cognitive, fatigue, and migraine symptoms) has been identified within 1 to 7 days of injury. Although this factor of symptoms generally resolves during the acute recovery period, it persists in select cases.13 Symptoms typically include fatigue, decreased energy levels, nonspecific headaches, potential sleep disruption, increased symptoms towards the end of the day, difficulty concentrating, and increased headache with cognitive activities.29,30,35 Routine schedule, daily physical activity, exertion therapy, pharmacologic intervention (eg, amantadine), and school/work accommodations are potential treatment options.30

Conclusion

Advancements in SRC management warrant change in the conversations regarding concussion in football. Specifically, conversations should address the current understanding of concussion and improvements in the safety of football through stricter concussion guidelines, detailed sideline evaluations, recognition of risk factors, improved acute management, and identification of concussion profiles that help to direct individualized treatment plans and targeted rehabilitation strategies. The biggest concerns related to concussions in football include underreporting of injury, premature return to play, and receiving routine rather than individualized treatment. Therefore, to further improve the safety of football and management of concussion it is essential that future efforts focus on the following 6 areas:

Education: Improved understanding of concussion is imperative to reducing poor outcomes and widespread concerns.

Immediate reporting: Reporting of concussion must be expected and encouraged through consistent responses by coaches to reduce underreporting and fear of reporting in athletes.

Prevention techniques: Athletes must be taught proper form and playing techniques to reduce the risk for concussion. Proper form and technique should be incentivized.

Targeted treatment: Individualized treatment plans and targeted rehabilitation strategies must be developed based on the identified clinical profile(s) to avoid nonspecific treatment recommendations.

Multidisciplinary treatment teams: Given the heterogeneous symptoms profiles and need for care provided by different medical specialties, multidisciplinary teams are essential.

Remain current: With the progress in understanding concussion, providers must remain vigilant of future advances in concussion management to further improve the safety of football.


Am J Orthop. 2016;45(6):352-356. Copyright Frontline Medical Communications Inc. 2016. All rights reserved.

Football is an important component of American culture, with approximately 3 million youth athletes, 1.1 million high school athletes, and 100,000 college athletes participating each year.1 Participation in football provides athletes with physical, social, psychological, and academic benefits. Despite these benefits, widespread focus has been placed on the safety of football due to the risk for sport-related concussion (SRC) and potentially long-term effects; however, little recognition has been given to the advancements in concussion management across time and occurrence of concussions during most life activities. Although it is reasonable for concerns to be presented, it is important to better understand SRC and the current factors leading to prolonged recoveries, increased risk for injury, and potentially long-term effects.

What Is a Concussion?

Concussions occur after sustaining direct or indirect injury to the head or other parts of the body, as long as the injury force is transmitted to the head. Athletes often experience physical, cognitive, emotional, and sleep-related symptoms post-concussion secondary to an “energy crisis” within the brain.2 The energy crisis occurs as the result of transient neurological dysfunction triggered by changes in the brain (eg, release of neurotransmitters, impaired axonal function).2,3 Concussion is undetectable with traditional imaging; however, advanced imaging techniques (eg, diffuse tensor imaging) have shown progress in assessing axonal injury.3 Symptom duration post-concussion is highly variable due to individual differences; a recent study showed recovery took 3 to 4 weeks for memory and symptoms.4,5

Previous Concussion Management

Identification techniques and return-to-play guidelines for concussion have significantly changed across time. In the past, concussion grading scales were utilized for diagnosis and return to play was possible within the same contest.6,7 It has since been recognized that initial concussion severity makes it difficult to predict recovery.3 For example, research revealed memory decline and increased symptoms 36 hours post-injury for athletes with a grade 1 concussion (ie, transient confusion, no loss of consciousness, concussion symptoms or mental status changes that resolve within 15 minutes of injury) compared to baseline.7 Another study found duration of mental status changes to be related to slower symptom resolution and memory impairment 36 hours to 7 days post-injury.6 Consequently, return to play within the same contest was likely too liberal. Guidelines today recommend immediate removal from play with suspected SRC. Nevertheless, the “play through pain” culture has led athletes to continue playing after SRC, contributing to prolonged recoveries and potentially long-term effects.

Current Concussion Management: Continued Concerns and Areas of Improvement

Despite increased awareness of concussions, recent estimates revealed high rates (ie, 27:1 ratio for general players) of underreporting in college football, particularly amongst offensive linemen.8 Researchers have studied recovery implications for remaining in play, with one study revealing a 2.2 times greater risk for prolonged recovery in college athletes with delayed vs immediate removal.9 Another similar study discovered an 8.8 times greater risk for prolonged recovery in adolescent and young adult athletes not removed vs removed from play.10 Further analysis found remaining in play to be the greatest risk factor for prolonged recovery compared to other previously studied risk factors (eg, age, sex, posttraumatic migraine).10 Additionally, significant differences in neurocognitive data were seen between the “removed” and “not removed” groups for verbal memory, visual memory, processing speed, and reaction time at 1 to 7 days and 8 to 30 days.10 The recovery implications of remaining in play and the additional risk for second impact syndrome (SIS), or repeat concussion when recovering from another injury, emphasizes the need for further education efforts amongst athletes to encourage immediate reporting of injury.11

Sideline Assessment

Sideline assessment has become a vital component of concussion management to rule out concussion and/or significant injury other than concussion. Assessment should include observation, cognitive/balance testing, neurologic examination, and possible exertion testing to ensure a comprehensive evaluation of all areas of potential dysfunction.12 Indications for emergency department evaluation include suspicion for cervical spine injury, intracranial hemorrhage, or skull fracture as well as prolonged loss of consciousness, high-risk mechanisms, posttraumatic seizure(s), and/or significant worsening of symptoms.12

Observation

On the sideline, it is important to identify any immediate signs of injury (ie, loss of consciousness, anterograde/retrograde amnesia, and disorientation/confusion). Since immediate signs are not always present, it is important to be aware of the most commonly reported symptoms, including headache, difficulty concentrating, fatigue, drowsiness, and dizziness.13 If symptoms are not reported by the athlete, balance problems, lack of coordination, increased emotionality, and difficulty following instructions may be observed during play.12

On-Field Assessment

Cognitive and balance testing are essential in determining if an athlete has sustained a concussion. Immediate declines in memory, concentration abilities, and balance abilities are common. Given limitations in administering long testing batteries on the sideline, brief standardized tests such as the Standardized Assessment of Concussion (SAC), Balance Error Scoring System (BESS), and Sport Concussion Assessment Tool (SCAT) are commonly utilized. Identification of cognitive and/or balance abnormalities can help the athlete recognize deficits following injury.12 Balance problems are experienced due to abnormalities in sensory organization and generally resolve during the acute recovery period.14,15 Cognitive difficulties typically persist longer than balance problems, though duration varies widely.

 

 

Neurologic Evaluation

A neurologic evaluation including cranial nerve testing and evaluation of motor-sensory function (ie, assessment for the strength and sensation of upper and lower extremities) is important to identify focal deficits (ie, sensation changes, loss of fine motor control) indicative of serious intracranial pathology.12 Additionally, clinicians have suggested inclusion of vestibular and oculomotor assessments due to frequent dysfunction post-concussion.12,15,16 Examination of the vestibular/oculomotor systems through tools such as the Vestibular/Ocular Motor Screening (VOMS) assessment (assesses both the vestibular and oculomotor systems) and King-Devick Test (primarily assesses saccadic eye movements) can elicit symptoms that may not present immediately. If assessment appears normal, exertion testing can be utilized to determine if symptoms are provoked through physical exercise that should include cardio, dynamic, and sport-specific activities to stress the vestibular system.12

Risk Factors for Injury and Prolonged Recovery

Medical professionals must consider the presence of risk factors when managing concussion in order to make appropriate treatment recommendations and return-to-play decisions. Research has demonstrated the role of female gender, learning disability, attention-deficit/hyperactivity disorder, psychiatric history, young age, motion sickness, sleep problems, somatization, concussion history, on-field dizziness, posttraumatic migraine, and fogginess in increased risk for injury and/or prolonged recovery.17-25 Additionally, athletes with ongoing symptoms from a previous injury are at risk for sustaining another injury.

Acute Home Concussion Management

Although strict rest has been recommended post-concussion, recent research evaluating strict rest vs usual care for adolescents revealed greater symptom reports and longer recovery periods for the strict rest group.26 Based on these findings and emphasis for regulation within the migraine literature (due to the common pathophysiology between migraine and concussion27), we recommend that athletes follow a regulated daily schedule post-concussion including: 1) regular sleep-wake schedule with avoidance of naps, 2) regular meals, 3) adequate fluid hydration, 4) light noncontact physical activity (ie, walking, with progressions recommended by a physician), and 5) stress management techniques. Use of these strategies immediately can help in preventing against increased symptoms and stress, and decreases the need for medication in select cases. Additionally, over-the-counter medications should be limited to 2 to 3 doses per week to avoid rebound headaches.28

In-Office Concussion Management

Athletes diagnosed with SRC will experience different symptoms based on the injury mechanism, risk factors, and management approach. Comprehensive evaluation should include assessment of risk factors, injury details, symptoms, neurocognitive functioning, vestibular/oculomotor dysfunction, tolerance of physical exertion, balance functioning, and cervical spine integrity (if necessary).29,30 Due to individual differences and the heterogeneous symptom profiles, concussion management must move beyond a “one size fits all” approach to avoid nonspecific treatment strategies and consequently prolonged recoveries.29 Clinicians and researchers at University of Pittsburgh Medical Center have identified 6 concussion clinical profiles (ie, vestibular, ocular, posttraumatic migraine, cervical, anxiety/mood, and cognitive/fatigue) that are generally identifiable 48 hours after injury.29,30 Identification of the clinical profile(s) through a comprehensive evaluation guides the development of individualized treatment plans and targeted rehabilitation strategies.29,30

Vestibular. The vestibular system is responsible for stabilizing vision while the head moves and balance control.15 Athletes can experience central and/or peripheral vestibular dysfunction to include benign paroxysmal positional vertigo (BPPV), visual motion sensitivity, vestibular ocular reflex impairment, and balance impairment.30,31 Symptoms typically include dizziness, impaired balance, blurry vision, difficulty focusing, and environmental sensitivity.15,29,30 Potential treatment options include vestibular rehabilitation, exertion therapy, and school/work accommodations.

Ocular. The oculomotor system is responsible for control of eye movements. Athletes can experience many different posttraumatic vision changes, including convergence problems, eye-tracking difficulties, refractive error, difficulty with pursuits/saccades, and accommodation insufficiency. Symptoms typically include light sensitivity, blurred vision, double vision, headaches, fatigue, and memory difficulties.15,29,30 Potential treatment options include vision therapy, vestibular rehabilitation, and school/work accommodations.32

Posttraumatic Migraine. Headache, the most common post-concussion symptom, can persist and meet criteria for posttraumatic migraine (ie, unilateral headache with accompanying nausea and/or photophobia and phonophobia).29,30,33 Implementation of a routine schedule, daily physical activity, exertion therapy, pharmacologic intervention, and school/work accommodations are potential treatment options.

Cervical. The cervical spine can be injured during whiplash-type injuries. Therefore, determining the location, onset, and typical exacerbations of pain can be helpful in identifying cervical involvement.29,30 Symptoms typically include headaches, neck pain, numbness, and tingling. Evaluation and therapy by a certified physical therapist and pharmacologic intervention (eg, muscle relaxants) are potential treatment options. 29,30

Anxiety/Mood. Anxiety, or worry and fear about everyday situations, is common post-concussion and can sometimes be related to ongoing vestibular impairment. Symptoms typically include ruminative thoughts, avoidance of specific situations, hypervigilance, feelings of being overwhelmed, and difficulty falling asleep.29,30 Potential treatment options include implementation of a routine schedule, exposure to provocative situations, psychotherapy, pharmacologic intervention, and school/work accommodations.34

 

 


Cognitive/Fatigue. A global concussion factor (including cognitive, fatigue, and migraine symptoms) has been identified within 1 to 7 days of injury. Although this factor of symptoms generally resolves during the acute recovery period, it persists in select cases.13 Symptoms typically include fatigue, decreased energy levels, nonspecific headaches, potential sleep disruption, increased symptoms towards the end of the day, difficulty concentrating, and increased headache with cognitive activities.29,30,35 Routine schedule, daily physical activity, exertion therapy, pharmacologic intervention (eg, amantadine), and school/work accommodations are potential treatment options.30

Conclusion

Advancements in SRC management warrant change in the conversations regarding concussion in football. Specifically, conversations should address the current understanding of concussion and improvements in the safety of football through stricter concussion guidelines, detailed sideline evaluations, recognition of risk factors, improved acute management, and identification of concussion profiles that help to direct individualized treatment plans and targeted rehabilitation strategies. The biggest concerns related to concussions in football include underreporting of injury, premature return to play, and receiving routine rather than individualized treatment. Therefore, to further improve the safety of football and management of concussion it is essential that future efforts focus on the following 6 areas:

Education: Improved understanding of concussion is imperative to reducing poor outcomes and widespread concerns.

Immediate reporting: Reporting of concussion must be expected and encouraged through consistent responses by coaches to reduce underreporting and fear of reporting in athletes.

Prevention techniques: Athletes must be taught proper form and playing techniques to reduce the risk for concussion. Proper form and technique should be incentivized.

Targeted treatment: Individualized treatment plans and targeted rehabilitation strategies must be developed based on the identified clinical profile(s) to avoid nonspecific treatment recommendations.

Multidisciplinary treatment teams: Given the heterogeneous symptoms profiles and need for care provided by different medical specialties, multidisciplinary teams are essential.

Remain current: With the progress in understanding concussion, providers must remain vigilant of future advances in concussion management to further improve the safety of football.


Am J Orthop. 2016;45(6):352-356. Copyright Frontline Medical Communications Inc. 2016. All rights reserved.

References

1. Dompier TP, Kerr ZY, Marshall SW, et al. Incidence of concussion during practice and games in youth, high school, and collegiate American football players. JAMA Pediatrics. 2015;169(7):659-665.

2. Giza C, Hovda D. The new neurometabolic cascade of concussion. Neurosurgery. 2014;75 Suppl 4:S24-S33.

3. Barkhoudarian G, Hovda DA, Giza CC. The molecular pathophysiology of concussive brain injury - an update. Phys Med Rehabil Clin N Am. 2016;27:373-393.

4. Henry L, Elbin R, Collins M, Marchetti G, Kontos A. Examining recovery trajectories after sport-related concussion with a multimodal clinical assessment approach. Neurosurgery. 2016;78(2):232-241.

5. McCrory P, Meeuwisse WH, Aubry M, et al. Consensus statement on concussion in sport: the 4th International Conference on Concussion in Sport held in Zurich, November 2012. Brit J Sports Med. 2013;47(5):250-258.

6. Lovell MR, Collins MW, Iverson GL, et al. Recovery from mild concussion in high school athletes. J Neurosurg. 2003;98(2):296-301.

7. Lovell MR, Collins MW, Iverson GL, Johnston KM, Bradley JP. Grade 1 or “ding” concussions in high school athletes. Am J Sports Med. 2004;32(1):47-54.

8. Baugh CM, Kroshus E, Daneshvar DH, Filali NA, Hiscox MJ, Glantz LH. Concussion management in United States college sports: compliance with National Collegiate Athletic Association concussion policy and areas for improvement. Am J Sports Med. 2015;43(1):47-56.

9. Asken BM, McCrea MA, Clugston JR, Snyder AR, Houck ZM, Bauer RM. “Playing through it”: Delayed reporting and removal from athletic activity after concussion predicts prolonged recovery. J Athl Train. 2016;51(4):329-335.

10. Elbin RJ, Sufrinko A, Schatz P, et al. Athletes that continue to play with concussion demonstrate worse recovery outcomes than athletes immediately removed from play. J Pediatr. In press.

11. Signoretti S, Lazzarino G, Tavazzi B, Vagnozzi R. The pathophysiology of concussion. PM R. 2011;3(10 Suppl 2):S359-S368.

12. Bloom J, Blount JG. Sideline evaluation of concussion. UpToDate. 2016. http://www.uptodate.com/contents/sideline-evaluation-of-concussion. Accessed July 13, 2016.

13. Kontos AP, Elbin RJ, Schatz P, et al. A revised factor structure for the post-concussion symptom scale: baseline and postconcussion factors. Am J Sports Med. 2012;40(10):2375-2384.

14. Guskiewicz KM, Ross SE, Marshall SW. Postural stability and neuropsychological deficits after concussion in collegiate athletes. J Athl Train. 2001;36(3):263.

15. Mucha A, Collins MW, Elbin R, et al. A brief Vestibular/Ocular Motor Screening (VOMS) assessment to evaluate concussions preliminary findings. Am J Sports Med. 2014;42(10):2479-2486.

16. Bloom J. Vestibular and ocular motor assessments: Important pieces to the concussion puzzle. Athletic Training and Sports Health Care. 2013;5(6):246-248.

17. Covassin T, Elbin R, Harris W, Parker T, Kontos A. The role of age and sex in symptoms, neurocognitive performance, and postural stability in athletes after concussion. Am J Sports Med. 2012;40(6):1303-1312.

18. Kontos A, Sufrinko A, Elbin R, Puskar A, Collins M. Reliability and associated risk factors for performance on the Vestibular/Ocular Motor Screening (VOMS) tool in healthy collegiate athletes. Am J Sports Med. 2016;44(6):1400-1406.

19. Guskiewicz KM, McCrea M, Marshall SW, et al. Cumulative effects associated with recurrent concussion in collegiate football players: the NCAA Concussion Study. JAMA. 2003;290(19):2549-2555.

20. Lau B, Lovell MR, Collins MW, Pardini J. Neurocognitive and symptom predictors of recovery in high school athletes. Clin J Sport Med. 2009;19(3):216-221.

21. Lau BC, Kontos AP, Collins MW, Mucha A, Lovell MR. Which on-field signs/symptoms predict protracted recovery from sport-related concussion among high school football players? Am J Sports Med. 2011;39(11):2311-2318.

22. Mihalik JP, Register-Mihalik J, Kerr ZY, Marshall SW, McCrea MC, Guskiewicz KM. Recovery of posttraumatic migraine characteristics in patients after mild traumatic brain injury. Am J Sports Med. 2013;41(7):1490-1496.

23. Covassin T, Moran R, Elbin RJ. Sex differences in reported concussion injury rates and time loss from participation: An update of the National Collegiate Athletic Association injury surveillance program from 2004-2005 through 2008-2009. J Athl Train. 2016;51(3):189-194.

24. Root JM, Zuckerbraun NS, Wang L, et al. History of somatization is associated with prolonged recovery from concussion. J Pediatr. 2016;174:39-44.

25. Sufrinko A, Pearce K, Elbin RJ, et al. The effect of preinjury sleep difficulties on neurocognitive impairment and symptoms after sport-related concussion. Am J Sports Med. 2015;43(4):830-838.

26. Thomas DG, Apps JN, Hoffmann RG, McCrea M, Hammeke T. Benefits of strict rest after acute concussion: a randomized controlled trial. Pediatrics. 2015;135(2):213-223.

27. Choe M, Blume H. Pediatric posttraumatic Headache: a review. J Child Neurol. 2016;31(1):76-85.

28. Tepper SJ, Tepper DE. Breaking the cycle of medication overuse. Cleve Clin J Med. 2010;77(4):236-242.

29. Collins M, Kontos A, Reynolds E, Murawski C, Fu F. A comprehensive, targeted approach to the clinical care of athletes following sport-related concussion. Knee Surg Sports Traumatol Arthrosc. 2014;22(2):235-246.

30. Reynolds E, Collins MW, Mucha A, Troutman-Ensecki C. Establishing a clinical service for the management of sports-related concussions. Neurosurgery. 2014;75 Suppl 4:S71-S81.

31. Broglio SP, Collins MW, Williams RM, Mucha A, Kontos AP. Current and emerging rehabilitation for concussion: a review of the evidence. Clin Sports Med. 2015;34(2):213-231.

32. Master C, Scheiman M, Gallaway M, et al. Vision diagnoses are common after concussion in adolescents. Clin Pediatr (Phila). 2016;55(3):260-267.

33. Headache Classification Committee of the International Headache Society (IHS). The international classification of headache disorders, 3rd edition (beta version). Cephalalgia. 2013;33(9):629-808.

34. Kontos A, Deitrick JM, Reynolds E. Mental health implication and consequences following sport-related concussion. Brit J Sports Med. 2016;50(3):139-140.

35. Kontos AP, Covassin T, Elbin R, Parker T. Depression and neurocognitive performance after concussion among male and female high school and collegiate athletes. Arch Phys Med Rehabil. 2012;93(10):1751-1756.

References

1. Dompier TP, Kerr ZY, Marshall SW, et al. Incidence of concussion during practice and games in youth, high school, and collegiate American football players. JAMA Pediatrics. 2015;169(7):659-665.

2. Giza C, Hovda D. The new neurometabolic cascade of concussion. Neurosurgery. 2014;75 Suppl 4:S24-S33.

3. Barkhoudarian G, Hovda DA, Giza CC. The molecular pathophysiology of concussive brain injury - an update. Phys Med Rehabil Clin N Am. 2016;27:373-393.

4. Henry L, Elbin R, Collins M, Marchetti G, Kontos A. Examining recovery trajectories after sport-related concussion with a multimodal clinical assessment approach. Neurosurgery. 2016;78(2):232-241.

5. McCrory P, Meeuwisse WH, Aubry M, et al. Consensus statement on concussion in sport: the 4th International Conference on Concussion in Sport held in Zurich, November 2012. Brit J Sports Med. 2013;47(5):250-258.

6. Lovell MR, Collins MW, Iverson GL, et al. Recovery from mild concussion in high school athletes. J Neurosurg. 2003;98(2):296-301.

7. Lovell MR, Collins MW, Iverson GL, Johnston KM, Bradley JP. Grade 1 or “ding” concussions in high school athletes. Am J Sports Med. 2004;32(1):47-54.

8. Baugh CM, Kroshus E, Daneshvar DH, Filali NA, Hiscox MJ, Glantz LH. Concussion management in United States college sports: compliance with National Collegiate Athletic Association concussion policy and areas for improvement. Am J Sports Med. 2015;43(1):47-56.

9. Asken BM, McCrea MA, Clugston JR, Snyder AR, Houck ZM, Bauer RM. “Playing through it”: Delayed reporting and removal from athletic activity after concussion predicts prolonged recovery. J Athl Train. 2016;51(4):329-335.

10. Elbin RJ, Sufrinko A, Schatz P, et al. Athletes that continue to play with concussion demonstrate worse recovery outcomes than athletes immediately removed from play. J Pediatr. In press.

11. Signoretti S, Lazzarino G, Tavazzi B, Vagnozzi R. The pathophysiology of concussion. PM R. 2011;3(10 Suppl 2):S359-S368.

12. Bloom J, Blount JG. Sideline evaluation of concussion. UpToDate. 2016. http://www.uptodate.com/contents/sideline-evaluation-of-concussion. Accessed July 13, 2016.

13. Kontos AP, Elbin RJ, Schatz P, et al. A revised factor structure for the post-concussion symptom scale: baseline and postconcussion factors. Am J Sports Med. 2012;40(10):2375-2384.

14. Guskiewicz KM, Ross SE, Marshall SW. Postural stability and neuropsychological deficits after concussion in collegiate athletes. J Athl Train. 2001;36(3):263.

15. Mucha A, Collins MW, Elbin R, et al. A brief Vestibular/Ocular Motor Screening (VOMS) assessment to evaluate concussions preliminary findings. Am J Sports Med. 2014;42(10):2479-2486.

16. Bloom J. Vestibular and ocular motor assessments: Important pieces to the concussion puzzle. Athletic Training and Sports Health Care. 2013;5(6):246-248.

17. Covassin T, Elbin R, Harris W, Parker T, Kontos A. The role of age and sex in symptoms, neurocognitive performance, and postural stability in athletes after concussion. Am J Sports Med. 2012;40(6):1303-1312.

18. Kontos A, Sufrinko A, Elbin R, Puskar A, Collins M. Reliability and associated risk factors for performance on the Vestibular/Ocular Motor Screening (VOMS) tool in healthy collegiate athletes. Am J Sports Med. 2016;44(6):1400-1406.

19. Guskiewicz KM, McCrea M, Marshall SW, et al. Cumulative effects associated with recurrent concussion in collegiate football players: the NCAA Concussion Study. JAMA. 2003;290(19):2549-2555.

20. Lau B, Lovell MR, Collins MW, Pardini J. Neurocognitive and symptom predictors of recovery in high school athletes. Clin J Sport Med. 2009;19(3):216-221.

21. Lau BC, Kontos AP, Collins MW, Mucha A, Lovell MR. Which on-field signs/symptoms predict protracted recovery from sport-related concussion among high school football players? Am J Sports Med. 2011;39(11):2311-2318.

22. Mihalik JP, Register-Mihalik J, Kerr ZY, Marshall SW, McCrea MC, Guskiewicz KM. Recovery of posttraumatic migraine characteristics in patients after mild traumatic brain injury. Am J Sports Med. 2013;41(7):1490-1496.

23. Covassin T, Moran R, Elbin RJ. Sex differences in reported concussion injury rates and time loss from participation: An update of the National Collegiate Athletic Association injury surveillance program from 2004-2005 through 2008-2009. J Athl Train. 2016;51(3):189-194.

24. Root JM, Zuckerbraun NS, Wang L, et al. History of somatization is associated with prolonged recovery from concussion. J Pediatr. 2016;174:39-44.

25. Sufrinko A, Pearce K, Elbin RJ, et al. The effect of preinjury sleep difficulties on neurocognitive impairment and symptoms after sport-related concussion. Am J Sports Med. 2015;43(4):830-838.

26. Thomas DG, Apps JN, Hoffmann RG, McCrea M, Hammeke T. Benefits of strict rest after acute concussion: a randomized controlled trial. Pediatrics. 2015;135(2):213-223.

27. Choe M, Blume H. Pediatric posttraumatic Headache: a review. J Child Neurol. 2016;31(1):76-85.

28. Tepper SJ, Tepper DE. Breaking the cycle of medication overuse. Cleve Clin J Med. 2010;77(4):236-242.

29. Collins M, Kontos A, Reynolds E, Murawski C, Fu F. A comprehensive, targeted approach to the clinical care of athletes following sport-related concussion. Knee Surg Sports Traumatol Arthrosc. 2014;22(2):235-246.

30. Reynolds E, Collins MW, Mucha A, Troutman-Ensecki C. Establishing a clinical service for the management of sports-related concussions. Neurosurgery. 2014;75 Suppl 4:S71-S81.

31. Broglio SP, Collins MW, Williams RM, Mucha A, Kontos AP. Current and emerging rehabilitation for concussion: a review of the evidence. Clin Sports Med. 2015;34(2):213-231.

32. Master C, Scheiman M, Gallaway M, et al. Vision diagnoses are common after concussion in adolescents. Clin Pediatr (Phila). 2016;55(3):260-267.

33. Headache Classification Committee of the International Headache Society (IHS). The international classification of headache disorders, 3rd edition (beta version). Cephalalgia. 2013;33(9):629-808.

34. Kontos A, Deitrick JM, Reynolds E. Mental health implication and consequences following sport-related concussion. Brit J Sports Med. 2016;50(3):139-140.

35. Kontos AP, Covassin T, Elbin R, Parker T. Depression and neurocognitive performance after concussion among male and female high school and collegiate athletes. Arch Phys Med Rehabil. 2012;93(10):1751-1756.

Issue
The American Journal of Orthopedics - 45(6)
Issue
The American Journal of Orthopedics - 45(6)
Page Number
352-356
Page Number
352-356
Publications
Publications
Topics
Article Type
Display Headline
Concussions in American Football
Display Headline
Concussions in American Football
Sections
Disallow All Ads
Article PDF Media

Exertional Heat Stroke and American Football: What the Team Physician Needs to Know

Article Type
Changed
Thu, 09/19/2019 - 13:25
Display Headline
Exertional Heat Stroke and American Football: What the Team Physician Needs to Know

Football, one of the most popular sports in the United States, is additionally recognized as a leading contributor to sports injury secondary to the contact collision nature of the endeavor. There are an estimated 1.1 million high school football players with another 100,000 participants combined in the National Football League (NFL), college, junior college, Arena Football League, and semipro levels of play.1 USA Football estimates that an additional 3 million youth participate in community football leagues.1 The National Center for Catastrophic Sports Injury Research recently calculated a fatality rate of 0.14 per 100,000 participants in 2014 for the 4.2 million who play football at all levels—and 0.45 per 100,000 in high school.1 While direct deaths from head and spine injury remain a significant contributor to the number of catastrophic injuries, indirect deaths (systemic failure) predominate. Exertional heat stroke (EHS) has emerged as one of the leading indirect causes of death in high school and collegiate football. Boden and colleagues2 reported that high school and college football players sustain approximately 12 fatalities annually, with indirect systemic causes being twice as common as direct blunt trauma.2The most common indirect causes identified included cardiac failure, heat illness, and complications of sickle cell trait (SCT). It was also noted that the risk of SCT, heat-related, and cardiac deaths increased during the second decade of the study, indicating these conditions may require a greater emphasis on diagnosis, treatment, and prevention. This review details for the team physician the unique challenge of exercising in the heat to the football player, and the prevention, diagnosis, management and return-to-play issues pertinent to exertional heat illness (EHI).

The Challenge

EHS represents the most severe manifestation of EHI—a gamut of diseases commonly encountered during the hot summer months when American football season begins. The breadth of EHI includes several important clinical diagnoses: exercise-associated muscle cramps (heat cramps); heat exhaustion with and without syncope; heat injury with evidence of end organ injury (eg, rhabdomyolysis); and EHS. EHS is defined as “a form of hyperthermia associated with a systemic inflammatory response leading to a syndrome of multi-organ dysfunction in which encephalopathy predominates.”3 EHS, if left untreated, or even if clinical treatment is delayed, may result in significant end organ morbidity and/or mortality.

During exercise, the human thermoregulatory system mitigates heat gain by increasing skin blood flow and sweating, causing an increased dissipation of heat to the surrounding environment by leveraging conduction, convection, and evaporation.4,5 Elevated environmental temperatures, increased humidity, and dehydration can impede the body’s ability to dissipate heat at a rate needed to maintain thermoregulation. This imbalance can result in hyperthermia secondary to uncompensated heat stress,5 which in turn can lead to EHI. Football players have unique challenges that make them particularly vulnerable to EHI. The summer heat during early-season participation and the requirement for equipment that covers nearly 60% of body surfaces pose increased risk of volume losses and hyperthermia that trigger the onset of EHI.6 Football athletes’ body compositions and physical size are additional contributing risk factors; the relatively high muscle and fat content increase thermogenicity, which require their bodies to dissipate more heat.7

An estimated 9000 cases of EHI occur annually across all high school sports,8 with an incidence of 1.6:100,000 athlete-exposures.8,9 Studies have demonstrated, however, that EHI occurs in football 11.4 times more often than in all other high school sports combined.10 The incidence of nonfatal EHI in all levels of football is 4.42-5:100,000.8,9 Between 2000 and 2014, 41 football players died from EHS.1 In football, approximately 75% of all EHI events occurred during practices, while only 25% of incidents occurred during games.8

Given these potentially deadly consequences, it is important that football team physicians are not only alert to the early symptoms of heat illness and prepared to intervene to prevent the progression to EHS, but are critical leaders in educating coaches and players in evidence-based EHI prevention practices and policies.

Prevention

EHS is a preventable condition, arguably the most common cause of preventable nontraumatic exertional death in young athletes in the United States. Close attention to mitigating risk factors should begin prior to the onset of preseason practice and continue through the early season, where athletes are at the highest risk of developing heat illness.

Primary Prevention

Primary prevention is fundamental to minimizing the occurrences of EHI. It focuses on the following methods: recognition of inherent risk factors, acclimatization, hydration, and avoidance of inciting substances (including supplements).

 

 

Pre-Participation Examination. The purpose of the pre-participation examination (PPE) is to maximize an athlete’s safety by identifying medical conditions that place the athlete at risk.11,12 The Preparticipation Physical Evaluation, 4th edition, the most widely used consensus publication, specifically queries if an athlete has a previous history of heat injury. However, it only indirectly addresses intrinsic risk factors that may predispose an athlete to EHI who has never had an EHI before. Therefore, providers should take the opportunity of the PPE to inquire about additional risk factors that may make an athlete high risk for sustaining a heat injury. Common risk factors for EHI are listed in Table 1.

Table 1
While identifying at-risk athletes is important in mitigating the risk of developing EHI, it will not identify all possible cases: a study of military recruits found that up to 50% of Marines who developed EHI lacked an identifiable risk factor.13Wet Bulb Globe Temperature. Humidity can heighten a player’s risk of developing thermogenic dysregulation during hot temperatures. As ambient temperature nears internal body temperature, heat may actually be absorbed by the skin rather than dissipated into the air. As a result, the body must increasingly rely on sweat evaporation to encourage heat loss; this process is hindered in very humid climates. Wet bulb globe temperature (WBGT) is a measure of heat stress that accounts for temperature, humidity, wind speed, and cloud cover. WBGT should be utilized to determine the relative risk of EHI based on local environmental conditions, as there is a direct correlation between elevated WBGT and risk of EHI.11,14 The greatest risk for EHS is performing high-intensity exercise (>75% VO2max) when WBGT >28°C (82°F).7 A study of hyperthermia-related deaths in football found that a majority of fatalities occurred on days classified as high risk (23°C-28°C) or extreme risk (>28°C) by WBGT.14 Consensus guidelines recommend that activities be modified based on WBGT (Table 2).7,12
Table 2.
The impact of WBGT does not end solely on the day of practice. Athletes who exercise in elevated WBGT environments on 2 consecutive days are at increased risk of EHI due to cumulative effects of exercise in heat.11Clothing. In football, required protective equipment may cover up to 60% of body surfaces. Studies have shown that wearing full uniform with pads increases internal body temperature and decreases time to exhaustion when compared to light clothing.5,15 In addition, athletic equipment traps heat close to the body and inhibits evaporation of sweat into the environment, thereby inhibiting radiant and evaporative heat dissipation.5,11 Likewise, wearing dark clothing encourages radiant absorption of heat, further contributing to potential thermal dysregulation.5 Use of a helmet is a specific risk factor for EHI, as significant heat dissipation occurs through the head.11 To mitigate these risk factors, the introduction of padded equipment should occur incrementally over the heat acclimatization period (see below). In addition, athletes should be encouraged to remove their helmets during rest periods to promote added heat dissipation and recovery.

Heat Acclimatization. The risk of EHI escalates significantly when athletes are subjected to multiple stressors during periods of heat exposure, such as sudden increases in intensity or duration of exercise; prolonged new exposures to heat; dehydration; and sleep loss.5 When football season begins in late summer, athletes are least conditioned as temperatures reach their seasonal peak, causing increased risk of EHI.15 Planning for heat acclimatization is vital for all athletes who exercise in hot environments. Acclimatization procedures place progressively mounting physiologic strains on the body to improve athletes’ ability to dissipate heat, diminishing thermoregulatory and cardiovascular exertion.4,5 Acclimatization begins with expansion of plasma volume on days 3 to 6, causing improvements in cardiac efficiency and resulting in an overall decrease in basal internal body temperature.4,5,15 This process results in improvements in heat tolerance and exercise performance, evolving over 10 to 14 days of gradual escalation of exercise intensity and duration.5,10,11,16 However, poor fitness levels and extreme temperatures can prolong this period, requiring up to 2 to 3 months to fully take effect.5,7

The National Athletic Trainers Association (NATA) and National Collegiate Athletic Association (NCAA) have released consensus guidelines regarding heat acclimatization protocols for football athletes at the high school and college levels (Tables 3 and 4). Each of these guidelines involves an initial period without use of protective equipment, followed by a gradual addition of further equipment.11,16
Table 3.

Secondary Prevention

Despite physicians’ best efforts to prevent all cases of EHI, athletes will still experience the effects of exercise-induced hyperthermia. The goal of secondary prevention is to slow the progression of this hyperthermia so that it does not progress to more dangerous EHI.

Table 4.

 

 

Hydration. Dehydration is an important risk factor for EHI. Sweat maintains thermoregulation by dissipating heat generated during exercise; however, it also contributes to body water losses. Furthermore, intravascular depletion decreases stroke volume, thereby increasing cardiovascular strain. It is estimated that for every 1% loss in body mass from dehydration, body temperature rises 0.22°C in comparison to a euhydrated state.6 Dehydration occurs more rapidly in hot environments, as fluid is lost through increased sweat production.7 After approximately 6% to 10% body weight volume loss, cardiac output cannot be maintained, diminishing sweat production and blood flow to both skin and muscle and causing diminished performance and a significant risk of heat exhaustion.7 If left unchecked, these physiologic changes result in further elevations in body temperature and increased cardiovascular strain, ultimately placing the athlete at significant risk for development of EHS.

Adequate hydration to maintain euvolemia is an important step in avoiding possible EHI. Multiple studies have shown that football players experience a baseline hypovolemia during their competition season,6 a deficit that is most marked during the first week of practices.17 This deficit is multifactorial, as football players expend a significant amount of fluid through sweat, are not able to adequately replace these losses during practice, and do not appropriately hydrate off the field.6,18 Some players, especially linemen, sweat at a higher rate than their teammates, posing a possible risk of significant dehydration.6 Coaches and players alike should be educated on the importance of adequate hydration to meet their fluid needs.

The goal of hydration during exercise is to prevent large fluid losses that can adversely affect performance and increase risk of EHI;6 it may be unrealistic to replace all fluid losses during the practice period. Instead, athletes should target complete volume replacement over the post-exercise period.6 Some recommend hydrating based upon thirst drive; however, thirst is activated following a volume loss of approximately 2% body mass, the same degree of losses that place athletes at an increased risk for performance impairment and EHI.4,6,11,12 Individuals should have access to fluids throughout practice and competition and be encouraged to hydrate as needed.6,12,15 Furthermore, staff should modify their practices based upon WBGT and acclimatization status to provide more frequent hydration breaks.

Hyperhydration and Salt Intake. Of note, there are inherent risks to hyperhydration. Athletes with low sweat rates have an increased risk of overhydration and the development of exercise-associated hyponatremia (EAH),6 a condition whose presentation is very similar to EHS. In addition, inadequate sodium intake and excessive sweating can also contribute to the development of EAH. EAH has been implicated in the deaths of 2 football players in 2014.1,6 Establishing team hydration guidelines and educating players and staff on appropriate hydration and dietary salt intake is essential to reduce the risk of both dehydration and hyperhydration and their complications.6Intra-Event Cooling. During exercise, team physicians can employ strategies for cooling athletes during exertion to mitigate their risk of EHI by decreasing thermal and cardiovascular strain.4,19 Cooling during exercise is hypothesized to allow for accelerated heat dissipation, where heat is lost from the body more effectively. This accelerated loss enables athletes to maintain a higher heat storage capacity over the duration of exercise, avoiding uncompensated heat stresses that ultimately cause EHI.19

Some intra-event cooling strategies include the use of cooling garments, cooling packs, and cold water/slurry ingestion. Cooling garments lower skin temperature, which in turn can decrease thermoregulatory strains;4 a recent meta-analysis of intra-event cooling modalities revealed that wearing an ice vest during exercise resulted in the greatest decrease in thermal heat strain.19 Internal cooling strategies—namely ingestion of cold fluids/ice slurry—have shown some mild benefit in decreasing internal temperatures; however, some studies have demonstrated some decrease in sweat production associated with cold oral intake used in isolation.19 Overall, studies have shown that combining external (cooling clothing, ice packs, fanning) and internal (cold water, ice slurry) cooling methods result in a greater cooling effect than a use of a single method.4

Tertiary Prevention

The goal of tertiary prevention is to mitigate the risk of long-term adverse outcomes following an EHS event. The most effective means of reducing risk for morbidity and mortality is rapid identification and treatment of EHS as well as close evaluation of an athlete’s return to activity in heat. This process is spearheaded by an effective and well-rehearsed emergency action plan.

Diagnosis and Management

Rapid identification and treatment of EHS is crucial to minimizing the risk of poor outcomes.7 Any delay in the treatment of EHS can dramatically increase the likelihood of associated morbidity and mortality.20

 

 

EHS is diagnosed by an elevated rectal temperature ≥40°C (104°F) and associated central nervous system (CNS) dysfunction.21 EHS should be strongly suspected in any athlete exercising in heat who exhibits signs of CNS dysfunction, including disorientation, confusion, dizziness, erratic behavior, irritability, headache, loss of coordination, delirium, collapse, or seizures.7,12,15 EHS may also present with symptoms of heat exhaustion, including fatigue, hyperventilation, tachycardia, vomiting, diarrhea, and hypotension.7,12,15

Rectal temperature should be taken for any athlete with suspected EHS, as other modalities—oral, skin, axillary, and aural—can be inaccurate and easily modified by ambient confounders such as ambient and skin temperature, athlete hyperventilation, and consumption of liquids.7,11,12 Athletes exhibiting CNS symptoms with moderately elevated rectal temperatures that do not exceed 40°C should also be assumed to be suffering from EHS and treated with rapid cooling.11 On the other hand, athletes with CNS symptoms who are normothermic should be assumed to have EAH until ruled out by electrolyte assessment; IV fluids should be at no more than keep vein open (KVO) pending this determination.11 In some cases, an athlete may initially present with altered mental status but return to “normal.” However, this improvement may represent a “lucid period”; evaluation should continue with rectal temperature and treatment, as EHS in these cases may progress quickly.15

Treatment is centered on rapid, whole body cooling initiated at the first sign of heat illness.7,22 The goal of treatment is to achieve a rectal temperature <38.9°C within 30 minutes of the onset of EHS.15 Upon diagnosis, the athlete should be quickly placed in a tub of ice water to facilitate cold water immersion (CWI) therapy. Some guidelines suggest the athlete’s clothing be removed to potentiate evaporative cooling during CWI;12 however, cooling should not be delayed due to difficulties in removing equipment. CWI, where a heat stroke victim is submerged in ice water up to their neck while water is continuously circulated, is generally considered to be the gold standard treatment as it is the modality with the highest recorded cooling rates and the lowest rate of morbidity and mortality.7,20,21 Multiple studies of CWI have shown that survival nears 100% when aggressive cooling starts within 5 minutes of collapse or identification of EHS.20,21,22

If whole body CWI is unavailable, alternative methods of rapid cooling should be employed. Partial CWI, with torso immersion being preferable to the extremities, has been shown to achieve an acceptable rate of cooling to achieve sufficient drops in internal body temperature.20,23 However, one popular treatment—applying ice packs to the whole body, in particular to the groin and axillae—has not been shown to be sufficient to achieve standard cooling goals.20 None of these methods have been shown to be as effective as CWI.23

Intravenous access should be initiated with fluid resuscitation dictated by the provider’s assessment. Normal saline is recommended as the resuscitative fluid of choice, with the rate dictated by clinical judgment and adjusted as guided by electrolyte determination and clinical response. It cannot be overstated that in normothermic patients with confusion, EAH is the diagnosis of exclusion and aggressive fluid resuscitation should be withheld until electrolyte determination.

Once rectal temperature descends appropriately (~38.9°C), the cooling process should stop and the individual should be transported to a hospital for further observation20 and evaluation of possible sequelae, including rhabdomyolysis and renal injury, cardiac dysfunction and arrhythmia, severe electrolyte abnormalities, acute respiratory distress syndrome, lactic acidosis, and other forms of end-organ failure (Figure).

Figure.


Rapid cooling is more crucial than transport; transport poses a risk of delayed cooling, which can dramatically increase an individual’s risk of morbidity and mortality.20,23 In situations where a patient can be cooled on-site, physicians should pursue cooling before transporting the patient to a medical treatment facility.

Emergency Action Plan

Team physicians should be proactive in developing an emergency action plan to address possible EHS events. These plans should be site-specific, addressing procedures for all practice and home competition locations.12 All competition venues should have a CWI tub on-site in events where there is an increased risk of EHS.12,15,20 This tub should be set up and functional for all high-risk activities, including practices.12

Following recognition of a potential case of EHS, treatment teams should have procedures in place to transport athletes to the treatment area, obtain rectal temperature, initiate rapid cooling, and stabilize the athlete for transport to an emergency department (ED) for further evaluation.12,15 A written record of treatments and medications provided during athlete stabilization should be maintained and transported with the athlete to the ED.15 A list of helpful equipment and supplies for treatment of EHS can be found in Table 5.

Table 5.


EHS is a unique life-threatening situation where it is best to treat the patient on the sideline before transport.15 Those athletes transported before cooling risk spending an increased amount of time above critical temperatures for cell damage, which has been associated with increased morbidity and mortality. This mantra of “cool first, transport second” cannot be overemphasized, as those individuals with EHS who present to the ED with a persisting rectal temperature >41°F may risk up to an 80% mortality rate.24 Conversely, a recent large, retrospective study of 274 EHS events sustained during the Falmouth Road Race found a 100% survival rate when athletes were rapidly identified via rectal thermometry and treated with aggressive, rapid cooling through CWI.21

 

 

Return to Play

Perhaps the most challenging and important role the team physician has is determining an athlete’s return to play following EHI, as there currently are no evidence-based guidelines for return to activity for these athletes.7 The decisions surrounding return to play are highly individualized, as recovery from EHS and heat injury is associated with the duration of internal body temperature elevation above the critical level (40°C).7,20 Guidelines for return to activity following recovery from EHI differ among experts and institutions.7,25 The general consensus from these guidelines is that, at minimum, athletes should not participate in any physical activity until they are asymptomatic and all blood tests have normalized.11 Following this asymptomatic period, most guidelines advocate for a slow, deliberate return to activity.11 The American College of Sports Medicine (ACSM) offers one reasonable approach to the returning athlete following EHS:7

  • No exercise for at least 7 days following release from medical care.
  • Follow-up with a physician 1 week after release from medical care for physical examination and any warranted lab or radiologic studies (based upon organ systems affected during EHS).
  • Once cleared to return to activity, the athlete begins exercise in a cool environment, gradually increasing the duration, intensity, and heat exposure over 2 weeks to demonstrate heat tolerance and acclimatization.
  • Athletes who cannot resume vigorous activity due to recurrent symptoms (eg, excessive fatigue) should be reevaluated after 4 weeks. Laboratory exercise-heat tolerance testing may be useful in this setting.
  • The athlete may resume full competition once they are able to participate in full training in the heat for 2 to 4 weeks without adverse effects.

Heat tolerance testing (HTT) in these athletes remains controversial.5 26 The ACSM recommends that HTT be considered only for those unable to return to vigorous activity after a suitable period (approximately 4 weeks). In contrast, the Israeli Defense Force (IDF) uses HTT to evaluate soldiers following EHS to guide decision-making about return to duty.27 The IDF HTT assumes that individuals will respond differently to heat stresses. They identify individuals who are “heat intolerant” as being unable to tolerate specific heat challenges, indicated by increases in body temperature occurring more rapidly than normal responders under identical environmental and exercise conditions. However, despite being used for more than 30 years, there is no clear evidence that HTT adequately predicts who will experience subsequent episodes of EHS.

Conclusion

While the recognized cornerstone of being a team physician is the provision of medical care, the ACSM Team Physician Consensus Statement28 further delineates the medical and administrative responsibilities as both (1) understanding medical management and prevention of injury and illness in athletes; and (2) awareness of or involvement in the development and rehearsal of an emergency action plan. These tenets are critical for the team physician who accepts the responsibility to cover sports at the high school level or higher. Football team physicians play an essential role in mitigating risk of EHI in their athletes. Through development and execution of both comprehensive prevention strategies and emergency action plans, physicians can work to minimize athletes’ risk of both developing and experiencing significant adverse outcomes from an EHI.

 

Am J Orthop. 2016;45(6):340-348. Copyright Frontline Medical Communications Inc. 2016. All rights reserved.

References

1. Kucera KL, Klossner D, Colgate B, Cantu RC. Annual Survey of Football Injury Research: 1931-2014. National Center for Catastrophic Sport Injury Research Web site. https://nccsir.unc.edu/files/2013/10/Annual-Football-2014-Fatalities-Final.pdf. Accessed May 31, 2016.

2. Boden BP, Breit I, Beachler JA, Williams A, Mueller FO. Fatalities in high school and college football players. Am J Sports Med. 2013;41(5):1108-1116.

3. Bouchama A, Knochel JP. Heat stroke. N Engl J Med. 2002;346(25):1978-1988.

4. Racinais S, Alonso JM, Coutts AJ, et al. Consensus recommendations on training and competing in the heat. Scand J Med Sci Sports. 2015;25 Suppl 1:6-19.

5. Pryor RR, Casa DJ, Adams WM, et al. Maximizing athletic performance in the heat. Strength Cond J. 2013;35(6):24-33.

6. Adams WM, Casa DJ. Hydration for football athletes. Sports Sci Exchange. 2015;28(141):1-5.

7. American College of Sports Medicine, Armstrong LE, Casa DJ, et al. American College of Sports Medicine position stand. Exertional heat illness during training and competition. Med Sci Sports Exerc. 2007;39(3):556-572.

8. Yard EE, Gilchrist J, Haileyesus T, et al. Heat illness among high school athletes--United States, 2005-2009. J Safety Res. 2010;41(6):471-474.

9. Huffman EA, Yard EE, Fields SK, Collins CL, Comstock RD. Epidemiology of rare injuries and conditions among United States high school athletes during the 2005-2006 and 2006-2007 school years. J Athl Train. 2008;43(6):624-630.

10. Kerr ZY, Casa DJ, Marshall SW, Comstock RD. Epidemiology of exertional heat illness among U.S. high school athletes. Am J Prev Med. 2013;44(1):8-14.

11. Casa DJ, DeMartini JK, Bergeron MF, et al. National Athletic Trainers’ Association position statement: exertional heat illnesses. J Athl Train. 2015;50(9):986-1000.

12. Casa DJ, Almquist J, Anderson SA. The inter-association task force for preventing sudden death in secondary school athletics programs: best-practices recommendations. J Athl Train. 2013;48(4):546-553.

13. Gardner JW, Kark JA, Karnei K, et al. Risk factors predicting exertional heat illness in male Marine Corps recruits. Med Sci Sports Exerc. 1996;28(8):939-944.

14. Gundstein AJ, Ramseyer C, Zhao F, et al. A retrospective analysis of American football hyperthermia deaths in the United States. Int J Biometerol. 2012;56(1):11-20.

15. Armstrong LE, Johnson EC, Casa DJ, et al. The American football uniform: uncompensable heat stress and hyperthermic exhaustion. J Athl Train. 2010;45(2):117-127.

16. Casa DJ, Csillan D; Inter-Association Task Force for Preseason Secondary School Athletics Participants, et al. Preseason heat-acclimatization guidelines for secondary school athletics. J Athl Train. 2009;44(3):332-333.

17. Godek SF, Godek JJ, Bartolozzi AR. Hydration status in college football players during consecutive days of twice-a-day preseason practices. Am J Sports Med. 2005;33(6):843-851.

18. Stover EA, Zachwieja J, Stofan J, Murray R, Horswill CA. Consistently high urine specific gravity in adolescent American football players and the impact of an acute drinking strategy. Int J Sports Med. 2006;27(4):330-335.

19. Bongers CC, Thijssen DH, Veltmeijer MTW, Hopman MT, Eijsvogels TM. Precooling and percooling (cooling during exercise) both improve performance in the heat: a meta-analytical review. Br J Sports Med. 2015;49(6):377-384.

20. Casa DJ, McDermott BP, Lee EC, Yeargin SW, Armstrong LE, Maresh CM. Cold water immersion: the gold standard for exertional heatstroke treatment. Exerc Sport Sci Rev. 2007;35(3):141-149.

21. DeMartini JK, Casa DJ, Stearns R, et al. Effectiveness of cold water immersion in the treatment of exertional heat stroke at the Falmouth Road Race. Med Sci Sports Exerc. 2015;47(2):240-245.

22. Casa DJ, Kenny GP, Taylor NA. Immersion treatment for exertional hyperthermia: cold or temperate water? Med Sci Sports Exerc. 2010;42(7):1246-1252.

23. Casa DJ, Armstrong LE, Kenny GP, O’Connor FG, Huggins RA. Exertional heat stroke: new concepts regarding cause and care. Curr Sports Med Rep. 2012;11(3):115-122.

24. Argaud L, Ferry T, Le QH, et al. Short- and long-term outcomes of heat stroke following the 2003 heat wave in Lyon, France. Arch Intern Med. 2007;167(20):2177-2183.

25. O’Connor FG, Casa DJ, Bergeron MF, et al. American College of Sports Medicine Roundtable on exertional heat stroke--return to duty/return to play: conference proceedings. Curr Sports Med Rep. 2010;9(5):314-321.

26. Kazman JB, Heled Y, Lisman PJ, Druyan A, Deuster PA, O’Connor FG. Exertional heat illness: the role of heat tolerance testing. Curr Sports Med Rep. 2013;12(2):101-105.

27. Moran DS, Heled Y, Still L, Laor A, Shapiro Y. Assessment of heat tolerance for post exertional heat stroke individuals. Med Sci Monit. 2004;10(6):CR252-CR257.

28. Herring SA, Kibler WB, Putukian M. Team Physician Consensus Statement: 2013 update. Med Sci Sports Exerc. 2013;45(8):1618-1622.

29. Heat stroke treatment. Korey Stringer Institute University of Connecticut Web site. http://ksi.uconn.edu/emergency-conditions/heat-illnesses/exertional-heat-stroke/heat-stroke-treatment/. Accessed June 14, 2016.

30. Headquarters, Department of the Army and the Air Force. Heat Stress Control and Heat Casualty Management. Technical Bulletin Medical 507. http://www.dir.ca.gov/oshsb/documents/Heat_illness_prevention_tbmed507.pdf. Published March 7, 2003. Accessed June 14, 2016.

Article PDF
Author and Disclosure Information

Authors’ Disclosure Statement: The authors report no actual or potential conflict of interest in relation to this article. The opinions and assertions contained herein are private views and are not to be construed as official or as reflecting the views of the US Army Medical Department, US Air Force Medical Corps, Uniformed Services University, or the Department of Defense at large.

Issue
The American Journal of Orthopedics - 45(6)
Publications
Topics
Page Number
340-348
Sections
Author and Disclosure Information

Authors’ Disclosure Statement: The authors report no actual or potential conflict of interest in relation to this article. The opinions and assertions contained herein are private views and are not to be construed as official or as reflecting the views of the US Army Medical Department, US Air Force Medical Corps, Uniformed Services University, or the Department of Defense at large.

Author and Disclosure Information

Authors’ Disclosure Statement: The authors report no actual or potential conflict of interest in relation to this article. The opinions and assertions contained herein are private views and are not to be construed as official or as reflecting the views of the US Army Medical Department, US Air Force Medical Corps, Uniformed Services University, or the Department of Defense at large.

Article PDF
Article PDF

Football, one of the most popular sports in the United States, is additionally recognized as a leading contributor to sports injury secondary to the contact collision nature of the endeavor. There are an estimated 1.1 million high school football players with another 100,000 participants combined in the National Football League (NFL), college, junior college, Arena Football League, and semipro levels of play.1 USA Football estimates that an additional 3 million youth participate in community football leagues.1 The National Center for Catastrophic Sports Injury Research recently calculated a fatality rate of 0.14 per 100,000 participants in 2014 for the 4.2 million who play football at all levels—and 0.45 per 100,000 in high school.1 While direct deaths from head and spine injury remain a significant contributor to the number of catastrophic injuries, indirect deaths (systemic failure) predominate. Exertional heat stroke (EHS) has emerged as one of the leading indirect causes of death in high school and collegiate football. Boden and colleagues2 reported that high school and college football players sustain approximately 12 fatalities annually, with indirect systemic causes being twice as common as direct blunt trauma.2The most common indirect causes identified included cardiac failure, heat illness, and complications of sickle cell trait (SCT). It was also noted that the risk of SCT, heat-related, and cardiac deaths increased during the second decade of the study, indicating these conditions may require a greater emphasis on diagnosis, treatment, and prevention. This review details for the team physician the unique challenge of exercising in the heat to the football player, and the prevention, diagnosis, management and return-to-play issues pertinent to exertional heat illness (EHI).

The Challenge

EHS represents the most severe manifestation of EHI—a gamut of diseases commonly encountered during the hot summer months when American football season begins. The breadth of EHI includes several important clinical diagnoses: exercise-associated muscle cramps (heat cramps); heat exhaustion with and without syncope; heat injury with evidence of end organ injury (eg, rhabdomyolysis); and EHS. EHS is defined as “a form of hyperthermia associated with a systemic inflammatory response leading to a syndrome of multi-organ dysfunction in which encephalopathy predominates.”3 EHS, if left untreated, or even if clinical treatment is delayed, may result in significant end organ morbidity and/or mortality.

During exercise, the human thermoregulatory system mitigates heat gain by increasing skin blood flow and sweating, causing an increased dissipation of heat to the surrounding environment by leveraging conduction, convection, and evaporation.4,5 Elevated environmental temperatures, increased humidity, and dehydration can impede the body’s ability to dissipate heat at a rate needed to maintain thermoregulation. This imbalance can result in hyperthermia secondary to uncompensated heat stress,5 which in turn can lead to EHI. Football players have unique challenges that make them particularly vulnerable to EHI. The summer heat during early-season participation and the requirement for equipment that covers nearly 60% of body surfaces pose increased risk of volume losses and hyperthermia that trigger the onset of EHI.6 Football athletes’ body compositions and physical size are additional contributing risk factors; the relatively high muscle and fat content increase thermogenicity, which require their bodies to dissipate more heat.7

An estimated 9000 cases of EHI occur annually across all high school sports,8 with an incidence of 1.6:100,000 athlete-exposures.8,9 Studies have demonstrated, however, that EHI occurs in football 11.4 times more often than in all other high school sports combined.10 The incidence of nonfatal EHI in all levels of football is 4.42-5:100,000.8,9 Between 2000 and 2014, 41 football players died from EHS.1 In football, approximately 75% of all EHI events occurred during practices, while only 25% of incidents occurred during games.8

Given these potentially deadly consequences, it is important that football team physicians are not only alert to the early symptoms of heat illness and prepared to intervene to prevent the progression to EHS, but are critical leaders in educating coaches and players in evidence-based EHI prevention practices and policies.

Prevention

EHS is a preventable condition, arguably the most common cause of preventable nontraumatic exertional death in young athletes in the United States. Close attention to mitigating risk factors should begin prior to the onset of preseason practice and continue through the early season, where athletes are at the highest risk of developing heat illness.

Primary Prevention

Primary prevention is fundamental to minimizing the occurrences of EHI. It focuses on the following methods: recognition of inherent risk factors, acclimatization, hydration, and avoidance of inciting substances (including supplements).

 

 

Pre-Participation Examination. The purpose of the pre-participation examination (PPE) is to maximize an athlete’s safety by identifying medical conditions that place the athlete at risk.11,12 The Preparticipation Physical Evaluation, 4th edition, the most widely used consensus publication, specifically queries if an athlete has a previous history of heat injury. However, it only indirectly addresses intrinsic risk factors that may predispose an athlete to EHI who has never had an EHI before. Therefore, providers should take the opportunity of the PPE to inquire about additional risk factors that may make an athlete high risk for sustaining a heat injury. Common risk factors for EHI are listed in Table 1.

Table 1
While identifying at-risk athletes is important in mitigating the risk of developing EHI, it will not identify all possible cases: a study of military recruits found that up to 50% of Marines who developed EHI lacked an identifiable risk factor.13Wet Bulb Globe Temperature. Humidity can heighten a player’s risk of developing thermogenic dysregulation during hot temperatures. As ambient temperature nears internal body temperature, heat may actually be absorbed by the skin rather than dissipated into the air. As a result, the body must increasingly rely on sweat evaporation to encourage heat loss; this process is hindered in very humid climates. Wet bulb globe temperature (WBGT) is a measure of heat stress that accounts for temperature, humidity, wind speed, and cloud cover. WBGT should be utilized to determine the relative risk of EHI based on local environmental conditions, as there is a direct correlation between elevated WBGT and risk of EHI.11,14 The greatest risk for EHS is performing high-intensity exercise (>75% VO2max) when WBGT >28°C (82°F).7 A study of hyperthermia-related deaths in football found that a majority of fatalities occurred on days classified as high risk (23°C-28°C) or extreme risk (>28°C) by WBGT.14 Consensus guidelines recommend that activities be modified based on WBGT (Table 2).7,12
Table 2.
The impact of WBGT does not end solely on the day of practice. Athletes who exercise in elevated WBGT environments on 2 consecutive days are at increased risk of EHI due to cumulative effects of exercise in heat.11Clothing. In football, required protective equipment may cover up to 60% of body surfaces. Studies have shown that wearing full uniform with pads increases internal body temperature and decreases time to exhaustion when compared to light clothing.5,15 In addition, athletic equipment traps heat close to the body and inhibits evaporation of sweat into the environment, thereby inhibiting radiant and evaporative heat dissipation.5,11 Likewise, wearing dark clothing encourages radiant absorption of heat, further contributing to potential thermal dysregulation.5 Use of a helmet is a specific risk factor for EHI, as significant heat dissipation occurs through the head.11 To mitigate these risk factors, the introduction of padded equipment should occur incrementally over the heat acclimatization period (see below). In addition, athletes should be encouraged to remove their helmets during rest periods to promote added heat dissipation and recovery.

Heat Acclimatization. The risk of EHI escalates significantly when athletes are subjected to multiple stressors during periods of heat exposure, such as sudden increases in intensity or duration of exercise; prolonged new exposures to heat; dehydration; and sleep loss.5 When football season begins in late summer, athletes are least conditioned as temperatures reach their seasonal peak, causing increased risk of EHI.15 Planning for heat acclimatization is vital for all athletes who exercise in hot environments. Acclimatization procedures place progressively mounting physiologic strains on the body to improve athletes’ ability to dissipate heat, diminishing thermoregulatory and cardiovascular exertion.4,5 Acclimatization begins with expansion of plasma volume on days 3 to 6, causing improvements in cardiac efficiency and resulting in an overall decrease in basal internal body temperature.4,5,15 This process results in improvements in heat tolerance and exercise performance, evolving over 10 to 14 days of gradual escalation of exercise intensity and duration.5,10,11,16 However, poor fitness levels and extreme temperatures can prolong this period, requiring up to 2 to 3 months to fully take effect.5,7

The National Athletic Trainers Association (NATA) and National Collegiate Athletic Association (NCAA) have released consensus guidelines regarding heat acclimatization protocols for football athletes at the high school and college levels (Tables 3 and 4). Each of these guidelines involves an initial period without use of protective equipment, followed by a gradual addition of further equipment.11,16
Table 3.

Secondary Prevention

Despite physicians’ best efforts to prevent all cases of EHI, athletes will still experience the effects of exercise-induced hyperthermia. The goal of secondary prevention is to slow the progression of this hyperthermia so that it does not progress to more dangerous EHI.

Table 4.

 

 

Hydration. Dehydration is an important risk factor for EHI. Sweat maintains thermoregulation by dissipating heat generated during exercise; however, it also contributes to body water losses. Furthermore, intravascular depletion decreases stroke volume, thereby increasing cardiovascular strain. It is estimated that for every 1% loss in body mass from dehydration, body temperature rises 0.22°C in comparison to a euhydrated state.6 Dehydration occurs more rapidly in hot environments, as fluid is lost through increased sweat production.7 After approximately 6% to 10% body weight volume loss, cardiac output cannot be maintained, diminishing sweat production and blood flow to both skin and muscle and causing diminished performance and a significant risk of heat exhaustion.7 If left unchecked, these physiologic changes result in further elevations in body temperature and increased cardiovascular strain, ultimately placing the athlete at significant risk for development of EHS.

Adequate hydration to maintain euvolemia is an important step in avoiding possible EHI. Multiple studies have shown that football players experience a baseline hypovolemia during their competition season,6 a deficit that is most marked during the first week of practices.17 This deficit is multifactorial, as football players expend a significant amount of fluid through sweat, are not able to adequately replace these losses during practice, and do not appropriately hydrate off the field.6,18 Some players, especially linemen, sweat at a higher rate than their teammates, posing a possible risk of significant dehydration.6 Coaches and players alike should be educated on the importance of adequate hydration to meet their fluid needs.

The goal of hydration during exercise is to prevent large fluid losses that can adversely affect performance and increase risk of EHI;6 it may be unrealistic to replace all fluid losses during the practice period. Instead, athletes should target complete volume replacement over the post-exercise period.6 Some recommend hydrating based upon thirst drive; however, thirst is activated following a volume loss of approximately 2% body mass, the same degree of losses that place athletes at an increased risk for performance impairment and EHI.4,6,11,12 Individuals should have access to fluids throughout practice and competition and be encouraged to hydrate as needed.6,12,15 Furthermore, staff should modify their practices based upon WBGT and acclimatization status to provide more frequent hydration breaks.

Hyperhydration and Salt Intake. Of note, there are inherent risks to hyperhydration. Athletes with low sweat rates have an increased risk of overhydration and the development of exercise-associated hyponatremia (EAH),6 a condition whose presentation is very similar to EHS. In addition, inadequate sodium intake and excessive sweating can also contribute to the development of EAH. EAH has been implicated in the deaths of 2 football players in 2014.1,6 Establishing team hydration guidelines and educating players and staff on appropriate hydration and dietary salt intake is essential to reduce the risk of both dehydration and hyperhydration and their complications.6Intra-Event Cooling. During exercise, team physicians can employ strategies for cooling athletes during exertion to mitigate their risk of EHI by decreasing thermal and cardiovascular strain.4,19 Cooling during exercise is hypothesized to allow for accelerated heat dissipation, where heat is lost from the body more effectively. This accelerated loss enables athletes to maintain a higher heat storage capacity over the duration of exercise, avoiding uncompensated heat stresses that ultimately cause EHI.19

Some intra-event cooling strategies include the use of cooling garments, cooling packs, and cold water/slurry ingestion. Cooling garments lower skin temperature, which in turn can decrease thermoregulatory strains;4 a recent meta-analysis of intra-event cooling modalities revealed that wearing an ice vest during exercise resulted in the greatest decrease in thermal heat strain.19 Internal cooling strategies—namely ingestion of cold fluids/ice slurry—have shown some mild benefit in decreasing internal temperatures; however, some studies have demonstrated some decrease in sweat production associated with cold oral intake used in isolation.19 Overall, studies have shown that combining external (cooling clothing, ice packs, fanning) and internal (cold water, ice slurry) cooling methods result in a greater cooling effect than a use of a single method.4

Tertiary Prevention

The goal of tertiary prevention is to mitigate the risk of long-term adverse outcomes following an EHS event. The most effective means of reducing risk for morbidity and mortality is rapid identification and treatment of EHS as well as close evaluation of an athlete’s return to activity in heat. This process is spearheaded by an effective and well-rehearsed emergency action plan.

Diagnosis and Management

Rapid identification and treatment of EHS is crucial to minimizing the risk of poor outcomes.7 Any delay in the treatment of EHS can dramatically increase the likelihood of associated morbidity and mortality.20

 

 

EHS is diagnosed by an elevated rectal temperature ≥40°C (104°F) and associated central nervous system (CNS) dysfunction.21 EHS should be strongly suspected in any athlete exercising in heat who exhibits signs of CNS dysfunction, including disorientation, confusion, dizziness, erratic behavior, irritability, headache, loss of coordination, delirium, collapse, or seizures.7,12,15 EHS may also present with symptoms of heat exhaustion, including fatigue, hyperventilation, tachycardia, vomiting, diarrhea, and hypotension.7,12,15

Rectal temperature should be taken for any athlete with suspected EHS, as other modalities—oral, skin, axillary, and aural—can be inaccurate and easily modified by ambient confounders such as ambient and skin temperature, athlete hyperventilation, and consumption of liquids.7,11,12 Athletes exhibiting CNS symptoms with moderately elevated rectal temperatures that do not exceed 40°C should also be assumed to be suffering from EHS and treated with rapid cooling.11 On the other hand, athletes with CNS symptoms who are normothermic should be assumed to have EAH until ruled out by electrolyte assessment; IV fluids should be at no more than keep vein open (KVO) pending this determination.11 In some cases, an athlete may initially present with altered mental status but return to “normal.” However, this improvement may represent a “lucid period”; evaluation should continue with rectal temperature and treatment, as EHS in these cases may progress quickly.15

Treatment is centered on rapid, whole body cooling initiated at the first sign of heat illness.7,22 The goal of treatment is to achieve a rectal temperature <38.9°C within 30 minutes of the onset of EHS.15 Upon diagnosis, the athlete should be quickly placed in a tub of ice water to facilitate cold water immersion (CWI) therapy. Some guidelines suggest the athlete’s clothing be removed to potentiate evaporative cooling during CWI;12 however, cooling should not be delayed due to difficulties in removing equipment. CWI, where a heat stroke victim is submerged in ice water up to their neck while water is continuously circulated, is generally considered to be the gold standard treatment as it is the modality with the highest recorded cooling rates and the lowest rate of morbidity and mortality.7,20,21 Multiple studies of CWI have shown that survival nears 100% when aggressive cooling starts within 5 minutes of collapse or identification of EHS.20,21,22

If whole body CWI is unavailable, alternative methods of rapid cooling should be employed. Partial CWI, with torso immersion being preferable to the extremities, has been shown to achieve an acceptable rate of cooling to achieve sufficient drops in internal body temperature.20,23 However, one popular treatment—applying ice packs to the whole body, in particular to the groin and axillae—has not been shown to be sufficient to achieve standard cooling goals.20 None of these methods have been shown to be as effective as CWI.23

Intravenous access should be initiated with fluid resuscitation dictated by the provider’s assessment. Normal saline is recommended as the resuscitative fluid of choice, with the rate dictated by clinical judgment and adjusted as guided by electrolyte determination and clinical response. It cannot be overstated that in normothermic patients with confusion, EAH is the diagnosis of exclusion and aggressive fluid resuscitation should be withheld until electrolyte determination.

Once rectal temperature descends appropriately (~38.9°C), the cooling process should stop and the individual should be transported to a hospital for further observation20 and evaluation of possible sequelae, including rhabdomyolysis and renal injury, cardiac dysfunction and arrhythmia, severe electrolyte abnormalities, acute respiratory distress syndrome, lactic acidosis, and other forms of end-organ failure (Figure).

Figure.


Rapid cooling is more crucial than transport; transport poses a risk of delayed cooling, which can dramatically increase an individual’s risk of morbidity and mortality.20,23 In situations where a patient can be cooled on-site, physicians should pursue cooling before transporting the patient to a medical treatment facility.

Emergency Action Plan

Team physicians should be proactive in developing an emergency action plan to address possible EHS events. These plans should be site-specific, addressing procedures for all practice and home competition locations.12 All competition venues should have a CWI tub on-site in events where there is an increased risk of EHS.12,15,20 This tub should be set up and functional for all high-risk activities, including practices.12

Following recognition of a potential case of EHS, treatment teams should have procedures in place to transport athletes to the treatment area, obtain rectal temperature, initiate rapid cooling, and stabilize the athlete for transport to an emergency department (ED) for further evaluation.12,15 A written record of treatments and medications provided during athlete stabilization should be maintained and transported with the athlete to the ED.15 A list of helpful equipment and supplies for treatment of EHS can be found in Table 5.

Table 5.


EHS is a unique life-threatening situation where it is best to treat the patient on the sideline before transport.15 Those athletes transported before cooling risk spending an increased amount of time above critical temperatures for cell damage, which has been associated with increased morbidity and mortality. This mantra of “cool first, transport second” cannot be overemphasized, as those individuals with EHS who present to the ED with a persisting rectal temperature >41°F may risk up to an 80% mortality rate.24 Conversely, a recent large, retrospective study of 274 EHS events sustained during the Falmouth Road Race found a 100% survival rate when athletes were rapidly identified via rectal thermometry and treated with aggressive, rapid cooling through CWI.21

 

 

Return to Play

Perhaps the most challenging and important role the team physician has is determining an athlete’s return to play following EHI, as there currently are no evidence-based guidelines for return to activity for these athletes.7 The decisions surrounding return to play are highly individualized, as recovery from EHS and heat injury is associated with the duration of internal body temperature elevation above the critical level (40°C).7,20 Guidelines for return to activity following recovery from EHI differ among experts and institutions.7,25 The general consensus from these guidelines is that, at minimum, athletes should not participate in any physical activity until they are asymptomatic and all blood tests have normalized.11 Following this asymptomatic period, most guidelines advocate for a slow, deliberate return to activity.11 The American College of Sports Medicine (ACSM) offers one reasonable approach to the returning athlete following EHS:7

  • No exercise for at least 7 days following release from medical care.
  • Follow-up with a physician 1 week after release from medical care for physical examination and any warranted lab or radiologic studies (based upon organ systems affected during EHS).
  • Once cleared to return to activity, the athlete begins exercise in a cool environment, gradually increasing the duration, intensity, and heat exposure over 2 weeks to demonstrate heat tolerance and acclimatization.
  • Athletes who cannot resume vigorous activity due to recurrent symptoms (eg, excessive fatigue) should be reevaluated after 4 weeks. Laboratory exercise-heat tolerance testing may be useful in this setting.
  • The athlete may resume full competition once they are able to participate in full training in the heat for 2 to 4 weeks without adverse effects.

Heat tolerance testing (HTT) in these athletes remains controversial.5 26 The ACSM recommends that HTT be considered only for those unable to return to vigorous activity after a suitable period (approximately 4 weeks). In contrast, the Israeli Defense Force (IDF) uses HTT to evaluate soldiers following EHS to guide decision-making about return to duty.27 The IDF HTT assumes that individuals will respond differently to heat stresses. They identify individuals who are “heat intolerant” as being unable to tolerate specific heat challenges, indicated by increases in body temperature occurring more rapidly than normal responders under identical environmental and exercise conditions. However, despite being used for more than 30 years, there is no clear evidence that HTT adequately predicts who will experience subsequent episodes of EHS.

Conclusion

While the recognized cornerstone of being a team physician is the provision of medical care, the ACSM Team Physician Consensus Statement28 further delineates the medical and administrative responsibilities as both (1) understanding medical management and prevention of injury and illness in athletes; and (2) awareness of or involvement in the development and rehearsal of an emergency action plan. These tenets are critical for the team physician who accepts the responsibility to cover sports at the high school level or higher. Football team physicians play an essential role in mitigating risk of EHI in their athletes. Through development and execution of both comprehensive prevention strategies and emergency action plans, physicians can work to minimize athletes’ risk of both developing and experiencing significant adverse outcomes from an EHI.

 

Am J Orthop. 2016;45(6):340-348. Copyright Frontline Medical Communications Inc. 2016. All rights reserved.

Football, one of the most popular sports in the United States, is additionally recognized as a leading contributor to sports injury secondary to the contact collision nature of the endeavor. There are an estimated 1.1 million high school football players with another 100,000 participants combined in the National Football League (NFL), college, junior college, Arena Football League, and semipro levels of play.1 USA Football estimates that an additional 3 million youth participate in community football leagues.1 The National Center for Catastrophic Sports Injury Research recently calculated a fatality rate of 0.14 per 100,000 participants in 2014 for the 4.2 million who play football at all levels—and 0.45 per 100,000 in high school.1 While direct deaths from head and spine injury remain a significant contributor to the number of catastrophic injuries, indirect deaths (systemic failure) predominate. Exertional heat stroke (EHS) has emerged as one of the leading indirect causes of death in high school and collegiate football. Boden and colleagues2 reported that high school and college football players sustain approximately 12 fatalities annually, with indirect systemic causes being twice as common as direct blunt trauma.2The most common indirect causes identified included cardiac failure, heat illness, and complications of sickle cell trait (SCT). It was also noted that the risk of SCT, heat-related, and cardiac deaths increased during the second decade of the study, indicating these conditions may require a greater emphasis on diagnosis, treatment, and prevention. This review details for the team physician the unique challenge of exercising in the heat to the football player, and the prevention, diagnosis, management and return-to-play issues pertinent to exertional heat illness (EHI).

The Challenge

EHS represents the most severe manifestation of EHI—a gamut of diseases commonly encountered during the hot summer months when American football season begins. The breadth of EHI includes several important clinical diagnoses: exercise-associated muscle cramps (heat cramps); heat exhaustion with and without syncope; heat injury with evidence of end organ injury (eg, rhabdomyolysis); and EHS. EHS is defined as “a form of hyperthermia associated with a systemic inflammatory response leading to a syndrome of multi-organ dysfunction in which encephalopathy predominates.”3 EHS, if left untreated, or even if clinical treatment is delayed, may result in significant end organ morbidity and/or mortality.

During exercise, the human thermoregulatory system mitigates heat gain by increasing skin blood flow and sweating, causing an increased dissipation of heat to the surrounding environment by leveraging conduction, convection, and evaporation.4,5 Elevated environmental temperatures, increased humidity, and dehydration can impede the body’s ability to dissipate heat at a rate needed to maintain thermoregulation. This imbalance can result in hyperthermia secondary to uncompensated heat stress,5 which in turn can lead to EHI. Football players have unique challenges that make them particularly vulnerable to EHI. The summer heat during early-season participation and the requirement for equipment that covers nearly 60% of body surfaces pose increased risk of volume losses and hyperthermia that trigger the onset of EHI.6 Football athletes’ body compositions and physical size are additional contributing risk factors; the relatively high muscle and fat content increase thermogenicity, which require their bodies to dissipate more heat.7

An estimated 9000 cases of EHI occur annually across all high school sports,8 with an incidence of 1.6:100,000 athlete-exposures.8,9 Studies have demonstrated, however, that EHI occurs in football 11.4 times more often than in all other high school sports combined.10 The incidence of nonfatal EHI in all levels of football is 4.42-5:100,000.8,9 Between 2000 and 2014, 41 football players died from EHS.1 In football, approximately 75% of all EHI events occurred during practices, while only 25% of incidents occurred during games.8

Given these potentially deadly consequences, it is important that football team physicians are not only alert to the early symptoms of heat illness and prepared to intervene to prevent the progression to EHS, but are critical leaders in educating coaches and players in evidence-based EHI prevention practices and policies.

Prevention

EHS is a preventable condition, arguably the most common cause of preventable nontraumatic exertional death in young athletes in the United States. Close attention to mitigating risk factors should begin prior to the onset of preseason practice and continue through the early season, where athletes are at the highest risk of developing heat illness.

Primary Prevention

Primary prevention is fundamental to minimizing the occurrences of EHI. It focuses on the following methods: recognition of inherent risk factors, acclimatization, hydration, and avoidance of inciting substances (including supplements).

 

 

Pre-Participation Examination. The purpose of the pre-participation examination (PPE) is to maximize an athlete’s safety by identifying medical conditions that place the athlete at risk.11,12 The Preparticipation Physical Evaluation, 4th edition, the most widely used consensus publication, specifically queries if an athlete has a previous history of heat injury. However, it only indirectly addresses intrinsic risk factors that may predispose an athlete to EHI who has never had an EHI before. Therefore, providers should take the opportunity of the PPE to inquire about additional risk factors that may make an athlete high risk for sustaining a heat injury. Common risk factors for EHI are listed in Table 1.

Table 1
While identifying at-risk athletes is important in mitigating the risk of developing EHI, it will not identify all possible cases: a study of military recruits found that up to 50% of Marines who developed EHI lacked an identifiable risk factor.13Wet Bulb Globe Temperature. Humidity can heighten a player’s risk of developing thermogenic dysregulation during hot temperatures. As ambient temperature nears internal body temperature, heat may actually be absorbed by the skin rather than dissipated into the air. As a result, the body must increasingly rely on sweat evaporation to encourage heat loss; this process is hindered in very humid climates. Wet bulb globe temperature (WBGT) is a measure of heat stress that accounts for temperature, humidity, wind speed, and cloud cover. WBGT should be utilized to determine the relative risk of EHI based on local environmental conditions, as there is a direct correlation between elevated WBGT and risk of EHI.11,14 The greatest risk for EHS is performing high-intensity exercise (>75% VO2max) when WBGT >28°C (82°F).7 A study of hyperthermia-related deaths in football found that a majority of fatalities occurred on days classified as high risk (23°C-28°C) or extreme risk (>28°C) by WBGT.14 Consensus guidelines recommend that activities be modified based on WBGT (Table 2).7,12
Table 2.
The impact of WBGT does not end solely on the day of practice. Athletes who exercise in elevated WBGT environments on 2 consecutive days are at increased risk of EHI due to cumulative effects of exercise in heat.11Clothing. In football, required protective equipment may cover up to 60% of body surfaces. Studies have shown that wearing full uniform with pads increases internal body temperature and decreases time to exhaustion when compared to light clothing.5,15 In addition, athletic equipment traps heat close to the body and inhibits evaporation of sweat into the environment, thereby inhibiting radiant and evaporative heat dissipation.5,11 Likewise, wearing dark clothing encourages radiant absorption of heat, further contributing to potential thermal dysregulation.5 Use of a helmet is a specific risk factor for EHI, as significant heat dissipation occurs through the head.11 To mitigate these risk factors, the introduction of padded equipment should occur incrementally over the heat acclimatization period (see below). In addition, athletes should be encouraged to remove their helmets during rest periods to promote added heat dissipation and recovery.

Heat Acclimatization. The risk of EHI escalates significantly when athletes are subjected to multiple stressors during periods of heat exposure, such as sudden increases in intensity or duration of exercise; prolonged new exposures to heat; dehydration; and sleep loss.5 When football season begins in late summer, athletes are least conditioned as temperatures reach their seasonal peak, causing increased risk of EHI.15 Planning for heat acclimatization is vital for all athletes who exercise in hot environments. Acclimatization procedures place progressively mounting physiologic strains on the body to improve athletes’ ability to dissipate heat, diminishing thermoregulatory and cardiovascular exertion.4,5 Acclimatization begins with expansion of plasma volume on days 3 to 6, causing improvements in cardiac efficiency and resulting in an overall decrease in basal internal body temperature.4,5,15 This process results in improvements in heat tolerance and exercise performance, evolving over 10 to 14 days of gradual escalation of exercise intensity and duration.5,10,11,16 However, poor fitness levels and extreme temperatures can prolong this period, requiring up to 2 to 3 months to fully take effect.5,7

The National Athletic Trainers Association (NATA) and National Collegiate Athletic Association (NCAA) have released consensus guidelines regarding heat acclimatization protocols for football athletes at the high school and college levels (Tables 3 and 4). Each of these guidelines involves an initial period without use of protective equipment, followed by a gradual addition of further equipment.11,16
Table 3.

Secondary Prevention

Despite physicians’ best efforts to prevent all cases of EHI, athletes will still experience the effects of exercise-induced hyperthermia. The goal of secondary prevention is to slow the progression of this hyperthermia so that it does not progress to more dangerous EHI.

Table 4.

 

 

Hydration. Dehydration is an important risk factor for EHI. Sweat maintains thermoregulation by dissipating heat generated during exercise; however, it also contributes to body water losses. Furthermore, intravascular depletion decreases stroke volume, thereby increasing cardiovascular strain. It is estimated that for every 1% loss in body mass from dehydration, body temperature rises 0.22°C in comparison to a euhydrated state.6 Dehydration occurs more rapidly in hot environments, as fluid is lost through increased sweat production.7 After approximately 6% to 10% body weight volume loss, cardiac output cannot be maintained, diminishing sweat production and blood flow to both skin and muscle and causing diminished performance and a significant risk of heat exhaustion.7 If left unchecked, these physiologic changes result in further elevations in body temperature and increased cardiovascular strain, ultimately placing the athlete at significant risk for development of EHS.

Adequate hydration to maintain euvolemia is an important step in avoiding possible EHI. Multiple studies have shown that football players experience a baseline hypovolemia during their competition season,6 a deficit that is most marked during the first week of practices.17 This deficit is multifactorial, as football players expend a significant amount of fluid through sweat, are not able to adequately replace these losses during practice, and do not appropriately hydrate off the field.6,18 Some players, especially linemen, sweat at a higher rate than their teammates, posing a possible risk of significant dehydration.6 Coaches and players alike should be educated on the importance of adequate hydration to meet their fluid needs.

The goal of hydration during exercise is to prevent large fluid losses that can adversely affect performance and increase risk of EHI;6 it may be unrealistic to replace all fluid losses during the practice period. Instead, athletes should target complete volume replacement over the post-exercise period.6 Some recommend hydrating based upon thirst drive; however, thirst is activated following a volume loss of approximately 2% body mass, the same degree of losses that place athletes at an increased risk for performance impairment and EHI.4,6,11,12 Individuals should have access to fluids throughout practice and competition and be encouraged to hydrate as needed.6,12,15 Furthermore, staff should modify their practices based upon WBGT and acclimatization status to provide more frequent hydration breaks.

Hyperhydration and Salt Intake. Of note, there are inherent risks to hyperhydration. Athletes with low sweat rates have an increased risk of overhydration and the development of exercise-associated hyponatremia (EAH),6 a condition whose presentation is very similar to EHS. In addition, inadequate sodium intake and excessive sweating can also contribute to the development of EAH. EAH has been implicated in the deaths of 2 football players in 2014.1,6 Establishing team hydration guidelines and educating players and staff on appropriate hydration and dietary salt intake is essential to reduce the risk of both dehydration and hyperhydration and their complications.6Intra-Event Cooling. During exercise, team physicians can employ strategies for cooling athletes during exertion to mitigate their risk of EHI by decreasing thermal and cardiovascular strain.4,19 Cooling during exercise is hypothesized to allow for accelerated heat dissipation, where heat is lost from the body more effectively. This accelerated loss enables athletes to maintain a higher heat storage capacity over the duration of exercise, avoiding uncompensated heat stresses that ultimately cause EHI.19

Some intra-event cooling strategies include the use of cooling garments, cooling packs, and cold water/slurry ingestion. Cooling garments lower skin temperature, which in turn can decrease thermoregulatory strains;4 a recent meta-analysis of intra-event cooling modalities revealed that wearing an ice vest during exercise resulted in the greatest decrease in thermal heat strain.19 Internal cooling strategies—namely ingestion of cold fluids/ice slurry—have shown some mild benefit in decreasing internal temperatures; however, some studies have demonstrated some decrease in sweat production associated with cold oral intake used in isolation.19 Overall, studies have shown that combining external (cooling clothing, ice packs, fanning) and internal (cold water, ice slurry) cooling methods result in a greater cooling effect than a use of a single method.4

Tertiary Prevention

The goal of tertiary prevention is to mitigate the risk of long-term adverse outcomes following an EHS event. The most effective means of reducing risk for morbidity and mortality is rapid identification and treatment of EHS as well as close evaluation of an athlete’s return to activity in heat. This process is spearheaded by an effective and well-rehearsed emergency action plan.

Diagnosis and Management

Rapid identification and treatment of EHS is crucial to minimizing the risk of poor outcomes.7 Any delay in the treatment of EHS can dramatically increase the likelihood of associated morbidity and mortality.20

 

 

EHS is diagnosed by an elevated rectal temperature ≥40°C (104°F) and associated central nervous system (CNS) dysfunction.21 EHS should be strongly suspected in any athlete exercising in heat who exhibits signs of CNS dysfunction, including disorientation, confusion, dizziness, erratic behavior, irritability, headache, loss of coordination, delirium, collapse, or seizures.7,12,15 EHS may also present with symptoms of heat exhaustion, including fatigue, hyperventilation, tachycardia, vomiting, diarrhea, and hypotension.7,12,15

Rectal temperature should be taken for any athlete with suspected EHS, as other modalities—oral, skin, axillary, and aural—can be inaccurate and easily modified by ambient confounders such as ambient and skin temperature, athlete hyperventilation, and consumption of liquids.7,11,12 Athletes exhibiting CNS symptoms with moderately elevated rectal temperatures that do not exceed 40°C should also be assumed to be suffering from EHS and treated with rapid cooling.11 On the other hand, athletes with CNS symptoms who are normothermic should be assumed to have EAH until ruled out by electrolyte assessment; IV fluids should be at no more than keep vein open (KVO) pending this determination.11 In some cases, an athlete may initially present with altered mental status but return to “normal.” However, this improvement may represent a “lucid period”; evaluation should continue with rectal temperature and treatment, as EHS in these cases may progress quickly.15

Treatment is centered on rapid, whole body cooling initiated at the first sign of heat illness.7,22 The goal of treatment is to achieve a rectal temperature <38.9°C within 30 minutes of the onset of EHS.15 Upon diagnosis, the athlete should be quickly placed in a tub of ice water to facilitate cold water immersion (CWI) therapy. Some guidelines suggest the athlete’s clothing be removed to potentiate evaporative cooling during CWI;12 however, cooling should not be delayed due to difficulties in removing equipment. CWI, where a heat stroke victim is submerged in ice water up to their neck while water is continuously circulated, is generally considered to be the gold standard treatment as it is the modality with the highest recorded cooling rates and the lowest rate of morbidity and mortality.7,20,21 Multiple studies of CWI have shown that survival nears 100% when aggressive cooling starts within 5 minutes of collapse or identification of EHS.20,21,22

If whole body CWI is unavailable, alternative methods of rapid cooling should be employed. Partial CWI, with torso immersion being preferable to the extremities, has been shown to achieve an acceptable rate of cooling to achieve sufficient drops in internal body temperature.20,23 However, one popular treatment—applying ice packs to the whole body, in particular to the groin and axillae—has not been shown to be sufficient to achieve standard cooling goals.20 None of these methods have been shown to be as effective as CWI.23

Intravenous access should be initiated with fluid resuscitation dictated by the provider’s assessment. Normal saline is recommended as the resuscitative fluid of choice, with the rate dictated by clinical judgment and adjusted as guided by electrolyte determination and clinical response. It cannot be overstated that in normothermic patients with confusion, EAH is the diagnosis of exclusion and aggressive fluid resuscitation should be withheld until electrolyte determination.

Once rectal temperature descends appropriately (~38.9°C), the cooling process should stop and the individual should be transported to a hospital for further observation20 and evaluation of possible sequelae, including rhabdomyolysis and renal injury, cardiac dysfunction and arrhythmia, severe electrolyte abnormalities, acute respiratory distress syndrome, lactic acidosis, and other forms of end-organ failure (Figure).

Figure.


Rapid cooling is more crucial than transport; transport poses a risk of delayed cooling, which can dramatically increase an individual’s risk of morbidity and mortality.20,23 In situations where a patient can be cooled on-site, physicians should pursue cooling before transporting the patient to a medical treatment facility.

Emergency Action Plan

Team physicians should be proactive in developing an emergency action plan to address possible EHS events. These plans should be site-specific, addressing procedures for all practice and home competition locations.12 All competition venues should have a CWI tub on-site in events where there is an increased risk of EHS.12,15,20 This tub should be set up and functional for all high-risk activities, including practices.12

Following recognition of a potential case of EHS, treatment teams should have procedures in place to transport athletes to the treatment area, obtain rectal temperature, initiate rapid cooling, and stabilize the athlete for transport to an emergency department (ED) for further evaluation.12,15 A written record of treatments and medications provided during athlete stabilization should be maintained and transported with the athlete to the ED.15 A list of helpful equipment and supplies for treatment of EHS can be found in Table 5.

Table 5.


EHS is a unique life-threatening situation where it is best to treat the patient on the sideline before transport.15 Those athletes transported before cooling risk spending an increased amount of time above critical temperatures for cell damage, which has been associated with increased morbidity and mortality. This mantra of “cool first, transport second” cannot be overemphasized, as those individuals with EHS who present to the ED with a persisting rectal temperature >41°F may risk up to an 80% mortality rate.24 Conversely, a recent large, retrospective study of 274 EHS events sustained during the Falmouth Road Race found a 100% survival rate when athletes were rapidly identified via rectal thermometry and treated with aggressive, rapid cooling through CWI.21

 

 

Return to Play

Perhaps the most challenging and important role the team physician has is determining an athlete’s return to play following EHI, as there currently are no evidence-based guidelines for return to activity for these athletes.7 The decisions surrounding return to play are highly individualized, as recovery from EHS and heat injury is associated with the duration of internal body temperature elevation above the critical level (40°C).7,20 Guidelines for return to activity following recovery from EHI differ among experts and institutions.7,25 The general consensus from these guidelines is that, at minimum, athletes should not participate in any physical activity until they are asymptomatic and all blood tests have normalized.11 Following this asymptomatic period, most guidelines advocate for a slow, deliberate return to activity.11 The American College of Sports Medicine (ACSM) offers one reasonable approach to the returning athlete following EHS:7

  • No exercise for at least 7 days following release from medical care.
  • Follow-up with a physician 1 week after release from medical care for physical examination and any warranted lab or radiologic studies (based upon organ systems affected during EHS).
  • Once cleared to return to activity, the athlete begins exercise in a cool environment, gradually increasing the duration, intensity, and heat exposure over 2 weeks to demonstrate heat tolerance and acclimatization.
  • Athletes who cannot resume vigorous activity due to recurrent symptoms (eg, excessive fatigue) should be reevaluated after 4 weeks. Laboratory exercise-heat tolerance testing may be useful in this setting.
  • The athlete may resume full competition once they are able to participate in full training in the heat for 2 to 4 weeks without adverse effects.

Heat tolerance testing (HTT) in these athletes remains controversial.5 26 The ACSM recommends that HTT be considered only for those unable to return to vigorous activity after a suitable period (approximately 4 weeks). In contrast, the Israeli Defense Force (IDF) uses HTT to evaluate soldiers following EHS to guide decision-making about return to duty.27 The IDF HTT assumes that individuals will respond differently to heat stresses. They identify individuals who are “heat intolerant” as being unable to tolerate specific heat challenges, indicated by increases in body temperature occurring more rapidly than normal responders under identical environmental and exercise conditions. However, despite being used for more than 30 years, there is no clear evidence that HTT adequately predicts who will experience subsequent episodes of EHS.

Conclusion

While the recognized cornerstone of being a team physician is the provision of medical care, the ACSM Team Physician Consensus Statement28 further delineates the medical and administrative responsibilities as both (1) understanding medical management and prevention of injury and illness in athletes; and (2) awareness of or involvement in the development and rehearsal of an emergency action plan. These tenets are critical for the team physician who accepts the responsibility to cover sports at the high school level or higher. Football team physicians play an essential role in mitigating risk of EHI in their athletes. Through development and execution of both comprehensive prevention strategies and emergency action plans, physicians can work to minimize athletes’ risk of both developing and experiencing significant adverse outcomes from an EHI.

 

Am J Orthop. 2016;45(6):340-348. Copyright Frontline Medical Communications Inc. 2016. All rights reserved.

References

1. Kucera KL, Klossner D, Colgate B, Cantu RC. Annual Survey of Football Injury Research: 1931-2014. National Center for Catastrophic Sport Injury Research Web site. https://nccsir.unc.edu/files/2013/10/Annual-Football-2014-Fatalities-Final.pdf. Accessed May 31, 2016.

2. Boden BP, Breit I, Beachler JA, Williams A, Mueller FO. Fatalities in high school and college football players. Am J Sports Med. 2013;41(5):1108-1116.

3. Bouchama A, Knochel JP. Heat stroke. N Engl J Med. 2002;346(25):1978-1988.

4. Racinais S, Alonso JM, Coutts AJ, et al. Consensus recommendations on training and competing in the heat. Scand J Med Sci Sports. 2015;25 Suppl 1:6-19.

5. Pryor RR, Casa DJ, Adams WM, et al. Maximizing athletic performance in the heat. Strength Cond J. 2013;35(6):24-33.

6. Adams WM, Casa DJ. Hydration for football athletes. Sports Sci Exchange. 2015;28(141):1-5.

7. American College of Sports Medicine, Armstrong LE, Casa DJ, et al. American College of Sports Medicine position stand. Exertional heat illness during training and competition. Med Sci Sports Exerc. 2007;39(3):556-572.

8. Yard EE, Gilchrist J, Haileyesus T, et al. Heat illness among high school athletes--United States, 2005-2009. J Safety Res. 2010;41(6):471-474.

9. Huffman EA, Yard EE, Fields SK, Collins CL, Comstock RD. Epidemiology of rare injuries and conditions among United States high school athletes during the 2005-2006 and 2006-2007 school years. J Athl Train. 2008;43(6):624-630.

10. Kerr ZY, Casa DJ, Marshall SW, Comstock RD. Epidemiology of exertional heat illness among U.S. high school athletes. Am J Prev Med. 2013;44(1):8-14.

11. Casa DJ, DeMartini JK, Bergeron MF, et al. National Athletic Trainers’ Association position statement: exertional heat illnesses. J Athl Train. 2015;50(9):986-1000.

12. Casa DJ, Almquist J, Anderson SA. The inter-association task force for preventing sudden death in secondary school athletics programs: best-practices recommendations. J Athl Train. 2013;48(4):546-553.

13. Gardner JW, Kark JA, Karnei K, et al. Risk factors predicting exertional heat illness in male Marine Corps recruits. Med Sci Sports Exerc. 1996;28(8):939-944.

14. Gundstein AJ, Ramseyer C, Zhao F, et al. A retrospective analysis of American football hyperthermia deaths in the United States. Int J Biometerol. 2012;56(1):11-20.

15. Armstrong LE, Johnson EC, Casa DJ, et al. The American football uniform: uncompensable heat stress and hyperthermic exhaustion. J Athl Train. 2010;45(2):117-127.

16. Casa DJ, Csillan D; Inter-Association Task Force for Preseason Secondary School Athletics Participants, et al. Preseason heat-acclimatization guidelines for secondary school athletics. J Athl Train. 2009;44(3):332-333.

17. Godek SF, Godek JJ, Bartolozzi AR. Hydration status in college football players during consecutive days of twice-a-day preseason practices. Am J Sports Med. 2005;33(6):843-851.

18. Stover EA, Zachwieja J, Stofan J, Murray R, Horswill CA. Consistently high urine specific gravity in adolescent American football players and the impact of an acute drinking strategy. Int J Sports Med. 2006;27(4):330-335.

19. Bongers CC, Thijssen DH, Veltmeijer MTW, Hopman MT, Eijsvogels TM. Precooling and percooling (cooling during exercise) both improve performance in the heat: a meta-analytical review. Br J Sports Med. 2015;49(6):377-384.

20. Casa DJ, McDermott BP, Lee EC, Yeargin SW, Armstrong LE, Maresh CM. Cold water immersion: the gold standard for exertional heatstroke treatment. Exerc Sport Sci Rev. 2007;35(3):141-149.

21. DeMartini JK, Casa DJ, Stearns R, et al. Effectiveness of cold water immersion in the treatment of exertional heat stroke at the Falmouth Road Race. Med Sci Sports Exerc. 2015;47(2):240-245.

22. Casa DJ, Kenny GP, Taylor NA. Immersion treatment for exertional hyperthermia: cold or temperate water? Med Sci Sports Exerc. 2010;42(7):1246-1252.

23. Casa DJ, Armstrong LE, Kenny GP, O’Connor FG, Huggins RA. Exertional heat stroke: new concepts regarding cause and care. Curr Sports Med Rep. 2012;11(3):115-122.

24. Argaud L, Ferry T, Le QH, et al. Short- and long-term outcomes of heat stroke following the 2003 heat wave in Lyon, France. Arch Intern Med. 2007;167(20):2177-2183.

25. O’Connor FG, Casa DJ, Bergeron MF, et al. American College of Sports Medicine Roundtable on exertional heat stroke--return to duty/return to play: conference proceedings. Curr Sports Med Rep. 2010;9(5):314-321.

26. Kazman JB, Heled Y, Lisman PJ, Druyan A, Deuster PA, O’Connor FG. Exertional heat illness: the role of heat tolerance testing. Curr Sports Med Rep. 2013;12(2):101-105.

27. Moran DS, Heled Y, Still L, Laor A, Shapiro Y. Assessment of heat tolerance for post exertional heat stroke individuals. Med Sci Monit. 2004;10(6):CR252-CR257.

28. Herring SA, Kibler WB, Putukian M. Team Physician Consensus Statement: 2013 update. Med Sci Sports Exerc. 2013;45(8):1618-1622.

29. Heat stroke treatment. Korey Stringer Institute University of Connecticut Web site. http://ksi.uconn.edu/emergency-conditions/heat-illnesses/exertional-heat-stroke/heat-stroke-treatment/. Accessed June 14, 2016.

30. Headquarters, Department of the Army and the Air Force. Heat Stress Control and Heat Casualty Management. Technical Bulletin Medical 507. http://www.dir.ca.gov/oshsb/documents/Heat_illness_prevention_tbmed507.pdf. Published March 7, 2003. Accessed June 14, 2016.

References

1. Kucera KL, Klossner D, Colgate B, Cantu RC. Annual Survey of Football Injury Research: 1931-2014. National Center for Catastrophic Sport Injury Research Web site. https://nccsir.unc.edu/files/2013/10/Annual-Football-2014-Fatalities-Final.pdf. Accessed May 31, 2016.

2. Boden BP, Breit I, Beachler JA, Williams A, Mueller FO. Fatalities in high school and college football players. Am J Sports Med. 2013;41(5):1108-1116.

3. Bouchama A, Knochel JP. Heat stroke. N Engl J Med. 2002;346(25):1978-1988.

4. Racinais S, Alonso JM, Coutts AJ, et al. Consensus recommendations on training and competing in the heat. Scand J Med Sci Sports. 2015;25 Suppl 1:6-19.

5. Pryor RR, Casa DJ, Adams WM, et al. Maximizing athletic performance in the heat. Strength Cond J. 2013;35(6):24-33.

6. Adams WM, Casa DJ. Hydration for football athletes. Sports Sci Exchange. 2015;28(141):1-5.

7. American College of Sports Medicine, Armstrong LE, Casa DJ, et al. American College of Sports Medicine position stand. Exertional heat illness during training and competition. Med Sci Sports Exerc. 2007;39(3):556-572.

8. Yard EE, Gilchrist J, Haileyesus T, et al. Heat illness among high school athletes--United States, 2005-2009. J Safety Res. 2010;41(6):471-474.

9. Huffman EA, Yard EE, Fields SK, Collins CL, Comstock RD. Epidemiology of rare injuries and conditions among United States high school athletes during the 2005-2006 and 2006-2007 school years. J Athl Train. 2008;43(6):624-630.

10. Kerr ZY, Casa DJ, Marshall SW, Comstock RD. Epidemiology of exertional heat illness among U.S. high school athletes. Am J Prev Med. 2013;44(1):8-14.

11. Casa DJ, DeMartini JK, Bergeron MF, et al. National Athletic Trainers’ Association position statement: exertional heat illnesses. J Athl Train. 2015;50(9):986-1000.

12. Casa DJ, Almquist J, Anderson SA. The inter-association task force for preventing sudden death in secondary school athletics programs: best-practices recommendations. J Athl Train. 2013;48(4):546-553.

13. Gardner JW, Kark JA, Karnei K, et al. Risk factors predicting exertional heat illness in male Marine Corps recruits. Med Sci Sports Exerc. 1996;28(8):939-944.

14. Gundstein AJ, Ramseyer C, Zhao F, et al. A retrospective analysis of American football hyperthermia deaths in the United States. Int J Biometerol. 2012;56(1):11-20.

15. Armstrong LE, Johnson EC, Casa DJ, et al. The American football uniform: uncompensable heat stress and hyperthermic exhaustion. J Athl Train. 2010;45(2):117-127.

16. Casa DJ, Csillan D; Inter-Association Task Force for Preseason Secondary School Athletics Participants, et al. Preseason heat-acclimatization guidelines for secondary school athletics. J Athl Train. 2009;44(3):332-333.

17. Godek SF, Godek JJ, Bartolozzi AR. Hydration status in college football players during consecutive days of twice-a-day preseason practices. Am J Sports Med. 2005;33(6):843-851.

18. Stover EA, Zachwieja J, Stofan J, Murray R, Horswill CA. Consistently high urine specific gravity in adolescent American football players and the impact of an acute drinking strategy. Int J Sports Med. 2006;27(4):330-335.

19. Bongers CC, Thijssen DH, Veltmeijer MTW, Hopman MT, Eijsvogels TM. Precooling and percooling (cooling during exercise) both improve performance in the heat: a meta-analytical review. Br J Sports Med. 2015;49(6):377-384.

20. Casa DJ, McDermott BP, Lee EC, Yeargin SW, Armstrong LE, Maresh CM. Cold water immersion: the gold standard for exertional heatstroke treatment. Exerc Sport Sci Rev. 2007;35(3):141-149.

21. DeMartini JK, Casa DJ, Stearns R, et al. Effectiveness of cold water immersion in the treatment of exertional heat stroke at the Falmouth Road Race. Med Sci Sports Exerc. 2015;47(2):240-245.

22. Casa DJ, Kenny GP, Taylor NA. Immersion treatment for exertional hyperthermia: cold or temperate water? Med Sci Sports Exerc. 2010;42(7):1246-1252.

23. Casa DJ, Armstrong LE, Kenny GP, O’Connor FG, Huggins RA. Exertional heat stroke: new concepts regarding cause and care. Curr Sports Med Rep. 2012;11(3):115-122.

24. Argaud L, Ferry T, Le QH, et al. Short- and long-term outcomes of heat stroke following the 2003 heat wave in Lyon, France. Arch Intern Med. 2007;167(20):2177-2183.

25. O’Connor FG, Casa DJ, Bergeron MF, et al. American College of Sports Medicine Roundtable on exertional heat stroke--return to duty/return to play: conference proceedings. Curr Sports Med Rep. 2010;9(5):314-321.

26. Kazman JB, Heled Y, Lisman PJ, Druyan A, Deuster PA, O’Connor FG. Exertional heat illness: the role of heat tolerance testing. Curr Sports Med Rep. 2013;12(2):101-105.

27. Moran DS, Heled Y, Still L, Laor A, Shapiro Y. Assessment of heat tolerance for post exertional heat stroke individuals. Med Sci Monit. 2004;10(6):CR252-CR257.

28. Herring SA, Kibler WB, Putukian M. Team Physician Consensus Statement: 2013 update. Med Sci Sports Exerc. 2013;45(8):1618-1622.

29. Heat stroke treatment. Korey Stringer Institute University of Connecticut Web site. http://ksi.uconn.edu/emergency-conditions/heat-illnesses/exertional-heat-stroke/heat-stroke-treatment/. Accessed June 14, 2016.

30. Headquarters, Department of the Army and the Air Force. Heat Stress Control and Heat Casualty Management. Technical Bulletin Medical 507. http://www.dir.ca.gov/oshsb/documents/Heat_illness_prevention_tbmed507.pdf. Published March 7, 2003. Accessed June 14, 2016.

Issue
The American Journal of Orthopedics - 45(6)
Issue
The American Journal of Orthopedics - 45(6)
Page Number
340-348
Page Number
340-348
Publications
Publications
Topics
Article Type
Display Headline
Exertional Heat Stroke and American Football: What the Team Physician Needs to Know
Display Headline
Exertional Heat Stroke and American Football: What the Team Physician Needs to Know
Sections
Disallow All Ads
Article PDF Media

Woman, 36, With Fever and Malaise

Article Type
Changed
Mon, 07/09/2018 - 10:47

IN THIS ARTICLE

  • Clinical presentation and evaluation
  • Terminology table
  • Outcome for the case patient

A 36-year-old Bengali woman with a history of well-controlled diabetes presents to the emergency department with complaints of feeling “unwell” for about two weeks. She does not speak English, and a hospital-provided phone translator is used to obtain history and explain hospital course. The patient is vague regarding symptomatology, describing general malaise and tiredness. She says she became “much worse” two days ago and has shaking chills, sore throat, headache, and nonproductive cough, but she denies shortness of breath or chest pain. She also developed nausea and vomiting, stating, “I can’t keep anything down.”

She has not recently traveled out of the country and has no known sick contacts. Influenza activity is high in the area, and the patient has not received immunization. She had a “normal” menstrual period two weeks ago and firmly states, “There is no way I can be pregnant.” She admits to vaginal “spotting” off and on for the past two weeks without abdominal pain. She is married with six children and has no history of miscarriage, ectopic pregnancy, or induced abortion; she is not taking any form of birth control.

On exam, the patient is tachycardic, with a heart rate of 127 beats/min, and has a fever of 103.3°F. Blood pressure, respiratory rate, and pulse oximetry are normal. She appears unwell and dehydrated. Her mucous membranes are dry, but no skin rash is noted. There is no tonsillar swelling or exudate and no meningismus; the lung exam is clear, with no adventitious sounds. Abdominal exam demonstrates mild, generalized tenderness in the lower abdomen without peritoneal signs. No costovertebral angle tenderness is noted. Initial diagnostic considerations include sources of fever (eg, influenza, pneumonia, urinary tract infection, viral illness), or abdominal sources, such as appendicitis.

An upright anteroposterior chest x-ray shows no infiltrate, pleural effusion, or cardiomegaly. Laboratory results include a high white blood cell (WBC) count (16.9 k/mm3) with bandemia and normal electrolytes without anion gap. Rapid influenza A and B testing is negative. A urine pregnancy test is positive, and the urinalysis shows no infection but +2 ketones. Rh factor is positive. A serum quantitative β-hCG is 130,581 mIU/mL. Blood cultures are obtained, but results are not available.

Due to cultural differences, the patient is very reluctant to consent to a pelvic exam. After extensive counseling, she agrees to a bimanual exam only. The uterus is boggy and enlarged to about 12 weeks. There is exquisite uterine tenderness and purulent discharge on the gloved finger. The cervical os is closed, and there is scant bleeding.

A transvaginal ultrasound is obtained; it reveals a thickened endometrium with echogenicity, without increased vascularity, and no identifiable intrauterine pregnancy. The adnexa have no masses, and there is no free fluid in the endometrium (see Figures 1 and 2).

The patient is given broad-spectrum antibiotics and urgently transported to the operating room by Ob-Gyn for uterine evacuation. She is found to have a septic abortion due to retained products of conception (RPOC) from an incomplete miscarriage.

Continue for discussion >>

 

 

DISCUSSIONIt is not uncommon for a woman to miscarry a very early pregnancy and not realize she had been pregnant.1 Many attribute it to a “heavy” or unusual period. In one study, 11% of patients who denied the possibility of pregnancy were, in fact, pregnant.2

Miscarriage is a frequent outcome of early pregnancy; it is estimated that 11% to 20% of early pregnancies result in a spontaneous miscarriage.3-5 Most resolve without complications, but risk increases with gestational age. When they do occur, complications include RPOC, heavy prolonged bleeding, and endometritis. RPOC refers to placental or fetal tissue that remains in the uterus after a miscarriage, surgical abortion, or preterm/term delivery (see Table for additional terminology related to miscarriage and abortion). Because of increased morbidity, it is important to suspect RPOC after a known miscarriage or an induced abortion, or in a pregnant patient with bleeding.

Incidence and pathophysiologySeptic abortion is a relatively rare complication of miscarriage. It can refer to a spontaneous miscarriage complicated by a subsequent intrauterine infection, often caused by RPOC. Septic abortion is much more common after an induced abortion, in which there is instrumentation of the uterus.

The infection after a spontaneous miscarriage usually begins as endometritis. It involves the necrotic RPOC, which are prone to infection by the cervical and vaginal flora. It may spread further into the parametrium/myometrium and the peritoneal cavity. The infection may then progress to bacteremia and sepsis. Typical causative organisms include Escherichia coli, Enterobacter aerogenes, Proteus vulgaris, hemolytic streptococci, staphylococci, and some anaerobic organisms, including Clostridium perfringens.3

Death, although rare in developed countries, is usually secondary to the sequela of sepsis, including septic shock, renal failure, adult respiratory distress syndrome, and disseminated intravascular coagulation.3,6,7 Pelvic adhesions and hysterectomy are also possible outcomes of a septic abortion.

Continue for clinical presentation and evaluation >>

 

 

Clinical presentation and evaluationMany findings suggestive of septic abortion are nonspecific, such as bleeding, pain, uterine tenderness, and fever. A combination of historical risk, physical exam, and laboratory and ultrasound findings will often be needed to confirm the diagnosis.

Fever is never to be expected in an uncomplicated miscarriage. Vaginal bleeding and some cramping are common after miscarriage; women will bleed, on average, between eight and 11 days afterward.5 Women who fall outside the normal range and experience prolonged bleeding, heavy bleeding, or severe abdominal pain should be evaluated.

A workup for patients with a possible septic abortion should include a complete blood count, blood culture with additional laboratory investigation if there is concern for bacteremia/sepsis, and type and screen for Rh factor and for possible blood transfusion, if needed.

All patients with postabortion complications should be screened for Rh factor; Rho(D) immune globulin (RhoGAM) should be administered if results indicate that the patient is Rh-negative and unsensitized. A quantitative β-hCG level can be obtained to confirm pregnancy. A single measurement will not be helpful; β-hCG can remain positive for weeks after an uncomplicated miscarriage. On the other hand, a low level does not exclude RPOC—the RPOC, if necrotic, may remain in the uterus without secreting hormone. The trend of β-hCG over time can be helpful if the diagnosis is unclear.

A careful physical exam, including a pelvic exam, should be performed. Assess for uterine tenderness, peritoneal signs, and purulent discharge from the cervix. An open cervical os is suggestive of RPOC, as the cervix closes quickly after a complete miscarriage, but a closed cervical os does not exclude the possibility of RPOC or septic abortion. The amount of bleeding should be noted, along with any tissue or clots within the vaginal vault or cervix.

A pelvic ultrasound should be obtained in all patients concerning for a septic incomplete miscarriage. Ultrasound findings can be nonspecific, because small amounts of retained tissue can look like blood (a common finding after miscarriage). Ultrasound findings of heterogeneous, echogenic material within the uterus or a thick, irregular endometrium support a diagnosis of RPOC in patients considered at risk.8,9 Increased color Doppler flow is often seen with RPOC, but there may be decreased flow in the case of necrotic RPOC. Ultrasound findings consistent with RPOC in a febrile, ill patient suggest a septic abortion.

Continue for treatment and prognosis >>

 

 

Treatment and prognosisPatients with a septic abortion require immediate evacuation of the uterus to prevent deadly complications; antibiotics may not be able to perfuse to the necrotic source of infection.10 Suction curettage is less likely than sharp curettage to cause perforation.

Broad-spectrum antibiotics should be administered. The bacteria associated with a septic incomplete miscarriage are usually polymicrobial and represent the normal flora of the vagina and cervix. The choice of agents recommended is usually the same as for pelvic inflammatory disease.11

The treatment regimen typically includes clin­damycin (900 mg IV q8h), plus gentamicin (5 mg/kg IV once a day), with or without ampicillin (2 g IV q4h).11,12 Alternatively, a combination of ampicillin, gentamicin, and metronidazole (500 mg IV q8h) can be used.

Further surgery, including laparotomy and possible hysterectomy, is indicated in patients who do not respond to uterine evacuation and parenteral antibiotics. Other possible complications requiring surgery include pelvic abscess, necrotizing Clostridium infections in the myometrium, and uterine perforation.

OUTCOME FOR THE CASE PATIENTThe patient was started on IV ampicillin, gentamicin, and clindamycin and taken promptly for a suction dilation and curettage. Pathology later showed a gestational sac with severe acute necrotizing chorioamnionitis and extensive bacterial growth. This confirmed the diagnosis of a septic, incomplete miscarriage.

Blood cultures remained without any growth, and the patient was afebrile on the second postop day. The WBC count and β-hCG level trended downward.

The patient was discharged on a 14-day course of oral doxycycline and metronidazole. She was then lost to further follow-up.

CONCLUSIONThe differential diagnosis in this ill, febrile patient was initially very broad. The importance of suspecting pregnancy in all women of childbearing age, especially those not using contraception, cannot be underestimated. The accuracy of patient history and recall of last menstrual period in determining the possibility of pregnancy is not sufficiently reliable.

References

1. Promislow JH, Baird DD, Wilcox AJ, et al. Bleeding following pregnancy loss prior to six weeks gestation. Hum Reprod. 2007;22(3):853-857.
2. Ramoska EA, Sacchetti AD, Nepp M. Reliability of patient history in determining the possibility of pregnancy. Ann Emerg Med. 1989;18(1):48-50.
3. Osazuwa H, Aziken M. Septic abortion: a review of social and demographic characteristics. Arch Gynecol Obstet. 2007;275(2):117-119.
4. Hure AJ, Powers JR, Mishra GD, et al. Miscarriage, preterm delivery, and stillbirth: large variations in rates within a cohort of Australian women. PLoS One. 2012;7(5):e37109.
5. Nielsen S, Hahlin M. Expectant management of first-trimester spontaneous abortion. Lancet. 1995;345(8942):84-86.
6. Eschenbach DA. Treating spontaneous and induced septic abortions. Obstet Gynecol. 2015;125(5):1042-1048.
7. Rana A, Pradhan N, Gurung G, Singh M. Induced septic abortion: a major factor in maternal mortality and morbidity. J Obstet Gynaecol Res. 2004;30(1):3-8.
8. Abbasi S, Jamal A, Eslamian L, Marsousi V. Role of clinical and ultrasound findings in the diagnosis of retained products of conception. Ultrasound Obstet Gynecol. 2008;32(5):704-707.
9. Esmaeillou H, Jamal A, Eslamian L, et al. Accurate detection of retained products of conception after first- and second-trimester abortion by color doppler sonography. J Med Ultrasound. 2015;23(7):34-38.
10. Finkielman JD, De Feo FD, Heller PG, Afessa B. The clinical course of patients with septic abortion admitted to an intensive care unit. Intensive Care Med. 2004;30(6):1097-1102.
11. CDC. Sexually transmitted diseases treatment guidelines, 2010. MMWR Recomm Rep. 2010;59(RR-12):1-110.
12. Mackeen AD, Packard RE, Ota E, Speer L. Antibiotic regimens for postpartum endometritis. Cochrane Database Syst Rev. 2015;2:CD001067.

Article PDF
Author and Disclosure Information

Sara Lolar, MS, PA-C

Sara Lolar is a PA at Detroit Receiving Hospital in Michigan and faculty in the Department of Physician Assistant Studies at Wayne State University in Detroit.

The author has no financial relationships to disclose.

Issue
Clinician Reviews - 26(9)
Publications
Topics
Page Number
30-33
Sections
Author and Disclosure Information

Sara Lolar, MS, PA-C

Sara Lolar is a PA at Detroit Receiving Hospital in Michigan and faculty in the Department of Physician Assistant Studies at Wayne State University in Detroit.

The author has no financial relationships to disclose.

Author and Disclosure Information

Sara Lolar, MS, PA-C

Sara Lolar is a PA at Detroit Receiving Hospital in Michigan and faculty in the Department of Physician Assistant Studies at Wayne State University in Detroit.

The author has no financial relationships to disclose.

Article PDF
Article PDF

IN THIS ARTICLE

  • Clinical presentation and evaluation
  • Terminology table
  • Outcome for the case patient

A 36-year-old Bengali woman with a history of well-controlled diabetes presents to the emergency department with complaints of feeling “unwell” for about two weeks. She does not speak English, and a hospital-provided phone translator is used to obtain history and explain hospital course. The patient is vague regarding symptomatology, describing general malaise and tiredness. She says she became “much worse” two days ago and has shaking chills, sore throat, headache, and nonproductive cough, but she denies shortness of breath or chest pain. She also developed nausea and vomiting, stating, “I can’t keep anything down.”

She has not recently traveled out of the country and has no known sick contacts. Influenza activity is high in the area, and the patient has not received immunization. She had a “normal” menstrual period two weeks ago and firmly states, “There is no way I can be pregnant.” She admits to vaginal “spotting” off and on for the past two weeks without abdominal pain. She is married with six children and has no history of miscarriage, ectopic pregnancy, or induced abortion; she is not taking any form of birth control.

On exam, the patient is tachycardic, with a heart rate of 127 beats/min, and has a fever of 103.3°F. Blood pressure, respiratory rate, and pulse oximetry are normal. She appears unwell and dehydrated. Her mucous membranes are dry, but no skin rash is noted. There is no tonsillar swelling or exudate and no meningismus; the lung exam is clear, with no adventitious sounds. Abdominal exam demonstrates mild, generalized tenderness in the lower abdomen without peritoneal signs. No costovertebral angle tenderness is noted. Initial diagnostic considerations include sources of fever (eg, influenza, pneumonia, urinary tract infection, viral illness), or abdominal sources, such as appendicitis.

An upright anteroposterior chest x-ray shows no infiltrate, pleural effusion, or cardiomegaly. Laboratory results include a high white blood cell (WBC) count (16.9 k/mm3) with bandemia and normal electrolytes without anion gap. Rapid influenza A and B testing is negative. A urine pregnancy test is positive, and the urinalysis shows no infection but +2 ketones. Rh factor is positive. A serum quantitative β-hCG is 130,581 mIU/mL. Blood cultures are obtained, but results are not available.

Due to cultural differences, the patient is very reluctant to consent to a pelvic exam. After extensive counseling, she agrees to a bimanual exam only. The uterus is boggy and enlarged to about 12 weeks. There is exquisite uterine tenderness and purulent discharge on the gloved finger. The cervical os is closed, and there is scant bleeding.

A transvaginal ultrasound is obtained; it reveals a thickened endometrium with echogenicity, without increased vascularity, and no identifiable intrauterine pregnancy. The adnexa have no masses, and there is no free fluid in the endometrium (see Figures 1 and 2).

The patient is given broad-spectrum antibiotics and urgently transported to the operating room by Ob-Gyn for uterine evacuation. She is found to have a septic abortion due to retained products of conception (RPOC) from an incomplete miscarriage.

Continue for discussion >>

 

 

DISCUSSIONIt is not uncommon for a woman to miscarry a very early pregnancy and not realize she had been pregnant.1 Many attribute it to a “heavy” or unusual period. In one study, 11% of patients who denied the possibility of pregnancy were, in fact, pregnant.2

Miscarriage is a frequent outcome of early pregnancy; it is estimated that 11% to 20% of early pregnancies result in a spontaneous miscarriage.3-5 Most resolve without complications, but risk increases with gestational age. When they do occur, complications include RPOC, heavy prolonged bleeding, and endometritis. RPOC refers to placental or fetal tissue that remains in the uterus after a miscarriage, surgical abortion, or preterm/term delivery (see Table for additional terminology related to miscarriage and abortion). Because of increased morbidity, it is important to suspect RPOC after a known miscarriage or an induced abortion, or in a pregnant patient with bleeding.

Incidence and pathophysiologySeptic abortion is a relatively rare complication of miscarriage. It can refer to a spontaneous miscarriage complicated by a subsequent intrauterine infection, often caused by RPOC. Septic abortion is much more common after an induced abortion, in which there is instrumentation of the uterus.

The infection after a spontaneous miscarriage usually begins as endometritis. It involves the necrotic RPOC, which are prone to infection by the cervical and vaginal flora. It may spread further into the parametrium/myometrium and the peritoneal cavity. The infection may then progress to bacteremia and sepsis. Typical causative organisms include Escherichia coli, Enterobacter aerogenes, Proteus vulgaris, hemolytic streptococci, staphylococci, and some anaerobic organisms, including Clostridium perfringens.3

Death, although rare in developed countries, is usually secondary to the sequela of sepsis, including septic shock, renal failure, adult respiratory distress syndrome, and disseminated intravascular coagulation.3,6,7 Pelvic adhesions and hysterectomy are also possible outcomes of a septic abortion.

Continue for clinical presentation and evaluation >>

 

 

Clinical presentation and evaluationMany findings suggestive of septic abortion are nonspecific, such as bleeding, pain, uterine tenderness, and fever. A combination of historical risk, physical exam, and laboratory and ultrasound findings will often be needed to confirm the diagnosis.

Fever is never to be expected in an uncomplicated miscarriage. Vaginal bleeding and some cramping are common after miscarriage; women will bleed, on average, between eight and 11 days afterward.5 Women who fall outside the normal range and experience prolonged bleeding, heavy bleeding, or severe abdominal pain should be evaluated.

A workup for patients with a possible septic abortion should include a complete blood count, blood culture with additional laboratory investigation if there is concern for bacteremia/sepsis, and type and screen for Rh factor and for possible blood transfusion, if needed.

All patients with postabortion complications should be screened for Rh factor; Rho(D) immune globulin (RhoGAM) should be administered if results indicate that the patient is Rh-negative and unsensitized. A quantitative β-hCG level can be obtained to confirm pregnancy. A single measurement will not be helpful; β-hCG can remain positive for weeks after an uncomplicated miscarriage. On the other hand, a low level does not exclude RPOC—the RPOC, if necrotic, may remain in the uterus without secreting hormone. The trend of β-hCG over time can be helpful if the diagnosis is unclear.

A careful physical exam, including a pelvic exam, should be performed. Assess for uterine tenderness, peritoneal signs, and purulent discharge from the cervix. An open cervical os is suggestive of RPOC, as the cervix closes quickly after a complete miscarriage, but a closed cervical os does not exclude the possibility of RPOC or septic abortion. The amount of bleeding should be noted, along with any tissue or clots within the vaginal vault or cervix.

A pelvic ultrasound should be obtained in all patients concerning for a septic incomplete miscarriage. Ultrasound findings can be nonspecific, because small amounts of retained tissue can look like blood (a common finding after miscarriage). Ultrasound findings of heterogeneous, echogenic material within the uterus or a thick, irregular endometrium support a diagnosis of RPOC in patients considered at risk.8,9 Increased color Doppler flow is often seen with RPOC, but there may be decreased flow in the case of necrotic RPOC. Ultrasound findings consistent with RPOC in a febrile, ill patient suggest a septic abortion.

Continue for treatment and prognosis >>

 

 

Treatment and prognosisPatients with a septic abortion require immediate evacuation of the uterus to prevent deadly complications; antibiotics may not be able to perfuse to the necrotic source of infection.10 Suction curettage is less likely than sharp curettage to cause perforation.

Broad-spectrum antibiotics should be administered. The bacteria associated with a septic incomplete miscarriage are usually polymicrobial and represent the normal flora of the vagina and cervix. The choice of agents recommended is usually the same as for pelvic inflammatory disease.11

The treatment regimen typically includes clin­damycin (900 mg IV q8h), plus gentamicin (5 mg/kg IV once a day), with or without ampicillin (2 g IV q4h).11,12 Alternatively, a combination of ampicillin, gentamicin, and metronidazole (500 mg IV q8h) can be used.

Further surgery, including laparotomy and possible hysterectomy, is indicated in patients who do not respond to uterine evacuation and parenteral antibiotics. Other possible complications requiring surgery include pelvic abscess, necrotizing Clostridium infections in the myometrium, and uterine perforation.

OUTCOME FOR THE CASE PATIENTThe patient was started on IV ampicillin, gentamicin, and clindamycin and taken promptly for a suction dilation and curettage. Pathology later showed a gestational sac with severe acute necrotizing chorioamnionitis and extensive bacterial growth. This confirmed the diagnosis of a septic, incomplete miscarriage.

Blood cultures remained without any growth, and the patient was afebrile on the second postop day. The WBC count and β-hCG level trended downward.

The patient was discharged on a 14-day course of oral doxycycline and metronidazole. She was then lost to further follow-up.

CONCLUSIONThe differential diagnosis in this ill, febrile patient was initially very broad. The importance of suspecting pregnancy in all women of childbearing age, especially those not using contraception, cannot be underestimated. The accuracy of patient history and recall of last menstrual period in determining the possibility of pregnancy is not sufficiently reliable.

IN THIS ARTICLE

  • Clinical presentation and evaluation
  • Terminology table
  • Outcome for the case patient

A 36-year-old Bengali woman with a history of well-controlled diabetes presents to the emergency department with complaints of feeling “unwell” for about two weeks. She does not speak English, and a hospital-provided phone translator is used to obtain history and explain hospital course. The patient is vague regarding symptomatology, describing general malaise and tiredness. She says she became “much worse” two days ago and has shaking chills, sore throat, headache, and nonproductive cough, but she denies shortness of breath or chest pain. She also developed nausea and vomiting, stating, “I can’t keep anything down.”

She has not recently traveled out of the country and has no known sick contacts. Influenza activity is high in the area, and the patient has not received immunization. She had a “normal” menstrual period two weeks ago and firmly states, “There is no way I can be pregnant.” She admits to vaginal “spotting” off and on for the past two weeks without abdominal pain. She is married with six children and has no history of miscarriage, ectopic pregnancy, or induced abortion; she is not taking any form of birth control.

On exam, the patient is tachycardic, with a heart rate of 127 beats/min, and has a fever of 103.3°F. Blood pressure, respiratory rate, and pulse oximetry are normal. She appears unwell and dehydrated. Her mucous membranes are dry, but no skin rash is noted. There is no tonsillar swelling or exudate and no meningismus; the lung exam is clear, with no adventitious sounds. Abdominal exam demonstrates mild, generalized tenderness in the lower abdomen without peritoneal signs. No costovertebral angle tenderness is noted. Initial diagnostic considerations include sources of fever (eg, influenza, pneumonia, urinary tract infection, viral illness), or abdominal sources, such as appendicitis.

An upright anteroposterior chest x-ray shows no infiltrate, pleural effusion, or cardiomegaly. Laboratory results include a high white blood cell (WBC) count (16.9 k/mm3) with bandemia and normal electrolytes without anion gap. Rapid influenza A and B testing is negative. A urine pregnancy test is positive, and the urinalysis shows no infection but +2 ketones. Rh factor is positive. A serum quantitative β-hCG is 130,581 mIU/mL. Blood cultures are obtained, but results are not available.

Due to cultural differences, the patient is very reluctant to consent to a pelvic exam. After extensive counseling, she agrees to a bimanual exam only. The uterus is boggy and enlarged to about 12 weeks. There is exquisite uterine tenderness and purulent discharge on the gloved finger. The cervical os is closed, and there is scant bleeding.

A transvaginal ultrasound is obtained; it reveals a thickened endometrium with echogenicity, without increased vascularity, and no identifiable intrauterine pregnancy. The adnexa have no masses, and there is no free fluid in the endometrium (see Figures 1 and 2).

The patient is given broad-spectrum antibiotics and urgently transported to the operating room by Ob-Gyn for uterine evacuation. She is found to have a septic abortion due to retained products of conception (RPOC) from an incomplete miscarriage.

Continue for discussion >>

 

 

DISCUSSIONIt is not uncommon for a woman to miscarry a very early pregnancy and not realize she had been pregnant.1 Many attribute it to a “heavy” or unusual period. In one study, 11% of patients who denied the possibility of pregnancy were, in fact, pregnant.2

Miscarriage is a frequent outcome of early pregnancy; it is estimated that 11% to 20% of early pregnancies result in a spontaneous miscarriage.3-5 Most resolve without complications, but risk increases with gestational age. When they do occur, complications include RPOC, heavy prolonged bleeding, and endometritis. RPOC refers to placental or fetal tissue that remains in the uterus after a miscarriage, surgical abortion, or preterm/term delivery (see Table for additional terminology related to miscarriage and abortion). Because of increased morbidity, it is important to suspect RPOC after a known miscarriage or an induced abortion, or in a pregnant patient with bleeding.

Incidence and pathophysiologySeptic abortion is a relatively rare complication of miscarriage. It can refer to a spontaneous miscarriage complicated by a subsequent intrauterine infection, often caused by RPOC. Septic abortion is much more common after an induced abortion, in which there is instrumentation of the uterus.

The infection after a spontaneous miscarriage usually begins as endometritis. It involves the necrotic RPOC, which are prone to infection by the cervical and vaginal flora. It may spread further into the parametrium/myometrium and the peritoneal cavity. The infection may then progress to bacteremia and sepsis. Typical causative organisms include Escherichia coli, Enterobacter aerogenes, Proteus vulgaris, hemolytic streptococci, staphylococci, and some anaerobic organisms, including Clostridium perfringens.3

Death, although rare in developed countries, is usually secondary to the sequela of sepsis, including septic shock, renal failure, adult respiratory distress syndrome, and disseminated intravascular coagulation.3,6,7 Pelvic adhesions and hysterectomy are also possible outcomes of a septic abortion.

Continue for clinical presentation and evaluation >>

 

 

Clinical presentation and evaluationMany findings suggestive of septic abortion are nonspecific, such as bleeding, pain, uterine tenderness, and fever. A combination of historical risk, physical exam, and laboratory and ultrasound findings will often be needed to confirm the diagnosis.

Fever is never to be expected in an uncomplicated miscarriage. Vaginal bleeding and some cramping are common after miscarriage; women will bleed, on average, between eight and 11 days afterward.5 Women who fall outside the normal range and experience prolonged bleeding, heavy bleeding, or severe abdominal pain should be evaluated.

A workup for patients with a possible septic abortion should include a complete blood count, blood culture with additional laboratory investigation if there is concern for bacteremia/sepsis, and type and screen for Rh factor and for possible blood transfusion, if needed.

All patients with postabortion complications should be screened for Rh factor; Rho(D) immune globulin (RhoGAM) should be administered if results indicate that the patient is Rh-negative and unsensitized. A quantitative β-hCG level can be obtained to confirm pregnancy. A single measurement will not be helpful; β-hCG can remain positive for weeks after an uncomplicated miscarriage. On the other hand, a low level does not exclude RPOC—the RPOC, if necrotic, may remain in the uterus without secreting hormone. The trend of β-hCG over time can be helpful if the diagnosis is unclear.

A careful physical exam, including a pelvic exam, should be performed. Assess for uterine tenderness, peritoneal signs, and purulent discharge from the cervix. An open cervical os is suggestive of RPOC, as the cervix closes quickly after a complete miscarriage, but a closed cervical os does not exclude the possibility of RPOC or septic abortion. The amount of bleeding should be noted, along with any tissue or clots within the vaginal vault or cervix.

A pelvic ultrasound should be obtained in all patients concerning for a septic incomplete miscarriage. Ultrasound findings can be nonspecific, because small amounts of retained tissue can look like blood (a common finding after miscarriage). Ultrasound findings of heterogeneous, echogenic material within the uterus or a thick, irregular endometrium support a diagnosis of RPOC in patients considered at risk.8,9 Increased color Doppler flow is often seen with RPOC, but there may be decreased flow in the case of necrotic RPOC. Ultrasound findings consistent with RPOC in a febrile, ill patient suggest a septic abortion.

Continue for treatment and prognosis >>

 

 

Treatment and prognosisPatients with a septic abortion require immediate evacuation of the uterus to prevent deadly complications; antibiotics may not be able to perfuse to the necrotic source of infection.10 Suction curettage is less likely than sharp curettage to cause perforation.

Broad-spectrum antibiotics should be administered. The bacteria associated with a septic incomplete miscarriage are usually polymicrobial and represent the normal flora of the vagina and cervix. The choice of agents recommended is usually the same as for pelvic inflammatory disease.11

The treatment regimen typically includes clin­damycin (900 mg IV q8h), plus gentamicin (5 mg/kg IV once a day), with or without ampicillin (2 g IV q4h).11,12 Alternatively, a combination of ampicillin, gentamicin, and metronidazole (500 mg IV q8h) can be used.

Further surgery, including laparotomy and possible hysterectomy, is indicated in patients who do not respond to uterine evacuation and parenteral antibiotics. Other possible complications requiring surgery include pelvic abscess, necrotizing Clostridium infections in the myometrium, and uterine perforation.

OUTCOME FOR THE CASE PATIENTThe patient was started on IV ampicillin, gentamicin, and clindamycin and taken promptly for a suction dilation and curettage. Pathology later showed a gestational sac with severe acute necrotizing chorioamnionitis and extensive bacterial growth. This confirmed the diagnosis of a septic, incomplete miscarriage.

Blood cultures remained without any growth, and the patient was afebrile on the second postop day. The WBC count and β-hCG level trended downward.

The patient was discharged on a 14-day course of oral doxycycline and metronidazole. She was then lost to further follow-up.

CONCLUSIONThe differential diagnosis in this ill, febrile patient was initially very broad. The importance of suspecting pregnancy in all women of childbearing age, especially those not using contraception, cannot be underestimated. The accuracy of patient history and recall of last menstrual period in determining the possibility of pregnancy is not sufficiently reliable.

References

1. Promislow JH, Baird DD, Wilcox AJ, et al. Bleeding following pregnancy loss prior to six weeks gestation. Hum Reprod. 2007;22(3):853-857.
2. Ramoska EA, Sacchetti AD, Nepp M. Reliability of patient history in determining the possibility of pregnancy. Ann Emerg Med. 1989;18(1):48-50.
3. Osazuwa H, Aziken M. Septic abortion: a review of social and demographic characteristics. Arch Gynecol Obstet. 2007;275(2):117-119.
4. Hure AJ, Powers JR, Mishra GD, et al. Miscarriage, preterm delivery, and stillbirth: large variations in rates within a cohort of Australian women. PLoS One. 2012;7(5):e37109.
5. Nielsen S, Hahlin M. Expectant management of first-trimester spontaneous abortion. Lancet. 1995;345(8942):84-86.
6. Eschenbach DA. Treating spontaneous and induced septic abortions. Obstet Gynecol. 2015;125(5):1042-1048.
7. Rana A, Pradhan N, Gurung G, Singh M. Induced septic abortion: a major factor in maternal mortality and morbidity. J Obstet Gynaecol Res. 2004;30(1):3-8.
8. Abbasi S, Jamal A, Eslamian L, Marsousi V. Role of clinical and ultrasound findings in the diagnosis of retained products of conception. Ultrasound Obstet Gynecol. 2008;32(5):704-707.
9. Esmaeillou H, Jamal A, Eslamian L, et al. Accurate detection of retained products of conception after first- and second-trimester abortion by color doppler sonography. J Med Ultrasound. 2015;23(7):34-38.
10. Finkielman JD, De Feo FD, Heller PG, Afessa B. The clinical course of patients with septic abortion admitted to an intensive care unit. Intensive Care Med. 2004;30(6):1097-1102.
11. CDC. Sexually transmitted diseases treatment guidelines, 2010. MMWR Recomm Rep. 2010;59(RR-12):1-110.
12. Mackeen AD, Packard RE, Ota E, Speer L. Antibiotic regimens for postpartum endometritis. Cochrane Database Syst Rev. 2015;2:CD001067.

References

1. Promislow JH, Baird DD, Wilcox AJ, et al. Bleeding following pregnancy loss prior to six weeks gestation. Hum Reprod. 2007;22(3):853-857.
2. Ramoska EA, Sacchetti AD, Nepp M. Reliability of patient history in determining the possibility of pregnancy. Ann Emerg Med. 1989;18(1):48-50.
3. Osazuwa H, Aziken M. Septic abortion: a review of social and demographic characteristics. Arch Gynecol Obstet. 2007;275(2):117-119.
4. Hure AJ, Powers JR, Mishra GD, et al. Miscarriage, preterm delivery, and stillbirth: large variations in rates within a cohort of Australian women. PLoS One. 2012;7(5):e37109.
5. Nielsen S, Hahlin M. Expectant management of first-trimester spontaneous abortion. Lancet. 1995;345(8942):84-86.
6. Eschenbach DA. Treating spontaneous and induced septic abortions. Obstet Gynecol. 2015;125(5):1042-1048.
7. Rana A, Pradhan N, Gurung G, Singh M. Induced septic abortion: a major factor in maternal mortality and morbidity. J Obstet Gynaecol Res. 2004;30(1):3-8.
8. Abbasi S, Jamal A, Eslamian L, Marsousi V. Role of clinical and ultrasound findings in the diagnosis of retained products of conception. Ultrasound Obstet Gynecol. 2008;32(5):704-707.
9. Esmaeillou H, Jamal A, Eslamian L, et al. Accurate detection of retained products of conception after first- and second-trimester abortion by color doppler sonography. J Med Ultrasound. 2015;23(7):34-38.
10. Finkielman JD, De Feo FD, Heller PG, Afessa B. The clinical course of patients with septic abortion admitted to an intensive care unit. Intensive Care Med. 2004;30(6):1097-1102.
11. CDC. Sexually transmitted diseases treatment guidelines, 2010. MMWR Recomm Rep. 2010;59(RR-12):1-110.
12. Mackeen AD, Packard RE, Ota E, Speer L. Antibiotic regimens for postpartum endometritis. Cochrane Database Syst Rev. 2015;2:CD001067.

Issue
Clinician Reviews - 26(9)
Issue
Clinician Reviews - 26(9)
Page Number
30-33
Page Number
30-33
Publications
Publications
Topics
Article Type
Sections
Disallow All Ads
Alternative CME
Article PDF Media

Neuroendocrine Tumor Research Foundation Posts RFPs for $1.2 Million in Grants

Article Type
Changed
Fri, 01/18/2019 - 08:37
Display Headline
Neuroendocrine Tumor Research Foundation Posts RFPs for $1.2 Million in Grants
Publications
Topics
Sections
Publications
Publications
Topics
Article Type
Display Headline
Neuroendocrine Tumor Research Foundation Posts RFPs for $1.2 Million in Grants
Display Headline
Neuroendocrine Tumor Research Foundation Posts RFPs for $1.2 Million in Grants
Sections
Disallow All Ads

Research Training Fellowship in Ataxia is Available

Article Type
Changed
Fri, 01/18/2019 - 08:37
Display Headline
Research Training Fellowship in Ataxia is Available
Publications
Topics
Sections
Publications
Publications
Topics
Article Type
Display Headline
Research Training Fellowship in Ataxia is Available
Display Headline
Research Training Fellowship in Ataxia is Available
Sections
Disallow All Ads

Do autologous blood and PRP injections effectively treat tennis elbow?

Article Type
Changed
Fri, 01/18/2019 - 08:36
Display Headline
Do autologous blood and PRP injections effectively treat tennis elbow?

EVIDENCE-BASED ANSWER:

Yes, both approaches reduce pain, but the improvement with platelet-rich plasma (PRP) is not clinically meaningful. Autologous blood injections (ABIs) are more effective than corticosteroid injections for reducing pain and disability in patients with tennis elbow in both the short and long term (strength of recommendation [SOR]: B, consistent findings in 2 randomized controlled trials [RCTs]).

PRP injections reduce pain more than sham injections for chronic tennis elbow (SOR: B, high-quality RCT). However, the magnitude of the difference is small.

Autologous blood injections reduce pain

A 2013 RCT assessed the effectiveness of ABI (2 mL venous blood and 1 mL 2% lidocaine) compared with injection of 40 mg methylprednisolone and 1 mL 2% lidocaine in 50 patients with tennis elbow (mean age 38.2 years, mean duration of symptoms 4.5 weeks).1 The degree of pain and disability were evaluated at baseline, 2 weeks, and 6 weeks using a visual analog pain scale (VAS) and Nirschl functional staging, respectively, both measured on 10-point scales. Researchers found no statistical difference between the groups at baseline or 2 weeks. At 6 weeks, however, the ABI group showed significant improvements over the steroid group in pain (mean VAS=1.52 vs 2.28; P=.0396) and disability (mean Nirschl stage=1.40 vs 2.40; P=.0045).

 

A previous RCT, in 2012, compared ABI (2 mL of venous blood and 1 mL of 0.5% bupivacaine) in 30 patients (mean age 42.9 years, mean duration of symptoms 9.5 weeks) with a corticosteroid injection (80 mg of methylprednisolone and 1 mL of 0.5% bupivacaine) in another 30 patients (mean age 42.2 years, mean duration of symptoms 7.7 weeks). Outcomes were assessed at 12 weeks and 6 months on a 10-point VAS and 7-point Nirschl stage.2

The ABI group showed a significant decrease in pain and disability compared with the steroid group (mean VAS at 12 weeks=0.6 vs 1.5, P=.0127; mean VAS at 6 months=0.5 vs 1.8; P=.0058; mean Nirschl stage at 12 weeks=0.43 vs 1.0; P=.0184; mean Nirschl stage at 6 months=0.36 vs 1.2; P=.0064).

 

 

PRP: Some efficacy, little significance

A 2014 double-blinded RCT analyzed the efficacy of PRP injection vs control injection for treating tennis elbow of at least 3 months’ duration.3 A total of 112 patients (mean age 48.4 years) received a 2- to 3-mL injection of PRP at a site blocked with bupivacaine; 113 patients (mean age 47.4 years) received an injection of 2 to 3 mL 0.5% bupivacaine only. Success was defined as a ≥25% improvement in pain score on a 100-point VAS.

At 24 weeks, the PRP group demonstrated a success rate of 83.9% compared with 68.3% in the control group (number needed to treat=6; P=.037). However, the difference between the mean VAS improvement of 38 points in the PRP group and the mean decrease of 36 points in the control group carries little clinical significance.

References

1. Jindal N, Gaury Y, Banshiwal RC, et al. Comparison of short term results of single injection of autologous blood and steroid injection in tennis elbow: a prospective study. J Orthop Surg Res. 2013;8:10.

2. Dojode C. A randomised control trial to evaluate the efficacy of autologous blood injection versus local corticosteroid injection for treatment of lateral epicondylitis. Bone Joint Res. 2012;1:192-197.

3. Mishra A, Skrepnik NV, Edwards SG, et al. Efficacy of platelet-rich plasma for chronic tennis elbow: a double-blind, prospective, multicenter, randomized controlled trial of 230 patients. Am J Sports Med. 2014;42:463-471.

Article PDF
Author and Disclosure Information

Luke Widstrom, DO
Andrew Slattengren, DO

University of Minnesota-North Memorial Family Medicine Residency, Minneapolis

EDITOR
Anne Mounsey, MD

University of North Carolina, Department of Family Medicine, Chapel Hill

Issue
The Journal of Family Practice - 65(9)
Publications
Topics
Page Number
635
Sections
Author and Disclosure Information

Luke Widstrom, DO
Andrew Slattengren, DO

University of Minnesota-North Memorial Family Medicine Residency, Minneapolis

EDITOR
Anne Mounsey, MD

University of North Carolina, Department of Family Medicine, Chapel Hill

Author and Disclosure Information

Luke Widstrom, DO
Andrew Slattengren, DO

University of Minnesota-North Memorial Family Medicine Residency, Minneapolis

EDITOR
Anne Mounsey, MD

University of North Carolina, Department of Family Medicine, Chapel Hill

Article PDF
Article PDF

EVIDENCE-BASED ANSWER:

Yes, both approaches reduce pain, but the improvement with platelet-rich plasma (PRP) is not clinically meaningful. Autologous blood injections (ABIs) are more effective than corticosteroid injections for reducing pain and disability in patients with tennis elbow in both the short and long term (strength of recommendation [SOR]: B, consistent findings in 2 randomized controlled trials [RCTs]).

PRP injections reduce pain more than sham injections for chronic tennis elbow (SOR: B, high-quality RCT). However, the magnitude of the difference is small.

Autologous blood injections reduce pain

A 2013 RCT assessed the effectiveness of ABI (2 mL venous blood and 1 mL 2% lidocaine) compared with injection of 40 mg methylprednisolone and 1 mL 2% lidocaine in 50 patients with tennis elbow (mean age 38.2 years, mean duration of symptoms 4.5 weeks).1 The degree of pain and disability were evaluated at baseline, 2 weeks, and 6 weeks using a visual analog pain scale (VAS) and Nirschl functional staging, respectively, both measured on 10-point scales. Researchers found no statistical difference between the groups at baseline or 2 weeks. At 6 weeks, however, the ABI group showed significant improvements over the steroid group in pain (mean VAS=1.52 vs 2.28; P=.0396) and disability (mean Nirschl stage=1.40 vs 2.40; P=.0045).

 

A previous RCT, in 2012, compared ABI (2 mL of venous blood and 1 mL of 0.5% bupivacaine) in 30 patients (mean age 42.9 years, mean duration of symptoms 9.5 weeks) with a corticosteroid injection (80 mg of methylprednisolone and 1 mL of 0.5% bupivacaine) in another 30 patients (mean age 42.2 years, mean duration of symptoms 7.7 weeks). Outcomes were assessed at 12 weeks and 6 months on a 10-point VAS and 7-point Nirschl stage.2

The ABI group showed a significant decrease in pain and disability compared with the steroid group (mean VAS at 12 weeks=0.6 vs 1.5, P=.0127; mean VAS at 6 months=0.5 vs 1.8; P=.0058; mean Nirschl stage at 12 weeks=0.43 vs 1.0; P=.0184; mean Nirschl stage at 6 months=0.36 vs 1.2; P=.0064).

 

 

PRP: Some efficacy, little significance

A 2014 double-blinded RCT analyzed the efficacy of PRP injection vs control injection for treating tennis elbow of at least 3 months’ duration.3 A total of 112 patients (mean age 48.4 years) received a 2- to 3-mL injection of PRP at a site blocked with bupivacaine; 113 patients (mean age 47.4 years) received an injection of 2 to 3 mL 0.5% bupivacaine only. Success was defined as a ≥25% improvement in pain score on a 100-point VAS.

At 24 weeks, the PRP group demonstrated a success rate of 83.9% compared with 68.3% in the control group (number needed to treat=6; P=.037). However, the difference between the mean VAS improvement of 38 points in the PRP group and the mean decrease of 36 points in the control group carries little clinical significance.

EVIDENCE-BASED ANSWER:

Yes, both approaches reduce pain, but the improvement with platelet-rich plasma (PRP) is not clinically meaningful. Autologous blood injections (ABIs) are more effective than corticosteroid injections for reducing pain and disability in patients with tennis elbow in both the short and long term (strength of recommendation [SOR]: B, consistent findings in 2 randomized controlled trials [RCTs]).

PRP injections reduce pain more than sham injections for chronic tennis elbow (SOR: B, high-quality RCT). However, the magnitude of the difference is small.

Autologous blood injections reduce pain

A 2013 RCT assessed the effectiveness of ABI (2 mL venous blood and 1 mL 2% lidocaine) compared with injection of 40 mg methylprednisolone and 1 mL 2% lidocaine in 50 patients with tennis elbow (mean age 38.2 years, mean duration of symptoms 4.5 weeks).1 The degree of pain and disability were evaluated at baseline, 2 weeks, and 6 weeks using a visual analog pain scale (VAS) and Nirschl functional staging, respectively, both measured on 10-point scales. Researchers found no statistical difference between the groups at baseline or 2 weeks. At 6 weeks, however, the ABI group showed significant improvements over the steroid group in pain (mean VAS=1.52 vs 2.28; P=.0396) and disability (mean Nirschl stage=1.40 vs 2.40; P=.0045).

 

A previous RCT, in 2012, compared ABI (2 mL of venous blood and 1 mL of 0.5% bupivacaine) in 30 patients (mean age 42.9 years, mean duration of symptoms 9.5 weeks) with a corticosteroid injection (80 mg of methylprednisolone and 1 mL of 0.5% bupivacaine) in another 30 patients (mean age 42.2 years, mean duration of symptoms 7.7 weeks). Outcomes were assessed at 12 weeks and 6 months on a 10-point VAS and 7-point Nirschl stage.2

The ABI group showed a significant decrease in pain and disability compared with the steroid group (mean VAS at 12 weeks=0.6 vs 1.5, P=.0127; mean VAS at 6 months=0.5 vs 1.8; P=.0058; mean Nirschl stage at 12 weeks=0.43 vs 1.0; P=.0184; mean Nirschl stage at 6 months=0.36 vs 1.2; P=.0064).

 

 

PRP: Some efficacy, little significance

A 2014 double-blinded RCT analyzed the efficacy of PRP injection vs control injection for treating tennis elbow of at least 3 months’ duration.3 A total of 112 patients (mean age 48.4 years) received a 2- to 3-mL injection of PRP at a site blocked with bupivacaine; 113 patients (mean age 47.4 years) received an injection of 2 to 3 mL 0.5% bupivacaine only. Success was defined as a ≥25% improvement in pain score on a 100-point VAS.

At 24 weeks, the PRP group demonstrated a success rate of 83.9% compared with 68.3% in the control group (number needed to treat=6; P=.037). However, the difference between the mean VAS improvement of 38 points in the PRP group and the mean decrease of 36 points in the control group carries little clinical significance.

References

1. Jindal N, Gaury Y, Banshiwal RC, et al. Comparison of short term results of single injection of autologous blood and steroid injection in tennis elbow: a prospective study. J Orthop Surg Res. 2013;8:10.

2. Dojode C. A randomised control trial to evaluate the efficacy of autologous blood injection versus local corticosteroid injection for treatment of lateral epicondylitis. Bone Joint Res. 2012;1:192-197.

3. Mishra A, Skrepnik NV, Edwards SG, et al. Efficacy of platelet-rich plasma for chronic tennis elbow: a double-blind, prospective, multicenter, randomized controlled trial of 230 patients. Am J Sports Med. 2014;42:463-471.

References

1. Jindal N, Gaury Y, Banshiwal RC, et al. Comparison of short term results of single injection of autologous blood and steroid injection in tennis elbow: a prospective study. J Orthop Surg Res. 2013;8:10.

2. Dojode C. A randomised control trial to evaluate the efficacy of autologous blood injection versus local corticosteroid injection for treatment of lateral epicondylitis. Bone Joint Res. 2012;1:192-197.

3. Mishra A, Skrepnik NV, Edwards SG, et al. Efficacy of platelet-rich plasma for chronic tennis elbow: a double-blind, prospective, multicenter, randomized controlled trial of 230 patients. Am J Sports Med. 2014;42:463-471.

Issue
The Journal of Family Practice - 65(9)
Issue
The Journal of Family Practice - 65(9)
Page Number
635
Page Number
635
Publications
Publications
Topics
Article Type
Display Headline
Do autologous blood and PRP injections effectively treat tennis elbow?
Display Headline
Do autologous blood and PRP injections effectively treat tennis elbow?
Sections
PURLs Copyright

Evidence-based answers from the Family Physicians Inquiries Network

Disallow All Ads
Article PDF Media

Which SSRIs most effectively treat depression in adolescents?

Article Type
Changed
Fri, 01/18/2019 - 08:36
Display Headline
Which SSRIs most effectively treat depression in adolescents?

EVIDENCE-BASED ANSWER:

We don’t know which selective serotonin reuptake inhibitors (SSRIs) are the most effective and safe because no studies have compared these antidepressants with each other.

Three SSRI antidepressant medications—fluoxetine, sertraline, and escitalopram—produce modest improvements (about 5% to 10%) in standardized depression scores without a significant increase in the risk of suicide-related outcomes (suicidal behavior or ideation) in adolescent patients with major depression of moderate severity. As a group, however, the newer-generation antidepressants, including SSRIs, increase suicide-related outcomes by 50%. Citalopram, paroxetine, venlafaxine, and mirtazapine don’t improve depression scores (strength of recommendation [SOR]: A, meta-analyses of randomized controlled trials [RCTs]).

An updated national guideline recommends specific psychological therapy for adolescents with mild depression and combined psychotherapy and fluoxetine for moderate or severe depression, with sertraline or citalopram as second-line agents (SOR: A, RCTs).

EVIDENCE SUMMARY

A Cochrane systematic review (19 RCTs; 3335 patients, total) of newer-generation antidepressants for treating depression in adolescents found that, overall, they produced both a small decrease in symptom severity scores and an increased risk of suicide-related outcomes.1

Three SSRIs slightly lower one symptom severity score

Investigators performed a meta-analysis of all trials (14 RCTs; 2490 patients, total) that used the same standardized symptom severity score (the Children’s Depression Rating Scale—Revised [CDRS-R], range 17 to 113 points) to evaluate the following medications: fluoxetine, sertraline, escitalopram, citalopram, paroxetine, venlafaxine, and mirtazapine.1

 

All participants were outpatients who met criteria for a primary diagnosis of major depression, excluding comorbid conditions. The CDRS-R scores were evaluated by clinicians; the mean baseline score was 57 (40 is considered a threshold score for diagnosis, and above 60 indicates severe symptoms). Only 5 trials reported patients’ self-rated depression symptom severity (in patients taking fluoxetine and paroxetine) and none reported improvement. Treatment courses ranged from 8 to 12 weeks.

As a group, the newer antidepressants slightly reduced CDRS-R scores in adolescents (by 4.21 points, 95% confidence interval [CI], 0.41-5.95) but increased suicide-related outcomes (relative risk [RR]=1.47; 95% CI, 0.99-2.19). The individual antidepressants fluoxetine, sertraline, and escitalopram each produced statistically significant but clinically small reductions in CDRS-R scores of 5% to 10% without significantly increasing suicide-related outcomes (TABLE1). The other medications evaluated individually didn’t improve CDRS-R scores, and only venlafaxine increased suicide-related outcomes.

 

 

Other symptom severity scores show no improvement with SSRIs

Five additional RCTs not included in the meta-analysis that used standardized symptom severity scores other than the CDRS-R (Schedule for Affective Disorders and Schizophrenia for School-Aged Children [K-SADS], Montgomery-Asberg Depression Rating Scale [MADR], and Hamilton Depression Rating Scale [HAM-D]) found no improvement with fluoxetine (2 RCTs; 63 patients, total), citalopram (one RCT, 233 patients), or paroxetine (2 RCTs; 466 patients, total).

Certain drugs cause significantly more adverse events than placebo

Ten RCTs evaluated adverse events in adolescents treated with fluoxetine, escitalopram, citalopram, and paroxetine and reported a small increase over placebo when all medications were combined as a group (RR=1.11; 95% CI, 1.05-1.17). Investigators reported that the individual antidepressants fluoxetine, escitalopram, venlafaxine, and mirtazapine produced significantly more adverse events than placebo (P values not given). No studies compared antidepressant medications against each other for either efficacy or potential harms.

RECOMMENDATIONS

A newly revised expert guideline recommends treating mildly depressed adolescents with a specific psychological therapy—individual cognitive behavioral therapy, interpersonal therapy, family therapy, or psychodynamic psychotherapy—for at least 3 months.2

For adolescents with moderate to severe depression, the guideline advocates psychotherapy with the option of adding fluoxetine, although using antidepressants in adolescents who haven’t at least tried psychotherapy is outside of the drug’s indications.

The guideline also recommends careful monitoring for adverse effects and close review of mental state—weekly for the first 4 weeks of treatment, for example. If fluoxetine doesn’t help, sertraline and citalopram are recommended as alternatives.

References

1. Hetrick SE, McKenzie JE, Cox GR, et al. Newer generation antidepressants for depressive disorders in children and adolescents. Cochrane Database Syst Rev. 2012;11:CD004851.

2. Hopkins K, Crosland P, Elliott N, et al. Diagnosis and management of depression in children and young people: summary of updated NICE guidance. BMJ. 2015;350:h824.

Article PDF
Author and Disclosure Information

Valory DeLucia, MD
Gary Kelsberg, MD

Valley Family Medicine Residency, University of Washington at Renton

Sarah Safranek, MLIS
University of Washington Health Science Library, Seattle

DEPUTY EDITOR
Jon Neher, MD

Valley Family Medicine Residency, University of Washington at Renton

Issue
The Journal of Family Practice - 65(9)
Publications
Topics
Page Number
632-634
Sections
Author and Disclosure Information

Valory DeLucia, MD
Gary Kelsberg, MD

Valley Family Medicine Residency, University of Washington at Renton

Sarah Safranek, MLIS
University of Washington Health Science Library, Seattle

DEPUTY EDITOR
Jon Neher, MD

Valley Family Medicine Residency, University of Washington at Renton

Author and Disclosure Information

Valory DeLucia, MD
Gary Kelsberg, MD

Valley Family Medicine Residency, University of Washington at Renton

Sarah Safranek, MLIS
University of Washington Health Science Library, Seattle

DEPUTY EDITOR
Jon Neher, MD

Valley Family Medicine Residency, University of Washington at Renton

Article PDF
Article PDF

EVIDENCE-BASED ANSWER:

We don’t know which selective serotonin reuptake inhibitors (SSRIs) are the most effective and safe because no studies have compared these antidepressants with each other.

Three SSRI antidepressant medications—fluoxetine, sertraline, and escitalopram—produce modest improvements (about 5% to 10%) in standardized depression scores without a significant increase in the risk of suicide-related outcomes (suicidal behavior or ideation) in adolescent patients with major depression of moderate severity. As a group, however, the newer-generation antidepressants, including SSRIs, increase suicide-related outcomes by 50%. Citalopram, paroxetine, venlafaxine, and mirtazapine don’t improve depression scores (strength of recommendation [SOR]: A, meta-analyses of randomized controlled trials [RCTs]).

An updated national guideline recommends specific psychological therapy for adolescents with mild depression and combined psychotherapy and fluoxetine for moderate or severe depression, with sertraline or citalopram as second-line agents (SOR: A, RCTs).

EVIDENCE SUMMARY

A Cochrane systematic review (19 RCTs; 3335 patients, total) of newer-generation antidepressants for treating depression in adolescents found that, overall, they produced both a small decrease in symptom severity scores and an increased risk of suicide-related outcomes.1

Three SSRIs slightly lower one symptom severity score

Investigators performed a meta-analysis of all trials (14 RCTs; 2490 patients, total) that used the same standardized symptom severity score (the Children’s Depression Rating Scale—Revised [CDRS-R], range 17 to 113 points) to evaluate the following medications: fluoxetine, sertraline, escitalopram, citalopram, paroxetine, venlafaxine, and mirtazapine.1

 

All participants were outpatients who met criteria for a primary diagnosis of major depression, excluding comorbid conditions. The CDRS-R scores were evaluated by clinicians; the mean baseline score was 57 (40 is considered a threshold score for diagnosis, and above 60 indicates severe symptoms). Only 5 trials reported patients’ self-rated depression symptom severity (in patients taking fluoxetine and paroxetine) and none reported improvement. Treatment courses ranged from 8 to 12 weeks.

As a group, the newer antidepressants slightly reduced CDRS-R scores in adolescents (by 4.21 points, 95% confidence interval [CI], 0.41-5.95) but increased suicide-related outcomes (relative risk [RR]=1.47; 95% CI, 0.99-2.19). The individual antidepressants fluoxetine, sertraline, and escitalopram each produced statistically significant but clinically small reductions in CDRS-R scores of 5% to 10% without significantly increasing suicide-related outcomes (TABLE1). The other medications evaluated individually didn’t improve CDRS-R scores, and only venlafaxine increased suicide-related outcomes.

 

 

Other symptom severity scores show no improvement with SSRIs

Five additional RCTs not included in the meta-analysis that used standardized symptom severity scores other than the CDRS-R (Schedule for Affective Disorders and Schizophrenia for School-Aged Children [K-SADS], Montgomery-Asberg Depression Rating Scale [MADR], and Hamilton Depression Rating Scale [HAM-D]) found no improvement with fluoxetine (2 RCTs; 63 patients, total), citalopram (one RCT, 233 patients), or paroxetine (2 RCTs; 466 patients, total).

Certain drugs cause significantly more adverse events than placebo

Ten RCTs evaluated adverse events in adolescents treated with fluoxetine, escitalopram, citalopram, and paroxetine and reported a small increase over placebo when all medications were combined as a group (RR=1.11; 95% CI, 1.05-1.17). Investigators reported that the individual antidepressants fluoxetine, escitalopram, venlafaxine, and mirtazapine produced significantly more adverse events than placebo (P values not given). No studies compared antidepressant medications against each other for either efficacy or potential harms.

RECOMMENDATIONS

A newly revised expert guideline recommends treating mildly depressed adolescents with a specific psychological therapy—individual cognitive behavioral therapy, interpersonal therapy, family therapy, or psychodynamic psychotherapy—for at least 3 months.2

For adolescents with moderate to severe depression, the guideline advocates psychotherapy with the option of adding fluoxetine, although using antidepressants in adolescents who haven’t at least tried psychotherapy is outside of the drug’s indications.

The guideline also recommends careful monitoring for adverse effects and close review of mental state—weekly for the first 4 weeks of treatment, for example. If fluoxetine doesn’t help, sertraline and citalopram are recommended as alternatives.

EVIDENCE-BASED ANSWER:

We don’t know which selective serotonin reuptake inhibitors (SSRIs) are the most effective and safe because no studies have compared these antidepressants with each other.

Three SSRI antidepressant medications—fluoxetine, sertraline, and escitalopram—produce modest improvements (about 5% to 10%) in standardized depression scores without a significant increase in the risk of suicide-related outcomes (suicidal behavior or ideation) in adolescent patients with major depression of moderate severity. As a group, however, the newer-generation antidepressants, including SSRIs, increase suicide-related outcomes by 50%. Citalopram, paroxetine, venlafaxine, and mirtazapine don’t improve depression scores (strength of recommendation [SOR]: A, meta-analyses of randomized controlled trials [RCTs]).

An updated national guideline recommends specific psychological therapy for adolescents with mild depression and combined psychotherapy and fluoxetine for moderate or severe depression, with sertraline or citalopram as second-line agents (SOR: A, RCTs).

EVIDENCE SUMMARY

A Cochrane systematic review (19 RCTs; 3335 patients, total) of newer-generation antidepressants for treating depression in adolescents found that, overall, they produced both a small decrease in symptom severity scores and an increased risk of suicide-related outcomes.1

Three SSRIs slightly lower one symptom severity score

Investigators performed a meta-analysis of all trials (14 RCTs; 2490 patients, total) that used the same standardized symptom severity score (the Children’s Depression Rating Scale—Revised [CDRS-R], range 17 to 113 points) to evaluate the following medications: fluoxetine, sertraline, escitalopram, citalopram, paroxetine, venlafaxine, and mirtazapine.1

 

All participants were outpatients who met criteria for a primary diagnosis of major depression, excluding comorbid conditions. The CDRS-R scores were evaluated by clinicians; the mean baseline score was 57 (40 is considered a threshold score for diagnosis, and above 60 indicates severe symptoms). Only 5 trials reported patients’ self-rated depression symptom severity (in patients taking fluoxetine and paroxetine) and none reported improvement. Treatment courses ranged from 8 to 12 weeks.

As a group, the newer antidepressants slightly reduced CDRS-R scores in adolescents (by 4.21 points, 95% confidence interval [CI], 0.41-5.95) but increased suicide-related outcomes (relative risk [RR]=1.47; 95% CI, 0.99-2.19). The individual antidepressants fluoxetine, sertraline, and escitalopram each produced statistically significant but clinically small reductions in CDRS-R scores of 5% to 10% without significantly increasing suicide-related outcomes (TABLE1). The other medications evaluated individually didn’t improve CDRS-R scores, and only venlafaxine increased suicide-related outcomes.

 

 

Other symptom severity scores show no improvement with SSRIs

Five additional RCTs not included in the meta-analysis that used standardized symptom severity scores other than the CDRS-R (Schedule for Affective Disorders and Schizophrenia for School-Aged Children [K-SADS], Montgomery-Asberg Depression Rating Scale [MADR], and Hamilton Depression Rating Scale [HAM-D]) found no improvement with fluoxetine (2 RCTs; 63 patients, total), citalopram (one RCT, 233 patients), or paroxetine (2 RCTs; 466 patients, total).

Certain drugs cause significantly more adverse events than placebo

Ten RCTs evaluated adverse events in adolescents treated with fluoxetine, escitalopram, citalopram, and paroxetine and reported a small increase over placebo when all medications were combined as a group (RR=1.11; 95% CI, 1.05-1.17). Investigators reported that the individual antidepressants fluoxetine, escitalopram, venlafaxine, and mirtazapine produced significantly more adverse events than placebo (P values not given). No studies compared antidepressant medications against each other for either efficacy or potential harms.

RECOMMENDATIONS

A newly revised expert guideline recommends treating mildly depressed adolescents with a specific psychological therapy—individual cognitive behavioral therapy, interpersonal therapy, family therapy, or psychodynamic psychotherapy—for at least 3 months.2

For adolescents with moderate to severe depression, the guideline advocates psychotherapy with the option of adding fluoxetine, although using antidepressants in adolescents who haven’t at least tried psychotherapy is outside of the drug’s indications.

The guideline also recommends careful monitoring for adverse effects and close review of mental state—weekly for the first 4 weeks of treatment, for example. If fluoxetine doesn’t help, sertraline and citalopram are recommended as alternatives.

References

1. Hetrick SE, McKenzie JE, Cox GR, et al. Newer generation antidepressants for depressive disorders in children and adolescents. Cochrane Database Syst Rev. 2012;11:CD004851.

2. Hopkins K, Crosland P, Elliott N, et al. Diagnosis and management of depression in children and young people: summary of updated NICE guidance. BMJ. 2015;350:h824.

References

1. Hetrick SE, McKenzie JE, Cox GR, et al. Newer generation antidepressants for depressive disorders in children and adolescents. Cochrane Database Syst Rev. 2012;11:CD004851.

2. Hopkins K, Crosland P, Elliott N, et al. Diagnosis and management of depression in children and young people: summary of updated NICE guidance. BMJ. 2015;350:h824.

Issue
The Journal of Family Practice - 65(9)
Issue
The Journal of Family Practice - 65(9)
Page Number
632-634
Page Number
632-634
Publications
Publications
Topics
Article Type
Display Headline
Which SSRIs most effectively treat depression in adolescents?
Display Headline
Which SSRIs most effectively treat depression in adolescents?
Sections
PURLs Copyright

Evidence-based answers from the Family Physicians Inquiries Network

Disallow All Ads
Article PDF Media

A Melting Pot of Mail

Article Type
Changed
Thu, 03/28/2019 - 15:03
Display Headline
A Melting Pot of Mail

VAPING DANGERS: CLEARING THE AIR

The liquid base of an e-cigarette contains either vegetable glycerin (VG) or propylene glycol, or more commonly, a proprietary combination of both. Each of these ingredients has varying effects on the body.

However, the first paragraph of Randy D. Danielsen’s editorial alluded to what I consider a bigger concern regarding the future medical complications of vaping. The description of a “… huge puff of cherry-scented smoke …” indicates that vapes are not puffed on the way cigarettes are.

Cigarette smoking is similar to drinking through a straw—the smoke is first captured in the mouth, then cooled and inhaled. In contrast, vaping involves inhaling smoke directly into the lungs. This action, along with the thick VG base, produces a high volume of smoke. Vape shops even sponsor contests to see who can produce the largest cloud of smoke.

Therefore, my concern regarding vaping is not limited to the toxicity of the ingredients; it extends to how the toxicants are delivered to the poor, unsuspecting alveoli.

Gary Dula, FNP-C
Houston, TX

Continue for Millenials: Not All Sitting at the Kids' Table >>

 

 

MILLENIALS: NOT ALL SITTING AT THE KIDS' TABLES

I received my master’s degree in 2015 and am nearing completion of a year-long FNP fellowship program. I was an Army nurse for four years and a float nurse at various hospitals for five. I am a “millennial”—and, according to the published letters about precepting, am hated by older nurses because of it. Considering I have practiced with many hard-working people my age who would lay down their lives for this country, I find this unprecedented.

I work hard, but the school I attended for my FNP did not prepare me well; it was difficult to get people to teach and precept me during school. This led me to apply for my current fellowship.

Throughout my nine-year nursing career, I have precepted many nurses, including those with associate degrees. I will continue to mentor and precept as an APRN. I take issue with the portrayal of millennials as lazy and unable to work hard. Why? Because we will not work for free, would like to collaboratively learn, and need help to develop our skills?

One day, you will grow old and need someone to take care of you. Why on earth would you berate the people who will be doing just that? Complaining about this generation is not going to change the fact that they are here and present in the workforce. We need more providers, and chastising the younger generation is not going to solve that problem.

Stephanie Butler-Cleland, FNP-BC
Colorado Springs, CO

Continue for The Pros of Precepting >>

 

 

THE PROS OF PRECEPTING

I am an urgent care NP in urban communities on the West Coast of Florida. I had taken a break from precepting as a result of negative experiences, but I recently resumed to precept my first NP student in years.

Prior to accepting the student I precepted, I received requests from two other students. One asked if I could change my schedule to be closer to where she lived. The other clearly didn’t want to commit to the drive or the hours I was available, and asked if I would work more weekends to accommodate her schedule. Needless to say, I refused both students.

Instead, I precepted a smart 28-year-old student from my alma mater, one of the Florida state universities. She was attentive, prepared, and eager. I was very, very impressed with her. She had been a nurse for four years and was a second-semester student. It was a pleasure to have her; I like being questioned and challenged. It was fun to see her enjoying my job, and it reminded me of why I love what I do.

Anne Conklin, MS, ARNP-C
Bradenton, FL

Continue for A Scheming Industry >>

 

 

A SCHEMING INDUSTRY

Intelligent health care policy has been frustrated by the enormous amount of money brought to bear on Congress by the insurance and pharmaceutical industries. Each dollar paid to an insurance company is used to construct buildings, hire workers, create a sales staff, and ultimately pay their shareholders a profit.

Since the insurance industry obtained an antitrust exemption in the 1940s, they are essentially immune from prosecution for price collusion. Until recently, it was difficult to know how much of the money paid was returned in the form of medical benefits. In order to keep profits rising, they must enroll more people. Promising coverage while impeding medical workups and care, making great profits, and needing more and more enrollees fits the definition of a Ponzi scheme.

Several years ago in California, the state insurance commission (under threat of decertification) got an industry representative to admit that the maximum percentage of dollars used for services was 70%. In other words, for each dollar spent, a patient would be lucky to get 70 cents worth of services.

All of us who practice know how the companies do this: We request a needed diagnostic test or treatment and are denied. I have interrupted my schedule on many days to call for a “peer to peer” review—only once was I denied. This is a roadblock that many busy practitioners will not challenge. Since insurance companies market how great their coverage is, patients often get angry at the provider.

The repeated argument is that the market forces will lower medical costs. This fallacy is easily debunked by noting the ever-escalating costs and comparing health care costs as a percent of gross domestic product (GDP) in our country versus others. France, for example, expends 12% of GDP on health and ranks first in health care outcomes by world standards. In the US, we are approaching 20% of GDP.

Since insurance adds nothing to care and increases costs dramatically (every provider has to have billers for the various insurance companies, since each has its own requirements), a single-payer system is the only system that will lower costs. Those who benefit from the current system declare that we can’t have “socialized medicine.” To which I would respond, fine; we’ll continue to pay 30% to 50% more so that insurance companies can have their profits at our expense.

Nelson Herilhy, PA-C, MHS
Concord, CA

Article PDF
Issue
Clinician Reviews - 26(9)
Publications
Topics
Page Number
12,54
Sections
Article PDF
Article PDF
Related Articles

VAPING DANGERS: CLEARING THE AIR

The liquid base of an e-cigarette contains either vegetable glycerin (VG) or propylene glycol, or more commonly, a proprietary combination of both. Each of these ingredients has varying effects on the body.

However, the first paragraph of Randy D. Danielsen’s editorial alluded to what I consider a bigger concern regarding the future medical complications of vaping. The description of a “… huge puff of cherry-scented smoke …” indicates that vapes are not puffed on the way cigarettes are.

Cigarette smoking is similar to drinking through a straw—the smoke is first captured in the mouth, then cooled and inhaled. In contrast, vaping involves inhaling smoke directly into the lungs. This action, along with the thick VG base, produces a high volume of smoke. Vape shops even sponsor contests to see who can produce the largest cloud of smoke.

Therefore, my concern regarding vaping is not limited to the toxicity of the ingredients; it extends to how the toxicants are delivered to the poor, unsuspecting alveoli.

Gary Dula, FNP-C
Houston, TX

Continue for Millenials: Not All Sitting at the Kids' Table >>

 

 

MILLENIALS: NOT ALL SITTING AT THE KIDS' TABLES

I received my master’s degree in 2015 and am nearing completion of a year-long FNP fellowship program. I was an Army nurse for four years and a float nurse at various hospitals for five. I am a “millennial”—and, according to the published letters about precepting, am hated by older nurses because of it. Considering I have practiced with many hard-working people my age who would lay down their lives for this country, I find this unprecedented.

I work hard, but the school I attended for my FNP did not prepare me well; it was difficult to get people to teach and precept me during school. This led me to apply for my current fellowship.

Throughout my nine-year nursing career, I have precepted many nurses, including those with associate degrees. I will continue to mentor and precept as an APRN. I take issue with the portrayal of millennials as lazy and unable to work hard. Why? Because we will not work for free, would like to collaboratively learn, and need help to develop our skills?

One day, you will grow old and need someone to take care of you. Why on earth would you berate the people who will be doing just that? Complaining about this generation is not going to change the fact that they are here and present in the workforce. We need more providers, and chastising the younger generation is not going to solve that problem.

Stephanie Butler-Cleland, FNP-BC
Colorado Springs, CO

Continue for The Pros of Precepting >>

 

 

THE PROS OF PRECEPTING

I am an urgent care NP in urban communities on the West Coast of Florida. I had taken a break from precepting as a result of negative experiences, but I recently resumed to precept my first NP student in years.

Prior to accepting the student I precepted, I received requests from two other students. One asked if I could change my schedule to be closer to where she lived. The other clearly didn’t want to commit to the drive or the hours I was available, and asked if I would work more weekends to accommodate her schedule. Needless to say, I refused both students.

Instead, I precepted a smart 28-year-old student from my alma mater, one of the Florida state universities. She was attentive, prepared, and eager. I was very, very impressed with her. She had been a nurse for four years and was a second-semester student. It was a pleasure to have her; I like being questioned and challenged. It was fun to see her enjoying my job, and it reminded me of why I love what I do.

Anne Conklin, MS, ARNP-C
Bradenton, FL

Continue for A Scheming Industry >>

 

 

A SCHEMING INDUSTRY

Intelligent health care policy has been frustrated by the enormous amount of money brought to bear on Congress by the insurance and pharmaceutical industries. Each dollar paid to an insurance company is used to construct buildings, hire workers, create a sales staff, and ultimately pay their shareholders a profit.

Since the insurance industry obtained an antitrust exemption in the 1940s, they are essentially immune from prosecution for price collusion. Until recently, it was difficult to know how much of the money paid was returned in the form of medical benefits. In order to keep profits rising, they must enroll more people. Promising coverage while impeding medical workups and care, making great profits, and needing more and more enrollees fits the definition of a Ponzi scheme.

Several years ago in California, the state insurance commission (under threat of decertification) got an industry representative to admit that the maximum percentage of dollars used for services was 70%. In other words, for each dollar spent, a patient would be lucky to get 70 cents worth of services.

All of us who practice know how the companies do this: We request a needed diagnostic test or treatment and are denied. I have interrupted my schedule on many days to call for a “peer to peer” review—only once was I denied. This is a roadblock that many busy practitioners will not challenge. Since insurance companies market how great their coverage is, patients often get angry at the provider.

The repeated argument is that the market forces will lower medical costs. This fallacy is easily debunked by noting the ever-escalating costs and comparing health care costs as a percent of gross domestic product (GDP) in our country versus others. France, for example, expends 12% of GDP on health and ranks first in health care outcomes by world standards. In the US, we are approaching 20% of GDP.

Since insurance adds nothing to care and increases costs dramatically (every provider has to have billers for the various insurance companies, since each has its own requirements), a single-payer system is the only system that will lower costs. Those who benefit from the current system declare that we can’t have “socialized medicine.” To which I would respond, fine; we’ll continue to pay 30% to 50% more so that insurance companies can have their profits at our expense.

Nelson Herilhy, PA-C, MHS
Concord, CA

VAPING DANGERS: CLEARING THE AIR

The liquid base of an e-cigarette contains either vegetable glycerin (VG) or propylene glycol, or more commonly, a proprietary combination of both. Each of these ingredients has varying effects on the body.

However, the first paragraph of Randy D. Danielsen’s editorial alluded to what I consider a bigger concern regarding the future medical complications of vaping. The description of a “… huge puff of cherry-scented smoke …” indicates that vapes are not puffed on the way cigarettes are.

Cigarette smoking is similar to drinking through a straw—the smoke is first captured in the mouth, then cooled and inhaled. In contrast, vaping involves inhaling smoke directly into the lungs. This action, along with the thick VG base, produces a high volume of smoke. Vape shops even sponsor contests to see who can produce the largest cloud of smoke.

Therefore, my concern regarding vaping is not limited to the toxicity of the ingredients; it extends to how the toxicants are delivered to the poor, unsuspecting alveoli.

Gary Dula, FNP-C
Houston, TX

Continue for Millenials: Not All Sitting at the Kids' Table >>

 

 

MILLENIALS: NOT ALL SITTING AT THE KIDS' TABLES

I received my master’s degree in 2015 and am nearing completion of a year-long FNP fellowship program. I was an Army nurse for four years and a float nurse at various hospitals for five. I am a “millennial”—and, according to the published letters about precepting, am hated by older nurses because of it. Considering I have practiced with many hard-working people my age who would lay down their lives for this country, I find this unprecedented.

I work hard, but the school I attended for my FNP did not prepare me well; it was difficult to get people to teach and precept me during school. This led me to apply for my current fellowship.

Throughout my nine-year nursing career, I have precepted many nurses, including those with associate degrees. I will continue to mentor and precept as an APRN. I take issue with the portrayal of millennials as lazy and unable to work hard. Why? Because we will not work for free, would like to collaboratively learn, and need help to develop our skills?

One day, you will grow old and need someone to take care of you. Why on earth would you berate the people who will be doing just that? Complaining about this generation is not going to change the fact that they are here and present in the workforce. We need more providers, and chastising the younger generation is not going to solve that problem.

Stephanie Butler-Cleland, FNP-BC
Colorado Springs, CO

Continue for The Pros of Precepting >>

 

 

THE PROS OF PRECEPTING

I am an urgent care NP in urban communities on the West Coast of Florida. I had taken a break from precepting as a result of negative experiences, but I recently resumed to precept my first NP student in years.

Prior to accepting the student I precepted, I received requests from two other students. One asked if I could change my schedule to be closer to where she lived. The other clearly didn’t want to commit to the drive or the hours I was available, and asked if I would work more weekends to accommodate her schedule. Needless to say, I refused both students.

Instead, I precepted a smart 28-year-old student from my alma mater, one of the Florida state universities. She was attentive, prepared, and eager. I was very, very impressed with her. She had been a nurse for four years and was a second-semester student. It was a pleasure to have her; I like being questioned and challenged. It was fun to see her enjoying my job, and it reminded me of why I love what I do.

Anne Conklin, MS, ARNP-C
Bradenton, FL

Continue for A Scheming Industry >>

 

 

A SCHEMING INDUSTRY

Intelligent health care policy has been frustrated by the enormous amount of money brought to bear on Congress by the insurance and pharmaceutical industries. Each dollar paid to an insurance company is used to construct buildings, hire workers, create a sales staff, and ultimately pay their shareholders a profit.

Since the insurance industry obtained an antitrust exemption in the 1940s, they are essentially immune from prosecution for price collusion. Until recently, it was difficult to know how much of the money paid was returned in the form of medical benefits. In order to keep profits rising, they must enroll more people. Promising coverage while impeding medical workups and care, making great profits, and needing more and more enrollees fits the definition of a Ponzi scheme.

Several years ago in California, the state insurance commission (under threat of decertification) got an industry representative to admit that the maximum percentage of dollars used for services was 70%. In other words, for each dollar spent, a patient would be lucky to get 70 cents worth of services.

All of us who practice know how the companies do this: We request a needed diagnostic test or treatment and are denied. I have interrupted my schedule on many days to call for a “peer to peer” review—only once was I denied. This is a roadblock that many busy practitioners will not challenge. Since insurance companies market how great their coverage is, patients often get angry at the provider.

The repeated argument is that the market forces will lower medical costs. This fallacy is easily debunked by noting the ever-escalating costs and comparing health care costs as a percent of gross domestic product (GDP) in our country versus others. France, for example, expends 12% of GDP on health and ranks first in health care outcomes by world standards. In the US, we are approaching 20% of GDP.

Since insurance adds nothing to care and increases costs dramatically (every provider has to have billers for the various insurance companies, since each has its own requirements), a single-payer system is the only system that will lower costs. Those who benefit from the current system declare that we can’t have “socialized medicine.” To which I would respond, fine; we’ll continue to pay 30% to 50% more so that insurance companies can have their profits at our expense.

Nelson Herilhy, PA-C, MHS
Concord, CA

Issue
Clinician Reviews - 26(9)
Issue
Clinician Reviews - 26(9)
Page Number
12,54
Page Number
12,54
Publications
Publications
Topics
Article Type
Display Headline
A Melting Pot of Mail
Display Headline
A Melting Pot of Mail
Sections
Disallow All Ads
Article PDF Media

“Unprecedented” VA Proposal? We Don’t Think So

Article Type
Changed
Thu, 03/28/2019 - 15:03
Display Headline
“Unprecedented” VA Proposal? We Don’t Think So

On May 25, 2016, the Department of Veterans Affairs (VA) published a proposed rule change in the Federal Register under the simple heading “Advanced Practice Registered Nurses.” From such modest beginnings stemmed a potential game-changer for advanced practice clinicians in this country: In summary, the VA proposed to “amend its medical regulations to permit full practice authority of all VA advanced practice registered nurses (APRNs) when they are acting within the scope of their VA employment.”1

The impetus for the VA’s proposal is that 505,000 veterans wait 30 days to access care within the VA system—and 300,000 wait between 31 and 60 days for health services.2 Granting plenary practice to VA APRNs would enable them to respond to this backlog of patients, since veterans would have direct access to APRNs who practice within the VA system, regardless of their state of licensure.

More than 4,800 NPs work within the VA; they provide clinical assessments, order appropriate tests and medications, and develop patient-centered care plans.2,3 Research has documented that outcomes for patients whose care is managed by NPs are equal to or better than outcomes for similar patients who are managed by physicians.4 As Major General Vincent Boles of the US Army (retired) stated, “Veterans rely on VA health care to take care of them, and the VA’s nurse practitioners are qualified to provide our veterans with the care they need and deserve.”4

Allowing veterans access to high-quality care is a 21st century solution that is “zero risk, zero cost, zero delay,” according to Dr. Cindy Cooke, President of the American Association of Nurse Practitioners (AANP).4 And it is not just the AANP that supports this rule change. Ninety-one percent of US households that are home to a veteran, and 88% of Americans overall, express support for the VA proposal. In a Mellman Group survey of more than 1,000 adults, strong support was noted across party lines (91% of Republicans; 90% of Democrats)—a rarity in our current political climate.4

Support for full practice authority for NPs at the VA has come from more than 60 organizations, including the Military Officers’ Association of America, the Air Force Sergeants Association, AARP (with 3.7 million veteran households in its membership), and 80 bipartisan members of Congress.5 At the AANP annual conference in San Antonio, Dr. Cooke was joined by leaders from the local American Legion and retired military officers who announced their support for this “change in practice.”3

However, among the more than 162,000 comments received by the VA during the public comment period, there were dissenting opinions. On July 13, 2016, Dr. Robert Wergin, Chair of the Board of the American Academy of Family Physicians (AAFP), sent a letter to Dr. David Shulkin, the Undersecretary of Health in the VA, stating that there were “significant concerns” about the rule change. His main point was that granting full practice authority to NPs would “alter the consistent standards of care for veterans over nonveterans in the states; further fragment the health care system; and dismantle physician-led team-based health care models.” He also stated that “the AAFP strongly opposes the unprecedented proposal to dismiss state practice authority regarding the authority of NPs.”6

Unprecedented? I don’t think so. I practiced as a family NP in the Navy for more than 20 years. I had my own patient panel, cared for active duty members and their families, and evaluated outcomes the same way my physician ­colleagues did. We practiced collaboratively and respectfully. We discussed patient plan issues, provided peer review on one another’s charts, and accepted new patients into our panels. It was a true collaborative practice.

Military nurses only need to be licensed in one state. The guidelines for NP practice were not based on the rules of the state in which we were licensed but were established by our professional practice association—just as the guidelines for physician practice were not based on the rules extant in their licensing state. I practiced successfully in many states and overseas, although I was licensed in a state that did not recognize plenary practice at the time.

The VA is attempting to respond to veterans’ need for access to care by adopting a model similar to what the military employs. It’s not a matter of superseding state regulations; it’s a matter of recognizing the education and training of health care professionals who can improve patient outcomes.

The opportunity to respond to the proposed amendment has now closed. Through its grassroots Veterans Deserve Care campaign, the AANP and its partners and supporters—clinicians, veterans, families, and others—submitted nearly 60,000 comments.2 Now we wait for the VA to review the abundance of feedback and issue their final decision.

I am hopeful that the VA will acknowledge the overwhelming evidence that our veterans deserve access to care led by highly qualified professionals. The old system isn’t working. Einstein said that the definition of insanity was to do the same thing over and over and expect a different outcome; maintaining a faulty system fits that description. NPs have a well-tested, evidence-based, high-quality education that encourages their ability to lead health care teams, perform collaboratively, and improve outcomes for those who have served our country.

Caring for active duty military and veterans is in the DNA of nurses. Florence Nightingale spent much of her post-Crimea life using evidence-based proposals and political influence to improve the health care of the soldiers and veterans of the British Empire. In Notes on Nursing, she spurred nurses to political action: “Let whoever [sic] is in charge keep this simple question in her [sic] head (not how can I always do this right thing myself, but) how can I provide for this right thing to be always done?7 This advice should be taken to heart by all health care professionals: We can honor our veterans by advocating for and providing the health care access they need.

To share your thoughts, please contact us at [email protected]

References

1. Advanced practice registered nurses [2016-12338]. Fed Regist. May 25, 2016. https://federalregister.gov/a/2016-12338.
2. American Association of Nurse Practitioners. AANP and Air Force Sergeants Association urge VA to swiftly enact proposed rule. July 25, 2016. www.aanp.org/legislation-regu lation/federal-legislation/va-proposed-rule/173-press-room/2016-press-releases/ 1987-aanp-and-air-force-sergeants-associa tion-urge-va-to-swiftly-enact-proposed-rule. Accessed August 9, 2016.
3. American Association of Nurse Practitioners. AANP and veterans groups call for streamlined access to veteran’s health care. June 23, 2016. www.aanp.org/press-room/press-releases/173-press-room/2016-press-releases/1959-aanp-veteran-groups-call-for-streamlined-access-to-veterans-health-care. Accessed August 9, 2016.
4. American Association of Nurse Practitioners. National survey finds overwhelming support for VA rule granting veterans direct access to nurse practitioner care. July 20, 2016. www.aanp.org/press-room/press-releases/173-press-room/2016-press-releases/1986-national-survey-finds-overwhelming-support-for-va-rule-granting-veterans-direct-access-to-nurse-practition er-care. Accessed August 9, 2016.
5. American Association of Nurse Anesthetists. Nursing coalition and veterans groups join forces in unprecedented response to VA proposed rule to increase veterans’ access to care. June 28, 2016. www.aana.com/newsandjournal/News/Pages/062816-Nursing-Coalition-and-Veterans-Groups-Join-Forces-in-Unprecedented-Response-to-VA-Proposed-Rule.aspx. Accessed August 9, 2016.
6. Wergin RL. Letter to David Shulkin. July 13, 2016. www.aafp.org/dam/AAFP/docu ments/advocacy/workforce/scope/LT-VHA-APRN-071316.pdf. Accessed August 9, 2016 .
7. Nightingale F. Notes on Nursing: What It Is and What It Is Not. New York, NY: D. Appleton and Company; 1860.

Article PDF
Author and Disclosure Information

Elizabeth R. Baker, PhD, APRN, BC, FAANP, FACHE, FNAP, FAAN

Elizabeth R. Barker, a member of the Clinician Reviews editorial board, is Professor of Clinical Nursing (Emeritus) and a family nurse practitioner at University Health Connections/University Health Services, The Ohio State University, Columbus.

Issue
Clinician Reviews - 26(9)
Publications
Topics
Page Number
11,52
Sections
Author and Disclosure Information

Elizabeth R. Baker, PhD, APRN, BC, FAANP, FACHE, FNAP, FAAN

Elizabeth R. Barker, a member of the Clinician Reviews editorial board, is Professor of Clinical Nursing (Emeritus) and a family nurse practitioner at University Health Connections/University Health Services, The Ohio State University, Columbus.

Author and Disclosure Information

Elizabeth R. Baker, PhD, APRN, BC, FAANP, FACHE, FNAP, FAAN

Elizabeth R. Barker, a member of the Clinician Reviews editorial board, is Professor of Clinical Nursing (Emeritus) and a family nurse practitioner at University Health Connections/University Health Services, The Ohio State University, Columbus.

Article PDF
Article PDF
Related Articles

On May 25, 2016, the Department of Veterans Affairs (VA) published a proposed rule change in the Federal Register under the simple heading “Advanced Practice Registered Nurses.” From such modest beginnings stemmed a potential game-changer for advanced practice clinicians in this country: In summary, the VA proposed to “amend its medical regulations to permit full practice authority of all VA advanced practice registered nurses (APRNs) when they are acting within the scope of their VA employment.”1

The impetus for the VA’s proposal is that 505,000 veterans wait 30 days to access care within the VA system—and 300,000 wait between 31 and 60 days for health services.2 Granting plenary practice to VA APRNs would enable them to respond to this backlog of patients, since veterans would have direct access to APRNs who practice within the VA system, regardless of their state of licensure.

More than 4,800 NPs work within the VA; they provide clinical assessments, order appropriate tests and medications, and develop patient-centered care plans.2,3 Research has documented that outcomes for patients whose care is managed by NPs are equal to or better than outcomes for similar patients who are managed by physicians.4 As Major General Vincent Boles of the US Army (retired) stated, “Veterans rely on VA health care to take care of them, and the VA’s nurse practitioners are qualified to provide our veterans with the care they need and deserve.”4

Allowing veterans access to high-quality care is a 21st century solution that is “zero risk, zero cost, zero delay,” according to Dr. Cindy Cooke, President of the American Association of Nurse Practitioners (AANP).4 And it is not just the AANP that supports this rule change. Ninety-one percent of US households that are home to a veteran, and 88% of Americans overall, express support for the VA proposal. In a Mellman Group survey of more than 1,000 adults, strong support was noted across party lines (91% of Republicans; 90% of Democrats)—a rarity in our current political climate.4

Support for full practice authority for NPs at the VA has come from more than 60 organizations, including the Military Officers’ Association of America, the Air Force Sergeants Association, AARP (with 3.7 million veteran households in its membership), and 80 bipartisan members of Congress.5 At the AANP annual conference in San Antonio, Dr. Cooke was joined by leaders from the local American Legion and retired military officers who announced their support for this “change in practice.”3

However, among the more than 162,000 comments received by the VA during the public comment period, there were dissenting opinions. On July 13, 2016, Dr. Robert Wergin, Chair of the Board of the American Academy of Family Physicians (AAFP), sent a letter to Dr. David Shulkin, the Undersecretary of Health in the VA, stating that there were “significant concerns” about the rule change. His main point was that granting full practice authority to NPs would “alter the consistent standards of care for veterans over nonveterans in the states; further fragment the health care system; and dismantle physician-led team-based health care models.” He also stated that “the AAFP strongly opposes the unprecedented proposal to dismiss state practice authority regarding the authority of NPs.”6

Unprecedented? I don’t think so. I practiced as a family NP in the Navy for more than 20 years. I had my own patient panel, cared for active duty members and their families, and evaluated outcomes the same way my physician ­colleagues did. We practiced collaboratively and respectfully. We discussed patient plan issues, provided peer review on one another’s charts, and accepted new patients into our panels. It was a true collaborative practice.

Military nurses only need to be licensed in one state. The guidelines for NP practice were not based on the rules of the state in which we were licensed but were established by our professional practice association—just as the guidelines for physician practice were not based on the rules extant in their licensing state. I practiced successfully in many states and overseas, although I was licensed in a state that did not recognize plenary practice at the time.

The VA is attempting to respond to veterans’ need for access to care by adopting a model similar to what the military employs. It’s not a matter of superseding state regulations; it’s a matter of recognizing the education and training of health care professionals who can improve patient outcomes.

The opportunity to respond to the proposed amendment has now closed. Through its grassroots Veterans Deserve Care campaign, the AANP and its partners and supporters—clinicians, veterans, families, and others—submitted nearly 60,000 comments.2 Now we wait for the VA to review the abundance of feedback and issue their final decision.

I am hopeful that the VA will acknowledge the overwhelming evidence that our veterans deserve access to care led by highly qualified professionals. The old system isn’t working. Einstein said that the definition of insanity was to do the same thing over and over and expect a different outcome; maintaining a faulty system fits that description. NPs have a well-tested, evidence-based, high-quality education that encourages their ability to lead health care teams, perform collaboratively, and improve outcomes for those who have served our country.

Caring for active duty military and veterans is in the DNA of nurses. Florence Nightingale spent much of her post-Crimea life using evidence-based proposals and political influence to improve the health care of the soldiers and veterans of the British Empire. In Notes on Nursing, she spurred nurses to political action: “Let whoever [sic] is in charge keep this simple question in her [sic] head (not how can I always do this right thing myself, but) how can I provide for this right thing to be always done?7 This advice should be taken to heart by all health care professionals: We can honor our veterans by advocating for and providing the health care access they need.

To share your thoughts, please contact us at [email protected]

On May 25, 2016, the Department of Veterans Affairs (VA) published a proposed rule change in the Federal Register under the simple heading “Advanced Practice Registered Nurses.” From such modest beginnings stemmed a potential game-changer for advanced practice clinicians in this country: In summary, the VA proposed to “amend its medical regulations to permit full practice authority of all VA advanced practice registered nurses (APRNs) when they are acting within the scope of their VA employment.”1

The impetus for the VA’s proposal is that 505,000 veterans wait 30 days to access care within the VA system—and 300,000 wait between 31 and 60 days for health services.2 Granting plenary practice to VA APRNs would enable them to respond to this backlog of patients, since veterans would have direct access to APRNs who practice within the VA system, regardless of their state of licensure.

More than 4,800 NPs work within the VA; they provide clinical assessments, order appropriate tests and medications, and develop patient-centered care plans.2,3 Research has documented that outcomes for patients whose care is managed by NPs are equal to or better than outcomes for similar patients who are managed by physicians.4 As Major General Vincent Boles of the US Army (retired) stated, “Veterans rely on VA health care to take care of them, and the VA’s nurse practitioners are qualified to provide our veterans with the care they need and deserve.”4

Allowing veterans access to high-quality care is a 21st century solution that is “zero risk, zero cost, zero delay,” according to Dr. Cindy Cooke, President of the American Association of Nurse Practitioners (AANP).4 And it is not just the AANP that supports this rule change. Ninety-one percent of US households that are home to a veteran, and 88% of Americans overall, express support for the VA proposal. In a Mellman Group survey of more than 1,000 adults, strong support was noted across party lines (91% of Republicans; 90% of Democrats)—a rarity in our current political climate.4

Support for full practice authority for NPs at the VA has come from more than 60 organizations, including the Military Officers’ Association of America, the Air Force Sergeants Association, AARP (with 3.7 million veteran households in its membership), and 80 bipartisan members of Congress.5 At the AANP annual conference in San Antonio, Dr. Cooke was joined by leaders from the local American Legion and retired military officers who announced their support for this “change in practice.”3

However, among the more than 162,000 comments received by the VA during the public comment period, there were dissenting opinions. On July 13, 2016, Dr. Robert Wergin, Chair of the Board of the American Academy of Family Physicians (AAFP), sent a letter to Dr. David Shulkin, the Undersecretary of Health in the VA, stating that there were “significant concerns” about the rule change. His main point was that granting full practice authority to NPs would “alter the consistent standards of care for veterans over nonveterans in the states; further fragment the health care system; and dismantle physician-led team-based health care models.” He also stated that “the AAFP strongly opposes the unprecedented proposal to dismiss state practice authority regarding the authority of NPs.”6

Unprecedented? I don’t think so. I practiced as a family NP in the Navy for more than 20 years. I had my own patient panel, cared for active duty members and their families, and evaluated outcomes the same way my physician ­colleagues did. We practiced collaboratively and respectfully. We discussed patient plan issues, provided peer review on one another’s charts, and accepted new patients into our panels. It was a true collaborative practice.

Military nurses only need to be licensed in one state. The guidelines for NP practice were not based on the rules of the state in which we were licensed but were established by our professional practice association—just as the guidelines for physician practice were not based on the rules extant in their licensing state. I practiced successfully in many states and overseas, although I was licensed in a state that did not recognize plenary practice at the time.

The VA is attempting to respond to veterans’ need for access to care by adopting a model similar to what the military employs. It’s not a matter of superseding state regulations; it’s a matter of recognizing the education and training of health care professionals who can improve patient outcomes.

The opportunity to respond to the proposed amendment has now closed. Through its grassroots Veterans Deserve Care campaign, the AANP and its partners and supporters—clinicians, veterans, families, and others—submitted nearly 60,000 comments.2 Now we wait for the VA to review the abundance of feedback and issue their final decision.

I am hopeful that the VA will acknowledge the overwhelming evidence that our veterans deserve access to care led by highly qualified professionals. The old system isn’t working. Einstein said that the definition of insanity was to do the same thing over and over and expect a different outcome; maintaining a faulty system fits that description. NPs have a well-tested, evidence-based, high-quality education that encourages their ability to lead health care teams, perform collaboratively, and improve outcomes for those who have served our country.

Caring for active duty military and veterans is in the DNA of nurses. Florence Nightingale spent much of her post-Crimea life using evidence-based proposals and political influence to improve the health care of the soldiers and veterans of the British Empire. In Notes on Nursing, she spurred nurses to political action: “Let whoever [sic] is in charge keep this simple question in her [sic] head (not how can I always do this right thing myself, but) how can I provide for this right thing to be always done?7 This advice should be taken to heart by all health care professionals: We can honor our veterans by advocating for and providing the health care access they need.

To share your thoughts, please contact us at [email protected]

References

1. Advanced practice registered nurses [2016-12338]. Fed Regist. May 25, 2016. https://federalregister.gov/a/2016-12338.
2. American Association of Nurse Practitioners. AANP and Air Force Sergeants Association urge VA to swiftly enact proposed rule. July 25, 2016. www.aanp.org/legislation-regu lation/federal-legislation/va-proposed-rule/173-press-room/2016-press-releases/ 1987-aanp-and-air-force-sergeants-associa tion-urge-va-to-swiftly-enact-proposed-rule. Accessed August 9, 2016.
3. American Association of Nurse Practitioners. AANP and veterans groups call for streamlined access to veteran’s health care. June 23, 2016. www.aanp.org/press-room/press-releases/173-press-room/2016-press-releases/1959-aanp-veteran-groups-call-for-streamlined-access-to-veterans-health-care. Accessed August 9, 2016.
4. American Association of Nurse Practitioners. National survey finds overwhelming support for VA rule granting veterans direct access to nurse practitioner care. July 20, 2016. www.aanp.org/press-room/press-releases/173-press-room/2016-press-releases/1986-national-survey-finds-overwhelming-support-for-va-rule-granting-veterans-direct-access-to-nurse-practition er-care. Accessed August 9, 2016.
5. American Association of Nurse Anesthetists. Nursing coalition and veterans groups join forces in unprecedented response to VA proposed rule to increase veterans’ access to care. June 28, 2016. www.aana.com/newsandjournal/News/Pages/062816-Nursing-Coalition-and-Veterans-Groups-Join-Forces-in-Unprecedented-Response-to-VA-Proposed-Rule.aspx. Accessed August 9, 2016.
6. Wergin RL. Letter to David Shulkin. July 13, 2016. www.aafp.org/dam/AAFP/docu ments/advocacy/workforce/scope/LT-VHA-APRN-071316.pdf. Accessed August 9, 2016 .
7. Nightingale F. Notes on Nursing: What It Is and What It Is Not. New York, NY: D. Appleton and Company; 1860.

References

1. Advanced practice registered nurses [2016-12338]. Fed Regist. May 25, 2016. https://federalregister.gov/a/2016-12338.
2. American Association of Nurse Practitioners. AANP and Air Force Sergeants Association urge VA to swiftly enact proposed rule. July 25, 2016. www.aanp.org/legislation-regu lation/federal-legislation/va-proposed-rule/173-press-room/2016-press-releases/ 1987-aanp-and-air-force-sergeants-associa tion-urge-va-to-swiftly-enact-proposed-rule. Accessed August 9, 2016.
3. American Association of Nurse Practitioners. AANP and veterans groups call for streamlined access to veteran’s health care. June 23, 2016. www.aanp.org/press-room/press-releases/173-press-room/2016-press-releases/1959-aanp-veteran-groups-call-for-streamlined-access-to-veterans-health-care. Accessed August 9, 2016.
4. American Association of Nurse Practitioners. National survey finds overwhelming support for VA rule granting veterans direct access to nurse practitioner care. July 20, 2016. www.aanp.org/press-room/press-releases/173-press-room/2016-press-releases/1986-national-survey-finds-overwhelming-support-for-va-rule-granting-veterans-direct-access-to-nurse-practition er-care. Accessed August 9, 2016.
5. American Association of Nurse Anesthetists. Nursing coalition and veterans groups join forces in unprecedented response to VA proposed rule to increase veterans’ access to care. June 28, 2016. www.aana.com/newsandjournal/News/Pages/062816-Nursing-Coalition-and-Veterans-Groups-Join-Forces-in-Unprecedented-Response-to-VA-Proposed-Rule.aspx. Accessed August 9, 2016.
6. Wergin RL. Letter to David Shulkin. July 13, 2016. www.aafp.org/dam/AAFP/docu ments/advocacy/workforce/scope/LT-VHA-APRN-071316.pdf. Accessed August 9, 2016 .
7. Nightingale F. Notes on Nursing: What It Is and What It Is Not. New York, NY: D. Appleton and Company; 1860.

Issue
Clinician Reviews - 26(9)
Issue
Clinician Reviews - 26(9)
Page Number
11,52
Page Number
11,52
Publications
Publications
Topics
Article Type
Display Headline
“Unprecedented” VA Proposal? We Don’t Think So
Display Headline
“Unprecedented” VA Proposal? We Don’t Think So
Sections
Disallow All Ads
Article PDF Media

When Man’s Legs “Give Out,” His Buttocks Takes the Brunt

Article Type
Changed
Mon, 07/09/2018 - 10:49
Display Headline
When Man’s Legs “Give Out,” His Buttocks Takes the Brunt

ANSWER

There are degenerative changes present. Bilateral hip prostheses are noted. Within the coccyx, there is bone remodeling and angulation that are likely chronic and related to remote trauma or injury (arrow). Below this, some cortical lucency (circled) is noted, most likely consistent with an acute fracture. The patient was prescribed a nonsteroidal medication and a mild narcotic pain medication.

Article PDF
Author and Disclosure Information

Nandan R. Hichkad, PA-C, MMSc, practices at the Georgia Neurosurgical Institute in Macon.

Issue
Clinician Reviews - 26(9)
Publications
Topics
Page Number
39,53
Sections
Author and Disclosure Information

Nandan R. Hichkad, PA-C, MMSc, practices at the Georgia Neurosurgical Institute in Macon.

Author and Disclosure Information

Nandan R. Hichkad, PA-C, MMSc, practices at the Georgia Neurosurgical Institute in Macon.

Article PDF
Article PDF
Related Articles

ANSWER

There are degenerative changes present. Bilateral hip prostheses are noted. Within the coccyx, there is bone remodeling and angulation that are likely chronic and related to remote trauma or injury (arrow). Below this, some cortical lucency (circled) is noted, most likely consistent with an acute fracture. The patient was prescribed a nonsteroidal medication and a mild narcotic pain medication.

ANSWER

There are degenerative changes present. Bilateral hip prostheses are noted. Within the coccyx, there is bone remodeling and angulation that are likely chronic and related to remote trauma or injury (arrow). Below this, some cortical lucency (circled) is noted, most likely consistent with an acute fracture. The patient was prescribed a nonsteroidal medication and a mild narcotic pain medication.

Issue
Clinician Reviews - 26(9)
Issue
Clinician Reviews - 26(9)
Page Number
39,53
Page Number
39,53
Publications
Publications
Topics
Article Type
Display Headline
When Man’s Legs “Give Out,” His Buttocks Takes the Brunt
Display Headline
When Man’s Legs “Give Out,” His Buttocks Takes the Brunt
Sections
Questionnaire Body

A 75-year-old man presents to the urgent care center for evaluation of pain in his buttocks after a fall. He states he was walking when his “legs gave out” and he hit the ground. He landed squarely on his buttocks, causing immediate pain. He was eventually able to get up with some assistance. He denies current weakness or any bowel or bladder complaints.

His medical/surgical history is significant for coronary artery disease, hypertension, and bilateral hip replacements. Physical exam reveals an elderly male who is uncomfortable but in no obvious distress. His vital signs are stable. He has moderate point tenderness over his sacrum but is able to move all his extremities well, with normal strength.

Radiograph of his sacrum/coccyx is shown. What is your impression?

Disallow All Ads
Article PDF Media